Vous êtes sur la page 1sur 53

Práctica 1 - Espacios Vectoriales

1. Demuestre que R| n (C
| n ) es un espacio vectorial sobre |R (C
| ) con la suma y el producto

por un escalar usuales.


| n un R
2. ¿ Es C | -espacio vectorial con la suma y el producto por un escalar usuales?

3. Compruebe que el conjunto de matrices de orden p × q a coeficientes reales (comple-


jos) |Rp×q (C
| p×q ) es un espacio vectorial real (complejo) con la suma y el producto

por un escalar usuales.

4. Probar que el conjunto de polinomios a coeficientes reales P es un R


| -espacio vectorial

con la suma y el producto por un escalar usuales.

5. Probar que el conjunto de funciones continuas a valores reales definidas en el inter-


valo [a, b], que denotamos C[a, b], es un |R-espacio vectorial con la suma y el producto
por un escalar usuales.

6. Suponga que A ∈ R| p×q y que b ∈ |Rp . ¿Qué condición debe cumplir b para que el
| q : Ax = b} sea un subespacio de R
conjunto S = {x ∈ R | q?

7. Empleando la respuesta al ejercicio anterior determine cuales de los siguientes con-


juntos son subespacios de los espacios que se indican.
| 3 : 2x + x + x = 0 ∧ x − x = 0}.
a) S = {x ∈ R 1 2 3 2 1
| n : x = x = · · · = x = 0}.
b) S = {x ∈ R 1 2 r
| 4 : x + x = 0 ∧ x − 2x + x = 1}.
c) S = {x ∈ R 1 2 1 3 4

8. Demuestre que Pn , el conjunto formado por el polinomio nulo y por los polinomios
de grado menor o igual a n, es un subespacio de P.

9. Determine cuales de los siguientes conjuntos son subespacios de los espacios vecto-
riales que se indican.
| 3 : v = [1 + r r 4r]T , r ∈ R
a) S = {v ∈ R | }.

| 2 : v = [r 2r]T , r ≥ 0}.
b) S = {v ∈ R
| 2×2 : A es singular}.
c) S = {A ∈ R
d ) S = {p ∈ P3 : p(1) = p(2)}.
e) S = {f ∈ C[0, 1] : f (0) = f (1) = 0}.
| ) : [f (x)]2 = f (x)}.
f ) S = {f ∈ C(R

1
10. Sean S1 y S2 subespacios de un espacio vectorial ¿ Qué condición debe cumplirse
para que S1 ∪ S2 también sea subespacio?
| 3 está generado por los vectores v = [1 − 1 2]T , v = [−1 0 3]T ,
11. Determine si R 1 2
T T
v3 = [0 − 1 5] y v4 = [3 − 2 2] .
12. Encuentre los valores de k ∈ R | para que los vectores [1 k 0]T , [1 k − 1 k]T y
[2 2k − 1 k 2 + k + 1]T sean linealmente dependientes en R| 3.

13. Determinar cuales de los siguientes conjuntos son linealmente independientes en el


espacio vectorial que se indica.
a) {[1 i]T , [i − 1]T } en C
| 2 como R
| -espacio vectorial y como C
| -espacio vectorial.

b) {2, 3 + t, 2 − t2 } en P.
c) {1, 2 + 2t, 1 − t + t2 , 2 − t2 } en P.
d ) {sen(x), cos(x)} en C(R
| ).

e) {sen2 (x), cos2 (x), 1} en C(R


| ).

14. ¿ Cuál es la dimensión de cada uno de los espacios vectoriales mencionados en los
ejercicios 1 a 5? ¿ Qué dimensión tiene Pn ?
15. Probar que si V es un C| -espacio vectorial de dimensión n entonces V es un |R-espacio

vectorial de dimensión 2n.


16. Encuentre bases para los siguientes subespacios:
| 4 : x + 2x + x = 0 ∧ 2x − x + x = 0}.
a) S = {x ∈ R 1 2 4 1 2 4

b) S = {p ∈ P3 : p(0) = p(1)}.
R1
c) S = {p ∈ P4 : 0 p(t)dt = 0}.
d ) S = {A ∈ |R3×3 : A = AT }.
En cada caso, extienda la base del subespacio que obtuvo a una base del espacio
vectorial correspondiente.
17. Encuentre una base del subespacio generado por las matrices
" # " # " # " #
1 −1 −1 2 2 −3 1 1
, , y .
−1 2 3 1 −3 2 1 6

18. Para cada una de las siguientes matrices A, halle bases de los subespacios funda-
mentales: Col(A), Nul(A), Fil(A) y Nul(AT ). Compare sus dimensiones. Calcule
rango(A) y rango(AT ).
 
  1 −1 1
1 −4 9 −7 " #
 −2 1 0  −1 1 0 2
   
i) A =  −1 2 −4 1 , ii) A =   , iii) A = .
 3 1 −4  1 1 −4 1
5 −6 10 7
1 −2 0

2
19. Sea A ∈ K n×m (K = R | ó C
| ), A = [u1 u2 · · · um ] con ui la i-ésima columna de A.

Deducir, a partir del hecho que


Ax = x1 u1 + x2 u2 · · · + xm um , si x = [x1 · · · xm ]T ,
lo siguiente:
(a) b ∈ Col(A) si y sólo si existe x tal que Ax = b.
(b) La ecuaciónAx = b tiene a lo sumo una solución si y sólo si las columnas de A
son linealmente independientes.
(c) La ecuación Ax = b tiene a lo sumo una solución si y sólo si rango(A) = m.
(d) La ecuación Ax = 0 admite solución no trivial si y sólo si las columnas de A
son linealmente dependientes.
(e) La ecuación Ax = b tiene solución para todo b si y sólo si rango(A) = n.
(f) La ecuación Ax = b tiene solución si y sólo si A y la matriz ampliada à = [A b]
tienen igual rango.
20. Sean A ∈ K n×m (K = R
| ó C
| ), A = [u1 u2 · · · um ] con ui la i-ésima columna de A y
r×n
B ∈ K . Explicar, a partir del hecho que
BA = [Bu1 Bu2 · · · Bum ],
por qué Col(BA) ⊆ Col(B). Dar ejemplos no triviales (A 6= I, 0, B 6= I, 0) en los
cuales se cumpla la inclusión estricta y otros en donde valga la igualdad.
| 4 , S = gen{[1 1 2 0]T , [2 0 3 − 1]T } y S = {x ∈ R
21. Dados los subespacios de R | 4 :
1 2
x1 + x2 + x3 + x4 = 0}, hallar bases de S1 ∩ S2 y de S1 + S2 .
22. Sean S1 y S2 subespacios de un espacio vectorial V de dimensión finita ¿ Qué relación
existe entre las dimensiones de S1 , S2 , S1 ∩ S2 y S1 + S2 ?
23. Demuestre que S1 ⊕ S2 = R | 3×3 con S = {A ∈ R| 3×3 : A = AT } y S = {A ∈ R
| 3×3 :
1 2
A = −AT }. ¿ Es cierta la igualdad precedente en R
| n×n ?

| 5,
24. i) Determinar si la suma de los siguientes subespacios de R
S1 = gen{[1 1 2 0 1]T , [2 0 3 0 1]T }, S2 = gen{[−1 1 −2 1 1]T }, S3 = gen{[0 1 0 1 1]T },
es directa y hallar una base del mismo.
ii) Idem anterior pero con
S1 = gen{[1 1 2 0 1]T , [2 0 3 0 −1]T }, S2 = gen{[−1 0 −2 1 1]T }, S3 = gen{[1 1 1 2 2]T }.

25. Suponga que {v1 , v2 }, {v3 , v4 , v5 } y {v6 , v7 } son, respectivamente, bases de los sube-
spacios S1 , S2 y S3 de un espacio vectorial V . Demuestre que B = {v1 , v2 , . . . , v7 }
genera S1 + S2 + S3 y que la suma resulta directa si y sólo si B resulta base. Gen-
eralize.

3
26. Encuentre las coordenadas de v en la base ordenada B en cada uno de los siguientes
casos:

a) v = [1 2 3]T y B = {[1 1 0]T ; [1 0 1]T ; [0 1 1]T }.


b) v = a + bt + ct2 y B = {1 + t + t2 ; 1 + t; 1}.

27. Sea B = {v1 ; v2 ; . . . ; vn } una base ordenada del K-espacio vectorial (K = R


| ó C
| ) V
n
y sea cB : V → K la aplicación que asigna a cada v ∈ V su correspondiente vector
de coordenadas en la base B, cB (v) ∈ K n .

a) Demuestre lo siguiente:
1) cB (v + v 0 ) = cB (v) + cB (v 0 ) y cB (αv) = αcB (v) para todo v, v 0 ∈ V y para
todo α ∈ K.
2) cB es biyectiva.
3) {u1 , . . . , ur } es l.i. en V si y sólo si {cB (u1 ), . . . , cB (ur )} es l.i. en |Rn .
b) Hallar la expresión de c−1
B .

28. Resolver los ejercicios 16 y 17 trabajando en coordenadas respecto de una base B a


elección.
| n y B = {v ; . . . ; v } una base ordenada
29. Sean E = {e1 ; . . . ; en } la base canónica de R 1 n
| n . ¿ Cuál es la expresión de la matriz de cambio de coordenadas de la base B
de R
a la base E, CBE ?

30. Supongamos que B, B 0 y B 00 son tres bases ordenadas del espacio vectorial V . ¿Cómo
puede obtenerse CBB 00 a partir de CBB 0 y CB 0 B 00 ?

31. Hallar la matriz de cambio de bases CBB 0 en los siguientes casos

a) B = {[1 2 3]T ; [1 0 1]T ; [3 4 6]T } y B 0 = {[1 − 1 0]T ; [1 − 2 3]; [1 1 0]T }.


b) B = {1; t − 1; (t − 1)2 } y B 0 = {1; t − 2; (t − 2)2 }.

4
Adicionales Práctica 1 - Wronskiano

1. Halle el wronskiano de los siguientes conjuntos de funciones y con su auxilio, cuando


sea posible, determine si son linealmente independientes.

(a) 1, x, . . . , xn .
(b) x, x1
(c) eαx , xeαx , (α ∈ R).
(d) ex , xex , e−x .
(e) sen x, sen(x + π4 ).
(f) arccos(x), arcsen(x).
(g) 1, sen2 x, cos 2x.
(h) sen x, cos x, cos(x + π3 ).
(i) e−3x sen 2x, e−3x cos 2x.
(j) senh(αx), cosh(αx), (α ∈ R).
2 −1
2. Hallar el wronskiano de las funciones 1, ex , ex , cos x, ex−1 .

3. Probar que el conjunto de funciones {x2 , x|x|} es linealmente independiente a pesar


de que su wronskiano es idénticamente nulo.

4. Suponga que f1 y f2 son funciones derivables en un intervalo abierto I. Sean


g1 = c11 f1 + c21 f2 y g2 = c12 f1 + c22 f2 con cij ∈ R. Pruebe que
¯ ¯
¯ c11 c12 ¯
¯
W (g1 , g2 )(x) = ¯ ¯ W (f1 , f2 )(x) ∀x ∈ I.
c21 c22 ¯

Generalice al caso en que se tienen funciones f1 , . . . , fn , que son n−1 veces derivables
en I y funciones g1 , . . . , gn que son combinaciones lineales de las funciones fi .

1
  
             

  !#"%$#&'!)(+*,&-&%.01 / !#2$#.435&'!)(+*,&6.738:9;&'"%(<3*<&%8>=#&@?ACB


DFEHG IKJL'MON P Q EHG IRSJUTVMWNYX
"%3!#8<Z4=#&'* 1 !#=#3@[#*<Z4\]&%*<36&'.^[#*,3_=`$#"%(<3@Z4!a(<&'*,!#3-" 1 ! 3/ !#Z4"'3cb D X+Q_d EeD N QPf.4$#&'2;30&%.^[#*<3g=#$#"h(+3
Z4!a(<&'*,!#3ibj"'3;\][#*,$C&'/ kl&'.43admb D X+Q_d E T Don Q np I D^q Q qrpsD B Q B 
I_  !#"%$#&'!)(+*,&](<3_=#3;80.438t9&%"%(<3*<&%8-=#&c?A B0u $#&]8,3!v3;*,(+3;23! 1 .4&'8 1xw EyG IzJe'M N "'3;!{&%.
[#*,3_=`$#"%(<3|Z4!)(+&'*,!#3|" 1 ! 3;/ !#Z7"%3#~}€t$C&6/ "%. 1 8,&m=#&6"'3!‚,$`!a(<3|ƒ„3;*<\ 1 !^…
T`†b 1 dˆ‡g& 1|‰ $#!x&%8<[ 1 "%Z739;&'"%(<3*<Z 1 .Š"%3!x[#*,3_=#$`"%(+35Z4!)(+&'*,!#3#‹gPc8,& 1wŒŽ‰ t&%\$#&%8,(+*,& u $#&
&%."'3!‚,$`!a(<3 ‰‘ ‹"'3;\][#$`&'8,(<3i[l3*m.438t9&%"%(+3;*<&'8@=#& ‰ u $`&|8,3!’3*,(<323! 1 .7&'8 1xw ‹Y&%8
$`!i8,$#ko&%8<[ 1 "'Z73]=#& ‰ 
bjk‘dt&%\$#&%8,(+*,& u $#&0&'!ˆ&'.“" 1 8<3]&'! u $#& ‰ E ?AC”ˆb;3/ • A ”dh‹ w—E™– ˜ P&'.“[#*,3_=`$#"%(<3]Z4!a(<&'*,!#3
&%8š&%.“" 1 ! 3/ !#Z7"%3#‹ ‰‘ &%8 $#!i8,$#kl&'8<[ 1 "'Z43]=#&6=#Z4\]&%!#8<Z 3/ !x›Jœ
 †b 1 d !#"%3!)(+* 1 * . 1 8C"'3!`=#Z7"%Z73!#&%8 u $#&t=#&%ko&%!"'$#\|[#.7Z4*C.438ž"%3_&hŸ‘"'Z4&'!)(+&%8   n¡n ‹#  n¢q ‹#  q£n PF  q¡q
[ 1 * 1 u $#&6. 1 &h¤_[#*,&'8<Z 3/ !
b D X+Q_d E   n¡n<Don Q n€p   n¢q¥Don Q qrp   q£n<D^q Q n€p   q¡q¥D^q Q q b£¦d
=`&%Ÿ‘! 1 $#!ˆ[#*<3g=#$#"h(+3fZ7!)(+&%*<!#3|&'!§?A q 
bjk‘dˆ¨ž3;\][#*,$#&'kl& u $#&qh. 1 ª;q &h¤_[#*,&'8<Z 3/ !sb£¦dš[#$#&%=#&&'8,"'*,Z7k#Z4*<8<&-&'!ƒ„3*<\ 1 "%3\|[ 1 "%( 1 b D X+Q_d E
D[`$#Nl!)© (+Q|3 "%3!a! (<&'© *,Z73Œ *:?A&%!ˆ(_&'/ *,\]}“¨xZ4!#3/3\|8 3=#&6[#$#. &%=#\ &'!(<&«*<¤`Z7­ [`©*<&'… 8 1 *<8,&6. 1 8C"'3!#=`Z7"'Z43!#&%8 u $#&m¬ 1 .4.¦30/ &'!x&%.
1 1 1
®_š‡_& 1t‰ $#!&'8,[ 1 "%Z73š9;&'"h(+3*,Z 1 .a"'3;![#*<3g=#$#"h(+3šZ7!)(+&%*<!#3 P@8,& 1]¯a°£¯ . 1 !#3*,\ 1 Z7!`=#$#"'Z4= 1 €± *<3k 1 *
.4358<Z72;$#Z7&%!a(<&;²
b 1 d³ ¯ D ¯ J ¯ Q ¯ ³a´ ¯ D JUQ ¯ [ 1 * 1 (+3g=#3 D PxQ]&'! ‰ 
bjk‘dˆ‡gZ D PxQF8,3!ˆ3*,(<323! 1 .7&'8ž&'!)(+3!`"'&'8 ¯ D5p Q ¯ q E ¯ D ¯ q p ¯ Q ¯ q bj±ZO(^1 / 2;3* 1 8+dh
µ 38,(<* 1 * u $#&69 1 .7&m. 1 *,&'";¶/ [#*,3_" 1 8,Z“&'.“&%8<[ 1 "%Z73f9&%"%(+3;*<Z 1 .·&'8 *<& 1 .¢
bj" d ¯ D0p Q ¯ q p ¯ D JŽQ ¯ q E I`b ¯ D ¯ q p ¯ Q ¯ q d¸ D X<Ql bj 8,( 1 Z4=#&'!)(<Z7= 1 =c&'8 "'3;!#3_"%Z7= 1 "'3\|3
. 1 .7&%Pc=#&'.“[ 1 * 1 .4&'.432* 1 \]3#¹d
bj=‘dˆ‡gZ ‰ &%8š$`!?Aº¡&'8,[ 1 "%Z7359&%"%(+3;*<Z 1 .¢‹
b D X+Q_d E  b ¯ D-p Q ¯ q J ¯ D JUQ ¯ q d ¸ D X+Q^»
bj¼53/ *,\$#. 1 =`&0[l3. 1 *<Z4­ 1 "%Z¦3/ !^d«

bj& di‡gZ ‰ &%8š$`!’• A º½&'8,[ 1 "'Z4359&'"h(+3*,Z 1 .¾‹
b D X+Q_d E  b ¯ D5p Q ¯ q J ¯ D JQ ¯ q dJÀ ¿ b ¯ Dp ¿ Q ¯ q J ¯ D J ¿ Q ¯ q d ¸ D X+Q^»
bj¼53/ *,\$#. 1 =`&0[l3. 1 *<Z4­ 1 "%Z¦3/ !^d«
Á`š‡_& 1f $#! 1 k 1 8,&03;*<=#&%! 1 = 1 =#&m$`!ˆ&%8<[ 1 "'Z7359;&'"%(<3*<Z 1 .·*,& 1 .bj3f"'3\|[#.4&Á,3ad ‰ =#&m=#Z7\|&'!`8<Z¦3;/ !
›€‹#Px8<& 1]ÄhÅ &%.“Z78<3;\]3*£Ÿ‘8<\|35=#&6"'3g3*,=#&'! 1 = 1 8'
b 1 dˆ‡g& 1 b ° X ° dž$#!ˆ[#*<3g=#$#"h(+3fZ7!)(<&'*<!`3]&%!?A~”ˆbj3f&'!Ž• A ”dh‹`Px8,& 1
b D X+Q_d,Æ E b ÄhÅ b D d«X Ä%Å b„Q_d<d ¸ D X+Q Œ‰ »
t&%\$#&%8,(+*,& u $#&]b ° X ° d,Æ{&%8 $#!i[`*<3g=#$#"%(<3fZ7!)(+&%*<!#3|&'! ‰ 
bjk‘dt&%\$#&%8,(+*,& u $#&68,Z~b ° X ° d,ƒ&'8 $#!ˆ[#*<3g=#$#"h(+3|Z7!)(+&%*<!#3f&%! ‰ ‹‘&'!)(+3!`"'&'8
b D X+Q_d E b Ä ÅÇ n b D d«X Ä ÅÇ n bjQ_d,d,Æ ¸ D X+Q Œ ?A ” b¡• A ” d
&%8š$`!i[#*,3_=`$#"%(<3|Z4!)(+&'*,!#3|&'!§?AC”ˆb„3|&%!’• A ”dh
bj" dˆr¤_[#.4Z u $#&6&'!ˆ[ 1 . 1 k#* 1 8 u $`&@8,Z72!#ZOŸ‘" 1 !’b 1 dCP’b„k^dh
È  t&%\$#&'8£(+*,& u $#&0&'!É q ‹
bj   p   n,ʓp   q+Ê q X+Ë  p Ë n,ʓp Ë q+Ê q d E    Ë  p   n Ë nrp   q Ë q
=#&hŸ‘!#&0$#!ˆ[#*<3g=#$#"h(+3fZ7!)(+&%*<!#35( 1 . u $#&6. 1 k 1 8,&  EÌ ;Í Ê Í Ê q Î &'8 3*,(<3!#3*,\ 1 .¢
Ï`†b 1 dt&%\$#&%8,(+*,& u $#&
b¾ÐlX<Ñ`d E™Ò  n Ðb Ê d¡ÑŠb Ê doÓ Ê
&%8 $#![#*<3g=#$#"h(+35Z4!a(<&'*,!#35&'!cÔ G՘ X''M¢ 8,"'*<Z4k 1 . 1 =#&'8,Z72$ 1 .7= 1 =c=#&0‡_"¥¬)Ö 1 *<­6[ 1 * 1 &'8£(+&
" 1 8<3`
bjk‘dt&%\$#&%8,(+*,& u $#&:&%._[#*,3_=`$#"%(<3>Z7!)(<&'*<!`3t=#&%Ÿ#!#Z7=#3>&'!&%._[#$`!a(<3 1 !)(+&%*<Z43*'‹VZ7!#=#$`"'&ž$`![#*<3‚º
=`$#"%(<3]Z4!a(<&'*,!#3f&'!xÉ ” ͑2&'!#&%* 1 .4Z7"%&t&'8£(+&6*<&%8<$#.O( 1 =#3f[ 1 * 1 8<$`ko&%8<[ 1 "%Z738 ‰‘ =`&m&'8<[ 1 "'Z438
9;&'"h(+3*,Z 1 .4&'8 ‰ u $`&0(<Z7&'!`&'!$#!ˆ[#*<3g=#$#"h(+3|Z7!)(+&%*<!#3#
bj" di¨ 1 .7"'$`.7&ˆq &'.i1 / !#2$`.73Ž&'!)(+*,& Ê P Ê q J ÊCp L× 1 .7.4&ˆ  Œ ?A [ 1 * 1 u $#&ЊØmÑl‹ž8,Z7&%!#=#3
Ðrb Ê d ERÊ p   Ê PcÑ·b Ê d ERÊ JL
Ù`š‡_& 1U‰ $#!L?A º½&'8,[ 1 "'Z43{9;&'"%(<3*<Z 1 . "'3!L[#*,3_=`$#"%(<3Z7!)(+&%*<!#3’P  E Ì Q n Í+Q q Í+Q B Î $`! 1 k 1 8,&
3*,=#&'! 1 = 1 =#& ‰ 
b 1 dt&%\$#&%8,(+*,& u $#&]b D X<Q`d E Ä%Å b D d NlÚ Ä%Å bjQ_dž"%3!
bjQ n X+Q n d bjQ n X+Q q d b„Q n X+Q B d ßÕà
Ú EÜÛÝÞ bjQ q X+Q n d bjQ q X+Q q d b„Q q X+Q B d á »
bjQ B X+Q n d bjQ B X+Q q d b„Q B X+Q B d
I
â . 1 \ 1 (<*<Z4­ Ú 8<&>. 1 =#&'!#3;\]Z4! 1 \ 1 (+*,Z7­ =#&%.o[#*,3_=`$#"%(<3Z7!)(+&%*<!#3-&'!F. 1 k 1 8,&>3*,=#&'! 1 = 1
 NŠxÚ ãš3;(<& u [$#& * Ú "'&'$ 8@. 8<u Z4\ä$#Z7&h&%/ (+* *<Z4" 1 ‹“?A&%8@B =`=`&'Z78,"'(<Z4*'Z7!)‹ (+Ú 3f=#E &6"%Út&'N *<3`‹Y PŽ=#&hŸ‘!#Z4= 1 [l38,Z4(+ZO9 1 ‹“&'80=#&'"%Z7*%‹
å}ot$žå& / ƒ„æL3;*<˜\ 1 = 1 u $#Z7&%1 *<& Ú 8<Z^å . Œ k 8<& &'8~3*£(+323;! .ç…}‘P58<Z^. k 8<& &'8~3*,(<3!#3*,\ .è…
é &'";¶/ [#*,3_" 1 \|10&'1 !)(+&V‹‘[#*<$#&%ko& u $#1 &0= 1 1 = 1  $#! 1 \ 1 (+*<Z4­ Ú1 8<Z4\ä&h/ (+*,Z7" 1 1 PF1 =#&%Ÿ‘!`Z7= 1 [o3;8<Z4(<Z491 1 ‹
b D X+Q_d E Ä%Å b D d N Ú Ä%Å b„Q`dž&%8š$`!i[#*,3_=`$#"%(<3|Z4!)(+&'*,!#3f&'! ‰ 
bjk‘dˆêm&'!`&'* 1 .7Z7"%&F&'.C[#$#!)(+3 1 !)(<&'*<Z43* 1 .ž" 1 8<3&'! u $#&x. 1 k 1 8,&  &'8,(‘1 / "'3\|[#$#&%8,( 1 [l3*ë
9;&'"h(+3*,&'8'‹l&'8 =#&%"'Z4*'‹ 1 .·" 1 8<3|&'! u $#& ‰ 8,& 1 =#&6=#Z7\|&'!`8<Z¦3;/ !xë_
bj" di}t& u $ž&m/ ƒ„3*<\ 1 8<3!(+3g=#38 .438 [o3;8<Z7k`.7&'8 [`*<3g=#$#"%(<38 Z7!)(+&%*<!#3;8 &'!?A ”…
bj=‘dˆ¨ž3;!#8<Z4=#&'*,& ‰ E É q ‹  EìÌ Í Ê Í Ê q Î Pc&'.Š[`*<3g=#$#"%(<3fZ7!)(+&%*<q !#3=`&%Ÿ‘!#Z4=#35&'!&%.Š&Á,&%*<"%Z7"'Z43
Ï_C× 1 .7.4&t. 1 \ 1 (<*<Z7­ Ú Px[l3* \]&%=#Z735=#&6&%.7. 1 " 1 .7"%$#.7&5b Ê X Ê J ʓp ídh
bj& d µ 3_=#ZOŸ‘" 1 !#=#3 1 =#&'"%$ 1 = 1 \|&'!)(<&>.73;8~&%!a$#!#"'Z 1 =#38%‹`[#*,$#&'kl&-b 1 dPibjk^d &'!c&'.^" 1 8,3-&%! u $#&
‰ 8<& 1 $`!i&%8<[ 1 "%Z7359&%"%(<3*<Z 1 .“"'3\|[#.4&Á,3#
b„ƒ+d× 1 .7.4&>. 1 \ 1 (+*<Z4­ Ú &'!&'.o" 1 8,3f&'! u $#& ‰ E • A B ‹  EîÌ_G  ¿ ˜ M N Í G UU'M N Í G ¿ ˜ä˜ M N Î
PF&'.“[#*,3_=#$`"%(+3|Z7!)(<&'*<!`3|&%8š&%.“" 1 ! 3/ !#Z7"%3ib<b D X+Q_d ERDoï Q_dh
 ˜ š‡_$`[o3!`2 1 u $#&šb ° X ° do&%8“$#!6[#*<3g=#$#"h(+3 Z7!)(+&%*<!#3 &%!?A B ( 1 . u $#&  EîÌ_G 06J@'M N Í Gð˜ 6'M N Í G J@0 ˜ M N Î
&%8 $`! 1 k 1 8<&63*£(+3!#3;*<\ 1 .¢
b 1 dˆ¨ 1 .7"'$`.7&|bjQ n X+Q q d:"'3!Q nžEñG œJL6'M N PxQ q:EG J@>I0I‚M N 
bjk‘d× 1 .7.4&t. 1 \ 1 (+*,Z7­t=`&'.“[#*,3_=#$`"%(+3fZ4!a(<&'*,!#3]&%!i. 1 k 1 8,&@" 1 ! 3/ !#Z4" 1 
 ± *<$`&'kl& u $#&f$#! 1 \ 1 (<*<Z7­ Ú =#&hŸ‘!#Z7= 1 [l38,Z4(+ZO9 1 &'8mZ4!a9;&'*,8<Z7k`.7&;Fb¢‡_$`2&'*,&'!#"%Z 1 ²m[#*,$#&'kl& u $#&
. 1 &'"'$ 1 "'Z 3/ ! Ú åER˜ (<Z7&%!#&m8<3.4$#"'Z 3/ !î$#/ !#Z4" 1 ¹d
t&%=#$#­'" 1 =`&m.73 1 !)(+&%*<Z43* u $#&6. 1 \ 1 (+*,Z7­š=#&6$#![#*<3g=#$#"h(+3fZ7!)(+&%*<!#3f&%!ˆ$`! 1 k 1 8<&m3*,=#&'! 1 = 1
&%8 8,Z7&%\][#*,&mZ7!)9&%*<8,Z7k#.4&;
íI_š‡_& 1 Ì Q n X °¦°¦° X+Qíò Î $#!i"%3!‚,$#!)(<3]3*£(+32;3! 1 .o=#&'.“&%8<[ 1 "%Z73f9&%"%(+3;*<Z 1 . ‰ ~t&'\$`&'8,(<*<& u $#&
¯hó n Q np °¦°¦° p ó ò+Qíò ¯ q E ³ ó n ³ q ¯ Q n ¯ q p °¦°¦° p ³ ó òV³ q ¯ Qíò ¯ q »
¦T`š¨ž3\|[#*,$#&'kl& u $#&  EìÌ Í Ê J qn Í Ê q J ʦp ôn Î &%8$#! 1 k 1 8<&:3;*,(+3;23! 1 .a=#&~qÉ B "'3;!5&'.g[#*<3g=#$#"h(+3
Z4!a(<&'*,!#3]=`&%Ÿ‘!#Z4=#3c&%!&'.&¡,&'*<"%Z7"%Z73]Ï` × 1 .7.7&m. 1 8 "'3g3*<=#&%! 1 = 1 8š=#& õ E™Ê JL@&'!i. 1 k 1 8<& Â
Px.4$#&'2;3|" 1 .7"%$#.7& 1 [ 1 *,(+Z4*:=#&&'/ 8£( 1 8 . 1 !`3*<\ 1 =#& õ“
  š‡_& 1 ! DxEHG Lö'M N PFQ EHG sI™JL%M N ž8,"'*,Z7k 1 D "'3;\]3f. 1 8,$#\ 1 =#&6=#38 9&%"%(<3*<&%8'‹l$#!#3
[ 1 * 1 .4&'.73 1 Q|Px3;(+*,3f3*,(<323! 1 . 1 Ql b¾¨ž3!#8,Z7=#&%*<&6&'.“[#*,3_=`$#"%(<3]Z4!a(<&'*,!#3f" 1 ! 3;/ !#Z7"%3adh
í®_ × 1 .7.4&ˆ&%.š[`$#!)(+3U=#&ˆ÷ø\S1 / 8]"%&'*," 1 !#3 1 Q Œ—‰ Ps" 1 .7"'$`.7&ˆÓŠb„Q^X«ùdf&%!œ" 1 = 1 $#!#3=`&§.73;8
8,Z72$#Z4&'!)(+&%8 " 1 8,38'²
b 1 dF÷ E 2;&'! Ì_G iˆ'M N X G iJi ˜ M N Î ‹)Q EHG  ˜i˜ M N P&'.‘[`*<3g=#$#"%(<36Z7!)(+&%*<!#36&'8 &%.‘&'8£(^1 / !#= 1 *
=`&?A B¦
T
bjk‘dF÷ E 2;&'! Ì_G ¿ JœS p ¿ M N Î ‹‘Q EHG  öË Ä M N Px&%.“[#*<3g=#$#"h(+3|Z7!)(+&%*<!#3f&%8 &%.“&'8£(^1 / !#= 1 * =#&
•A B 
bj" dx÷ E 2&'! Ì X Ê J qn Î ‹ Q EzÊ q pRÊp cP&%.ž[#*,3_=#$`"%(+3Z4!)(+&'*,!#3&'8-&'.:=#&hŸ‘!#Z4=#3&'!U&%.
&¡,&'*,"'Z7"%Z73|Ï`
¦Á`š‡_& 1xúYû . 1 [#*,3¦P&'"%"'Z 3/ !Ž3*,(<323! 1 .r8<3;k#*<&5&%.r8<$`ko&%8<[ 1 "%Z73x÷î=#&%.&'8,[ 1 "%Z73F9&%"%(<3*<Z 1 . ‰ 0t&«º
\$`&'8,(<*<&6.73f8,Z72$#Z4&'!)(+&V²
b 1 d úû Q E Qc[ 1 * 1 (+3g=#35Q Œ ÷š
bjk‘d ¯ Q ¯ q E ¯húYû Q ¯ q p ¯ Q@J úYû Q ¯ q [ 1 * 1 (+3g=#3fQ ŒŽ‰ 
bj" d ¯hú€ü Q ¯ ´ ¯ Q ¯ [ 1 * 1 (+3g=#3fQ ŒŽ‰ b¾}&%! u $C&6/ " 1 8<3|8<&0"'$#\|[#.7&m. 1 Z72$ 1 .7= 1 =^…ídh
bj=‘dˆ‡gZ wîŒ ÷öP’bjQ@J w d“Øs÷š‹^&'!)(+3;!#"'&%8 w E úû Ql
bj& dýY3g=#3iQ ŒS‰ 8,&][`$#&'=#&|&'8,"'*,Z7k#Z4*mQ E Q n p Q q "%3!’Q n Œ ÷ìPQ q Øt÷šF} 8ä$#/ !`Z7" 1 . 1
=`&'8<"%3\|[o38,Z7"%Z¦3/ !^…
 È  µ &%=#Z 1 !a(<&&'.[#*,3_"%&'=#Z4\]Z4&'!)(+3 =`&žêm* 1 \fºÃ‡_"¥¬#\|Z7=`(Š¬ 1 .7.4&r$#! 1 k 1 8<&3*,(<323! 1 .;=#&%.)8<$#kl&'8,[ 1 "%Z73
÷ö2&%!#&'* 1 =#3|[o3;* .738 8,Z72$#Z4&'!)(+&%8 9&'"h(+3*,&'8 =#&@?ACþ ²
G äIÿIe'MWNYX G œÿJLœJœ'MWNYX G IRJLSJLUIíMONYX
Px.4$#&'2;3|" 1 .7"%$#.7& úYû_w [ 1 * 15w EHG S ˜œ˜ M N 
¦Ï` × 1 .7. 1 *$#! 1 k 1 8,&m3*,(<3!#3*,\ 1 .l=`&>É q "'3;q !xÎ &'.·[#*,3_=`$#"%(<3Z7!)(+&%*<!#35=`&%Ÿ‘!#Z4=#3&%!ˆ&%.Š&¡,&'*<"%Z7"%Z735Ï`‹
1 [ 1 *£(+Z4* =#&6. 1 k 1 8,&6" 1 ! 3;/ !#Z7" 1 Ì ;X Ê X Ê 
¦Ù`š¨ 1 .4"'$#.4&]. 1 [#*,3¦P&'"%"'Z 3/ !U3*,(<323! 1 . =#& DsE G   ˜ M 1 .~8<$#kl&'8,[ 1 "%Z732&%!#Î&'* 1 =#3[l3*0Q n5E
 J M N PQ  %M N 8<Z4!U3*,(<323! 1 .7Z4­ 1 *6&'.ž"'3!‚,$`!a(<3 Ì Q n X<Q q ÿb¾‡_$#2&%*<&'!`"'Z 1 ²
&%G \][#.4&'&|b„=^˜ d:=#&'.“&¡,qF&'*<E "%Z7"%G Z73x¦˜ Á`¹d
I ˜ š‡_& 1 ! ‰ $`!x &'8<[ 1 "'Z43-9&%"%(+3;*<Z 1 .o"%3!x[#*<3g=#$#"h(+3Z4!)(+&'*,!#3-Pf÷ E 2&%! Ì Q n X+Q q X » » »%X+Q‚ò Î ± *<3k 1 *
u $#& wŒ ÷ 8,Z·Px8‘3/ .43f8<Z~b w X+Q n d E b w X+Q q d E °¦°¦° E b w X<Q‚ò+d Ee˜ 
I_ × 1 .7. 1 *C÷  &%!i.438:8<Z42$#Z4&'!)(+&%8 " 1 8<3;8'²
b 1 dF÷ E 2;&'! Ì_G eJöR ˜ M N X G ™JL ˜e˜ M N Î P&%.€[`*<3g=#$#"%(<3cZ4!a(<&'*,!#3c&%8>&'." 1 ! 3/ !#Z4"'3]=#&
?A~þ 
bjk‘dF÷R"%3\]3f&%!i&%.“[#$#!)(+3 1 !)(+&%*<Z73;*'‹#[l&'*,3§b D X+Q_d E I Don Q n€päD^q Q q€p T D B Q B psD þ Q þ 
bj" dx÷ &'8m&'.€8<$#kl&'8,[ 1 "%Z73x=#&F• A#B 2&'!#&%* 1 =`3x[o3* G  ˜  p ¿ M N P G IH ¿ M N Pi&'.€[#*<3g=#$#"h(+3
Z4!)(+&'*,!#3|&'8 &'.“" 1 ! 3/ !#Z4"'3#
II_†b 1 dˆ‡g& 1 ! © Œ ?A  ª ” P÷ EÌ¦å Œ ?Až” ² © åEî˜ Î  t&%\$#&%8,(+*,& u $#&@÷  &%8,(‘1 / 2&%!#&'* 1 =#3
[l3*:. 1 8CŸ#. 1 8:=`&t. 1 \ 1 (<*<Z7­ © 8<Zo8<&m"%3!#8<Z4=#&'* 1 &'.o[#*< 3g=#$#"h(+35Z7!)(<&'*<!`35" 1 ! 3;/ !#Z7"%3#~}“t$ž& /
*,&'. 1 "'Z 3/ ![#$#&'=`&0&%8,( 1 k#.7&%"'&%*š&%!a(<*<&6=#Z4\§bç÷ d«‹‘=#Z7\i bç÷ dCPx&%.“* 1 !`23f=#&6. 1 \ 1 (+*,Z7­ © …
‡gZ“*<&'&%\][`. 1 ­ 1 ?A [l3*m• A } u $#Zh&'/ !#&%8š2;&'!#&%* 1 !÷ …

bjk‘d× 1 .7.4&tk 1 8,&'8 =#&6.738 "%3\|[#.7&%\]&%!a(<38 3*£(+323;! 1 .7&%8:=#&0.738ž8<Z72;$#Z7&%!a(<&'8 8<$`ko&%8<[ 1 "%Z738%²
Z¾:÷ EìÌ¦å Œ ?AžB ²gI åon€pUå^q J å B Ee˜ Î 
Z7Z¾ž÷ EìÌ¦å Œ ?AC” ² åonCEöå^q E » » » ELå ò Ee˜ Î 
Z4Z7Z¢~÷ EìÌ¦å Œ • A þ ² åon J ¿ å^qrp bà J ¿ d å B E™˜ b¾I p ¿ d å^qpUå þ E™˜ Î 
bj" dˆt&%8<"%*<Z7k 1 &%.“8<$#kl&'8,[ 1 "'Z43]÷ö=#&%.“&Á,&%*<"'Z4"'Z43ˆ¦®ib 1 dž[o3;* \]&%=#Z73f=#&6&'"%$ 1 "%Z73!#&%8š.4Z7!#& 1 .7&%8'
}6¨ž$ 1 / .>&'8c&'.>!>$`/ \]&%*<3\i¶/ !#Z4\]3Ž=#&&%"'$ 1 "'Z73;!#&'8 u $#&8,&!#&%"'&'8,Z4( 1 !^… ;$#8£(+Z4Ÿ u $`&8<$
*,&'8,[#$#&'8£( 1 
I;T`š‡_& 1 ÷H$`!U8<$#kl&'8,[ 1 "'Z43=#&F$`!&'8,[ 1 "'Z43§9&%"%(<3*<Z 1 . ‰ =#&F=#Z4\]&%!#8<Z 3/ !{Ÿ‘!`Z4( 1 =#3;( 1 =#3=#&]$#!
[#*,3_=`$#"%(<3|Z4!)(+&'*,!#3#~t&'\|38£(+* 1 * u $#& ú û Q E Q-J úû Q][ 1 * 1 (<3_=#3fQ ŒŽ‰ 
}€¨x3/ \]3|&'\|[#.4& 1 *'¶/ 1 . 1 *,&'. 1 "'Z¦3;/ !i&%!a(<*<& úYû P ú û  [ 1 * 1 " 1 .7"%$#. 1 * . 1 [#*<3¦P;&'"'"%Z¦3/ ! 1 ÷š‹o&%!§&%.
" 1 8<3f&'! u $#&m÷L&%8 $#!x8<$`ko&%8<[ 1 "%Z73f=#&0?A ”bV35/ =#&• A ”dC=#&%8<"'*,Z7[`(<35[o3;*:&'"%$ 1 "'Z43!#&%8 .7Z7!`& 1 .4&'8CP
&%.“[#*<3g=#$#"h(+3fZ7!)(+&%*<!#3|&'8 &%.“&'8,(‘1 / !#= 1 *¥…
I  š‡_& 1‰ $`!&'Î8,[ 1 "'Z43i9;&'"%(<3*<Z 1 .~"'3![`*<3g=#$#"%(<3§Z4!a(<&'*,!#3§P’8<& 1  !  E Ì Q n X » » »%X+
Q Î P Â E
Ì[lw 3* n u X'$C» &» / »'X Qw X » »'k »'1 X+8<
Q &% )8X 3*,(<X 3»'2» 3»%! X 1 .7&%Î8€=#&'&C8 .7$#3! 8Y8<$`k ko&%8,8<&@[ 31 *£"%(+Z7332;8Y3÷{! P6.o÷ =#& *<&%8< [l&'"%(<Z49 1 \|&'!)(+&Vr¤_[#.7Z u $#&
Ì n w n w 1 1 1 ‰
I®_š¨ž3!`8<Z7=`&'*<& ‰ E É ” "'3!i› en I5P&%.[#*<3g=#$#"h(+3cZ4!)(+&'*,!#3§n b¢ÐlX,Ñ#d E  Ç n n Ðb Ê dÃъΠb Ê dÃÓ Ê ž 1 =`3]&%.
8,$#ko&%8<[ 1 "'Z73f÷ EìÌ õ Œ’‰ ²   õYb Ê d'b£ pUÊ d£Ó Ê Ee˜  õb Ê d'bà J Ê d£Ó Ê Ee˜ 8,&6[#Z7=#&V²
b 1 d× 1 .7. 1 *:$#! 1 k 1 8<&03*,(<323! 1 .Š=#&m÷  
bjk‘d× 1 .7. 1 *:. 1 [#*<3¦P&%"'"%Z¦3/ !i3*,(<323! 1 .Š=#&mъb Ê d ERÊ q 8<3;k#*<&m÷  
bj" dˆ× 1 .7. 1 *:&%.“&'.7&%\]&%!)(+3f=#&m÷ u $#&@\|&Á,3;* 1 [#*<3 ¤_Z7\ 151 ÑcPx" 1 .7"%$#. 1 *:Óob„ÑlX<÷:dh
I;Á`š¨ž3!`Î 8<Z7=`&'*<& &%!?A B &'.[`*<3g=#$#"%(<3 Z7!)(+&%*<!#3 " 1 ! 3/ !#Z4"'3:P>&'.)8<$`ko&%8<[ 1 "%Z73 ÷ E 2&%! Ì_G @ ˜ M N X G 00J
'M N  !#"'$`&'!)(+*,&@(+3g=#38 .438 Q]( 1 .7&%8 u $#& úû b„Q_d EG I@IeJS'M N P ¯ Q ¯ E ®_
 !ˆ.738 8,Z72$#Z4&'!)(+&%8 &Á,&'*,"'Z4"'Z73;8 "'3!#8,Z7=#&%*<&6&'.“[`*<3g=#$#"%(<3]Z4!)(+&'*,!#3f" 1 ! 3/ !#Z7"%3f=#&@?A ”_
I È †b 1 dˆ‡g& 1 ! w Œ ?A ”`‹ wyE– ˜ ‹ŠPi÷ E—Ì¦å Œ ?A ”ˆ² w N å’E˜ Î 0t&%\$#&'8£(+*,& u $#&5ÓobjQ^X<÷:d E
³ w N Q“³  ¯«w]¯ P u $#& úYû Q E Q- J     w 
bjk‘dˆ‡g& 1  E Ì¦å Œ ?A ”i² w N å{E Ë Î "%3! w "'3\|3x&'!Ž&'. [`$#!)(+3 1 !)(+&%*<Z73;*>PË Œ ?A !#3
!`&'"'&%8 1 *,Z 1 \|&'!)(<&@!a$#.43#
Z¾ t&'\$`&'8,(<*<& u $#&f8<Z w Î -Œ  ‹“&'!)(<3!#"'&%8@Óob„QlX  d E ӊb„Q5J w  X,÷ d«‹Y8,Z7&%!#=#3x÷ E
Ì¦å Œ ?A ”5² w N åEe˜ ! ¡!)(+&'*,[#*<&h(+&0. 1 Z72;$ 1 .7= 1 =F2&%3\ä&h/ (+*,Z7" 1 \]&%!a(<&;
Z7Z¾ t&'\|38,(<* 1 * u $`&@ÓobjQ^X  d E ³ w N Q@JË)³  ¯«w|¯ 
I;Ï`š‡_& 1 ! w Pm÷{"'3\|3 &'!b 1 d·=#&'.a&¡,&'*<"%Z7"%Z73 1 !)(+&'*,Z73*%r± 1 * 1 " 1 = 1 Q Œ ?AŠ” 8<& #1 " bjQ_d E Q“J@$I       
b 1 dˆ¨ž3;!#8<Z4=#&'*,& w EñG œJL%M N PxQ EñG äIíM N ~êm* 1 Ÿ u $`&:,$`!a(<38 ÷>‹^Q|P " bjQ_dh
bjk‘dˆ¨ž3;\][#*,$#&'kl& u $#& " b„Q`d E úYû Q-J ú û  Q^
®
jb " dˆt&%8<"%*<Z7k 1 2&%3\ä&%/ (+*,Z7" 1 \]&%!)(+&t. 1 (<* 1 !`8,ƒ„3*,\ 1 "%Z¦3/ ! " 
bj=‘dt&%\$#&%8,(+*,& q u $#&x[ 1 * 1 (+3g=#3Q Œ ?A ” ‹ " bjQ_d E&% Ql‹ 8,Z7&%!#=#3 % Œ ?A ” ª ” . 1 \ 1 (<*<Z7­
% ( E ' J    wšw N šb % &%8 . 1 \ 1 (+*<Z4­t=#&0ך3;$#8<&%¬#3.7=`&'* 1 8<3g"'Z 1 = 1f1 .Š9;&'"h(+3* w d«
bj& dˆt&%\]3;8,(+* 1 * u $#& % &%8 8<Z7\U&%/ (<*<Z4" 1 ‹_!#3f8<Z7!`2$#. 1 *žP u $#& % Ç n E)%yE)% N 
b„ƒ+dt&%\]3;8,(+* 1 * u $#&c. 1 8m"'3.4$#\|! 1 86=#&|. 1 \ 1 (<*<Z4­ % ƒ„3*,\ 1 !’$`! 1 k 1 8<&|3*,(<3!#3*,\ 1 .r=#&
?A ”`
;I Ù` × 1 .7. 1 *&'. 8,Z7\ä&%/ (<*<Z7"%3=#&ˆQ*<&%8<[l&'"%(<3v=#&%. ¬#Z7[l&'*,[#. 1 !#3÷À&%!ÿ" 1 = 1 $#!#3’=#&ˆ.73858,Z72$#Z4&'!)(+&%8
" 1 8<38%²
b 1 dF÷ EìÌ¦å Œ ?A B>²-I åon€pUå^q J å B Ee˜ Î PxQ EHG  -Ë Ä M N 
bjk‘dF÷ E 2;&'! Ì_G JtIR ˜S˜ M N X G J0 ˜  ˜ M N X G՘ÿ˜œ˜ JL'M N Î PxQ EG œ ˜ 'M N 
T ˜   !#"%$#&'!)(+*,&-$#! 1 \ 1 (<*<Z7­>=#&0ך3$#8,&'¬#3;.7=#&%* % ( 1 . u $#& %UG ÿœJL'M N EG+* T ˜œ˜ M N 

Á
  
     

  !#"$&%  &%$' (  )
* +

,.-0/2143657-98;:=<?>;@*AB@C@*8D/E-F>;-9: GH-9:I@KJB@*AB5*<H57<H-9:I@7GML;A=-F>;N;5O/E-P<H8Q/=@*A=84-+5*1R8 -9S 8;<?5*-;T


U T.VWN4LX-984YQ1+Z[N;@\G]1R:5*-9G?N;^8_1R:>;@&`bacdfeQg0hjilk d e[g0hfm :B-98$-9An/E-9YR-98_1RG?@*: >;-9:o1+>;-9:7T.p qrNs@ S
L;N4@*>;@t>;@*57<HA 1R57@*AB5 1P>;@t`uv`wix`\yz`mB{
| T.VW@P>;<?5*@PZ[N;@}~acd e[g0e€ilk d eQg0eQmr@*:CN;8;1^+10/EAB<H‚&>4@L4A=-„ƒ9@75*5*< -9S 8:B<…}u†}‡iˆ}ty‰†}\m
ƒŠ}C‹&†Œ}PTDr@*^&N4@*:B/=A=@+ZFN4@Ž:=<z}w@*:\N;8_1^+10/=A=<H‚P>;@L;AB-ƒR@*575*<„-RS 8’‘ @*8Q/=-98;5*@7:“L_1RA=15*1R>_1
”57-9a€G˜ix}cd mse @7iKGMk d :=eN;m•–—‘_@*} :BL_” 1R57@*<H: -GH1&Y9@*L;8;A=@7-„AEƒ91R@7>45*-+57<„L—-9S 8-9AI-9G]AB10/=:I-9Y95*-9-9G?8_N;10^+Gv>;8;@1R:I” >;:B@t-9–;}ABT@t@7GM:=N;–—@*:BL_1R57<H-5*-9G˜iˆ}\mO‘4:B<H@78;>;-
™ T.VW@*1.š›a€cdve/210GFZ[N;@tœ•š+œf† U T s-9^L;A=N;@7–X@žZ[N;@I}Ÿ† šoš u“@*:¡N;8_1o^+10/=A=<H‚…>;@žL;AB-ƒR@*575*<„-RS 8’T
p¢sN 1RS GM@7:@7GMAE1R84Y9-P>;@t}{$pM:=-R–;A=@CZ[Nz@tS :=N;–—@*:BL_1R5*<?-+L4A=-„ƒ9@757/219{
£ Tr@7^&N;@*:n/EAB@Z[N;@&}¤@7:oN;8_1+^Ž10/=A=<?‚>;@L;AB-„ƒ9@*575*< -9S 8):=<!ƒD:_-9S G?-Ž:B<¦¥ eC§ }¤@7:oN;8_1^Ž1¨/EA=<?‚C>;@
L;AB-„ƒ9@*575*< -9S 8’Tzp¡VF-9–;A=@CZ[Nz@tS :=N;–—@*:BL_1R57<H-L;A=-„ƒ9@757/E1Ž¥ er§ }{
© T.p’ª…@*AB>_1R>;@7A=--P«x10GH:=-Q{
iˆ19mVF< G]1R:o5*-9G?N;^+8;1R:r>;@“GH1¬^Ž10/=A=<?‚`­YR@*8;@7AE1R8€@*G¦:=N;–—@*:BL_1R5*<?-®j¯cdeŽ@*8Q/E-9845*@*:\`t`u
@7:.GH1&^Ž1¨/EA=<?‚r>;@CL;AB-ƒR@*575*<„-RS 8°:=-R–;A=@®.T
ix–_mVF<4G]10:¦5*-RGHN;^8_1R:¡>;@fG]1^Ž1¨/EA=<?‚s`±Y9@*84@*AE108“@*G4:BN;–—@*:=L;1R5*<?-r® ¯±cd!e.ƒ:B-98“-9An/E-9Y9-R8_1RGH@7:*‘
@78Q/E-98;57@*:o`\`ru)@7:GH1“^Ž10/=A=<?‚r>;@CL;A=-„ƒR@*5*57<„-9S 8:=-R–;A=@C®oT
ix5 mVF<_G]1R:657-9GHN;^8_1R:¦>;@ G]1r^+10/EAB<H‚f`‰:B-98N;8_1–_1R:B@-RAB/E-R8;-9A=^+1RGW>;@*G;:BN;–—@*:=L;1R5*<?-t®±¯±cd¡e
@78Q/E-98;57@*:o`\` u @7:GH1“^Ž10/=A=<?‚r>;@CL;A=-„ƒR@*5*57<„-9S 8:=-R–;A=@C®oT
ix>_mVF<MG]1R:I^+10/EAB<H57@*: }jƒD}²³L;AB-ƒR@*5O/21R8:=-9–;AB@C@*GM^<H:=^-“:=N4–X@7:=L_1R57<H-@*8Q/E-R8;5*@7:o}›†±}C²HT
ix@ mVF<¡}ja´cd e[g0e @7: N;8_1P^Ž10/=A=<?‚r>;@tL;A=-„ƒ9@75*57<„-9S 8ƒD}w†¶µ ¥ e ‘_@*8Q/=-98;5*@7:o@7GMAE1R8;YR->4@\}·@7:
1PGH-“:BN;^-¸ § U T
i¹«EmVF<!}@*:N;8_1“^+10/EAB<H‚r>;@CL;AB-„ƒ9@*575*< -9S 8ƒº@•»W<H:B/=@t}“¼4½7‘4@*8Q/E-9845*@*:o}Ÿ†¶¥ e T
¾ T¿68;57N;@*8Q/EAB@ŠGH1^+10/EAB<H‚$}À>;@)L;A=-„ƒR@*5*57<„-9S 8±:=-R–;A=@)@7Gr:=N;–—@*:BL_1R57<H-Á®w@*8 5 10>_1ÂN;8;-Á>;@)GH-R:
:B<HY9N;<?@*8Q/E@7: 5 1R:B-9:*3
iˆ19m¬®Ã†ÄYR@*8vÅWÆ UÂ||Ç u¢ÈzÆ § || § U*Ç u!É[T
ix–_m¬®sÊ$†±Y9@*8vÅWÆ UÃU™0Ç u ÈzÆ § UÃU § U*Ç u É[T
ix5 mº®Ã†Ëń̏a´cd…Ír3IÌ ½¡Î | Ì ‹ § Ì³Ï Î Ì Í †¶ÐFÉ[T
Ñ T.VW@*1tҟa)cd Ï g Ï T ¿68;57N;@*8Q/=A=@oGH1r^Ž1¨/EA=<?Ï ‚f>;@ L;AB-ƒR@*575*<„-RS 8Ž15*-RG˜i¹Òm :E1R–4<H@*84>;-CZFN4@.A=1R8;Y9-;ixÒm…†
| ƒºZ[N;@tL_1RA=1P57<H@7AB/21“^+10/EAB<H‚Óa€cd g ‹
U U ×ÙØ
ÒrÓj† ÔÕÖ § U | ړÛ
Ð ™
U
Ü T.VW@*1tҟa)cd Í g Í T ¿68;57N;@*8Q/=A=@oGH1r^Ž1¨/EA=<?‚f>;@ L;AB-ƒR@*575*<„-RS 8Ž15*-RG˜i¹Òm :E1R–4<H@*84>;-CZFN4@.A=1R8;Y9-;ixÒm…†
™ ƒºZ[N;@CÌ;u—ÒĆ¶Ð“:=<’Ì;u†Æ UÝUUÃU*Ç T
Þ Tr@7^&N;@*:n/EAB@Z[N;@:B<¢}ba´ß e[g0e ixßà†Ëcdá-S k d mI@*:oN48_1+^+10/EAB<H‚>;@tL;AB-ƒR@*575*<„-RS 8)>;@\A=1R8;Y9-Žâ
@78[/=-98;57@*:.¥ e§ }j/=<H@78;@AE1R84Y9-P¸ § âXT
U Ð4T.s-984:=<H>4@*A=@CGH1P^+10/=A=<H‚ Uã0™ UãR™ UãR™ × Ø
}Ÿ† ÔÕÖ Uã0™ ©9ãR¾ § UãR¾ Ú Û
Uã0™ § UãR¾ ©9ãR¾
<]mfs-9^L;A=N;@7–X@CZ[N;@t@*: N;8_1P^+10/EAB<H‚r>;@CL;AB-„ƒ9@*575*< -9S 8’T
<?<ämo¿68;57N;@*8Q/EAB@+N48_1^+10/=A=<H‚>;@P5*-9G?N;^+8;1R:r-9An/E-98;-RA=^+1RGH@7:`aÃcd Ï gRå 5*-98Šâ€†¤AE1R8;YR-_iˆ}\mO‘
/E1RG’Z[N;@\}Ÿ†Ä`t`u¡T
U9U T¿68;57-98Q/EAE10AG]1>;@*:B5*-9^L—-9:=<?5*<„-RS 8`tæ&‘¦/2108[/=-°8;-°8;-9A=^+1RG?<H‚ 10>_1º5*-9^-84-9A=^+1RGH<?‚ 1R>;1D>4@ŽG]10:
:B<HY9N;<?@*8Q/E@7: ×Ù^ŽØ 10/=A=<?5*@*:73
U | U | £ ×ÙØ U | ™ ×ÙØ
1Qm´ÔÕ U Ú –³mŠÔÕ U U Ú 5 mŠÔÕ U U Ú >_m$ç UU UU UUŽè T
Ö Ð Ö Ð Ö Ð
U £ U £ ¾ U £ ¾
U| T.VWN4LX-984YQ1Z[N;@+Ò#@7:\N;8;1^+10/EAB<H‚PA=@*1RG…¸Âéêë/210G6ZFN4@ŽÒ­†­`tì•æ.섑¡57-98`Cì¬a±cd eQg0h >;@
57-9GHN;^8_1R:-9An/E-9Y9-R8_1RGH@7:*‘_ƒ°æoìCaÂcdfhfg0hÄ/=A=<]108;Y9N;GH1RAI:=N;L—@*AB<H-9Aƒ5*-98$N48;-9:r@78$G]1>;<]1RYR-98_1RG
L;AB<H8;57<HL_10GíT.r@*^&N;@7:B/=A=@“Z[N;@&GH1R:o57-9GHN;^8_1R:.>;@“`Cìr«¹-9A=^+1R8$N48)5*-98¨JBN48[/=-ºY9@*84@*AE10>;-9Ar-9An/E-0î
Y9-R8_1RG¢>;@“5*-9GíixÒrmO‘vƒZ[N;@“G]10:oL;AB<H^@*AE10:oïf57-9GHN;^8_1R:.>;@P`CìY9@*8;@7AE1R8€@7G¦@*:BL_1R57<H-ºY9@78;@*A=1R>;-
L—-9AIG]10:IL;A=<?^+@7AE1R:fï…57-9GHN;^8_1R:I>4@tÒ\T
,-R/2143¢A=@7:=N;@7G?ð01r@*G4@lJB@*AB5*<?5*<H-o:=<H8“:BN;L—-98;@*A6Z[N;@ `Cì6ƒ&æ.ìž:B-98“G]10: Z[N;@I:=@I-9–W/E<H@78;@*8P10L;GH<?5 1R8;>4-
@7GM^Á@OS /E-F>;-“>;@C-9AB/=-9Y9-98;1RGH<?‚ 1R57<„-9S 8¬>4@ñAE10^PîlVW52ò;^<H>4/f1PGH1R:I5*-RGHN;^8_1R:I>;@CÒ\T
U„™ T.VW@*1Òwacdoe[g0h‰>;@PAE1R8;YR-$â$/21RG¦ZFN4@+Òb†b`\æb5*-98`­ƒ€æb/E1RGH@7:tZ[N;@+`acd.e[gRå“/=<H@*84@
57-9GHN;^8_1R:¦-9AB/=-98;-9AB^Ž1RG?@*:7‘Rƒ“æËa€cd åg0h @*:6/EAB<]1R84Y9N;G]10A¦:=N4LX@7A=<H-RA¦ƒ“>;@ AE108;Y9-tâXT6r@7^&N;@7:B/EAB@
Z[N;@&GH1R:5*-9G?N;^+8;1R:.>4@&`¤«¹-9A=^+1R8N;8_1Ž–_10:=@&-9An/E-984-9A=^+1RGM>;@\5*-9G˜ixÒm•‘—ƒZ[N;@“}†Ë`t`ru@7:
GH1&^Ž10/=A=<?‚r>;@CL;A=-„ƒR@*5*57<„-9S 8:=-9–4A=@C5*-9GíixÒrmOT
VWN4Y9@*AB@*8;57<]143 >;@7^&N;@*:n/EAB@&L;A=<?^+@7A=-+ZFN4@&:=<¡Òjƒ°Ó­:=-98°^Ž10/=A=<?5*@*:/21RG?@*:Z[N;@“ÒˆjÓ&óL_1RA=1
57<H@*An/21t^Ž10/=A=<?‚.óP‘F@*8Q/E-9845*@*:I5*1R>_157-9GHN4^+8_1C>;@.Ò¶@*:s5*-9^&–4<H8_1R57<„-9S 8GH<?8;@ 10G_>;@oGH1R:z57-9GHN4^+8_10:
>;@tӎ‘_@78-0/EAE10:IL_1RG]10–;AE1R:7‘;5*-RG˜i¹Òmf¯ô5*-9GíiˆÓ&mOT
,-R/2143¦A=@*:BN;@*Gõð01P@7G’@lJB@7A=57<H5*<?-“:=<?8º:=N;L—-98;@7AIZFN4@\`›ƒºæ›:=-98DGH1R:fZ[N;@:=@-R–4/E<?@*8;@7810L;GH<?5 1R8;>4-
@7GM^Á@OS /E-F>;-“>;@C-9AB/=-9Y9-98;1RGH<?‚ 1R57<„-9S 8¬>4@ñAE10^PîlVW52ò;^<H>4/f1PGH1R:I5*-RGHN;^8_1R:I>;@CÒ\T
U £ T¿¦»WL;GH<?Z[N;@CLX-9AIZ[Nz@ S
U | ™ ×ÙØ
ÒĆëÔÕÖ Ð Ð £ Ú †¶`tæ&È
Ð Ð ©
57-98°`›†Ä¥7Ï ƒæ‰†Äҟ8;-@*:N;8_1“>;@7:=57-9^+L—-9:B<H5*< -9S 8`\æË8;-9AB^Ž1RG?<H‚*1R>_1>;@tÒTzp!¿6:.L—-9:=<?–;GH@
-9–W/E@*84@*A N;8_1P>;@*:B5*-9^L—-9:=<?5*<„-RS 8D`\æË8;-9A=^+1RG?<H‚ 10>_1>;@CÒË1“L_1RAB/=<HAf>4@\GH1R:f5*-RGHN;^8_1R:I>;@t`\{
|
U© T¿68;57-98Q/EAE10AN48_1>4@*:=57-9^LX-9:B<H57<„-9S 8D`tæj8;-RA=^+1RGH<?‚ 1R>_1\>;@CG]1“^Ž1¨/EA=<?‚oқa€cd e[g0e
U ö ½ ÷„÷„÷ ö e ¼4½ ×ÙØØØ
ÒĆ ÔÕÕÕÕÖ U TT ö TT ½ ÷„÷„÷ ö e TT ¼4½ Ø Û
U ö ½ ÷„÷„÷„÷„÷÷ ö e ¼4½ Ú
U„¾ T¿68;57-98Q/EAE10A*‘³A=@7:=-9GõðW<?@*8;>4-+GH1R: @*5*N;1R5*<?-98;@*:8;-9AB^Ž10GH@*:7‘4/E-F>_1R: GH1R: :=-9G?N;5*<?-98;@*: L—-9A 5*N_10>;AE1R>4-9:
^øS 8;<?^+-R:s>;@CG]1&@75*N_105*<„-RS 8ÒÌD†±ù3
iˆ19m
U | ×ÙØ £ ×ÙØ
ÒĆúÔÕÖ § | § ™ Ú È ù † ÔÕÖ U Ú Û
§U ™ |
ix–_m
U U Ð ×ÙØØØ U ×ÙØØØ
U U Ð
Ò¶† ÔÕÕÕÖ U Ð U È ù † ÔÕÕÕÖ Ü Û ™
U Ð UÚ | Ú
U Ñ T.VW@*1R8
™ £ ×Ø U9U × Ø
Ò¶†ëÔÕÖ § | U Ú È ù †úÔÕÖ § Þ Ú È ûD†àç ©U è ƒ ” †ç ©| è Û
™ £ © § §
f1RG?5*N;G?@ºÒoûüƒÂÒ ” ‘zƒ5*-9^L 1RS AB@*GH-R:&5*-98üùTp ý¡-F>;A*øS 1):=@7AûÝG]1):=-RGHN;57<„-9S 8ÁL—-9A“5*N_10>;AE1R>4-9:
^øS 8;<?^+-R:s>;@CÒ.̺†±ù7{Âiís-R8[/=@*:n/E@:B<H8D5 10GH5*N4G]1RAfGH1R:I:=-RGHN;57<H-98;@7:ILX-RAI5*N_1R>4AE1R>;-R:^øS 84<H^-9:*Tþm
U„Ü T¿68€5 1R>;15*1R:=-;‘MAB@*:BN;@*GõðR1ºL—-9A5*N;1R>;AE10>;-9:C^øS 8;<H^-9:G]1¬@75*N_1R57<„-9S 8€Ò̆·ù&NW/E<HG?<H‚*1R8;>;-+N;8_1
>;@7:=57-9^+L—-9:B<H5*< -9S 8`\æ›8;-9AB^Ž10GH<H‚*1R>_1>4@tGH1“^Ž10/=A=<?‚rÒ3
iˆ19m
ÒĆç |U |U è ù †àç ܏| è Û
ix–_m
U | ×Ø | ×Ø
Ò±† ÔÕÖ | U Ú ù† ÔÕÖ ™ Ú Û
U §U Ð
™
U„Þ T.f1RG?<?ÿ_Z[N;@5*1R>_110ÿ_AB^Ž1R57<„-9S 8º57-9^-&ð9@*AB>_1R>;@7AE1-P«x10GH:E1WTRN;:B/=<?ÿ_Z[N;@C5 10>_1AB@*:BL;N;@*:n/214T
iˆ19mD¿6G…L;AB-9–;GH@7^Ž1¬YR@*8;@7AE1RG6>;@P57N_1R>;A=1R>;-9:t^øS 8;<H^-9:.5*-98;:B<H:n/E@P@*8´@*8;57-98Q/EA=1RA\G?-9:rÌÝacd e
Z[N;@tò_1R57@*8^øS 8;<H^+1GH1&>;<H:n/21R8;57<]1&@78[/=A=@tÒ.̏ƒºùT
ix–_m 8_1:B-9GH N;57<„-9S 8PL—-9Až5*N_1R>4AE1R>;-R:s^øS 8;<?^+-9:¦>;@.Ò́†±ù @7:sN;8ð9@*5O/E-9A ̬ /210G—Z[N;@.Ò Ìº † ù ‘
:B<H@78;>;- ùrG]1&L;AB-„ƒ9@*575*< -9S 8°-9An/E-9YR-98_1RG’>4@tùo:=-9–;AB@C@*GM@*:BL_1R57<H-P5*-9G?N;^8_1“>;@CÒT
ix5 m 8_1:B-9GHN45*<„-RS 8´L—-9AC5*N;1R>;AE10>;-9:C^øS 8;<H^-9:r>4@+Ò̆#ù@*:tN;8ŠðR@*57/=-9A Ì /21RG6Z[N;@L_1RA=1
/=-W>4-PÌ°a´cd e ‘ œ•ù § ÒÌ6
œ ·œ•ù § Ò Ì… œT
ix>_m !1P@*57N_1R5*< -9S 8DÒuXÒ.̺†±Ò u—ùr:=<?@*^L;A=@/E<?@*8;@:B-9GHN45*<„-RS 8’T
ix@ m !1P@*57N_1R5*< -9S 8DÒ u Ò.̺†±Ò u ùr:=<?@*^L;A=@/E<?@*8;@:B-9GHN45*<„-RS 8ŸN4S 8;<H5*14T
i¹«EmVF<MG]1R:f57-9GHN4^+8_10:I>;@қ:=-98DGH<?8;@ 10GH^@*8Q/E@.<H8;>;@7LX@78;>;<?@*8Q/E@7:*‘³@*8Q/E-R8;5*@7:.GH1&:=-9G?N;5*< -9S 8ºLX-RA
57N_1R>;A=1R>;-9:^øS 8;<?^+-9:ž>;@CÒ.ÌD†±ùr@7:€N;S 84<H5 1WT
ixYQmVF<¢ùoLX@7AB/E@78;@*57@&1P5*-9GíixÒrmO‘³@*8Q/=-98;5*@7:./=-W>_1“:B-9GHN;57<„-9S 8D>;@tÒ.ÌD†Äù@*:N;8_1P:=-9G?N;5*< -9S 8DLX-RA
57N_1R>;A=1R>;-9:^øS 8;<?^+-9:7T
| Ð4T.VWN4LX-984YQ1&Z[N;@rÌD†jÆ UŠ| § U*Ç u @7:IN;8_1&:=-RGHN;57<„-9S 8¬L—-9Az5*N_1R>4AE1R>;-R:I^°øS 8;<H^-9:…>;@G]1t@*57N_1R5*< -9S 8
ÒÌD†Äùr57-98 U U | ×Ø
ÒĆ ÔÕÖ § U | U Ú Û
§U U Ð
<]ms¿68;5*N;@78Q/EA=@t/E-F>_1R: G]10:f:=-9G?N;5*<?-98;@7:L—-9AI57N_1R>;A=1R>;-9: ^øS 8;<?^+-9:z>;@/21RGv@*5*N;1R5*< -9S 8’T
<?<ämfV410–;<H@78;>;-PZ[N;@ºœ•ù[œI† ™ ‘;ò;1RGHG?@rGH-9:IL—-9:B<H–;G?@*:fð01RGH-RA=@*:I>4@tùT
|WU T.VW@*1қa€cd e[g0h
iˆ19mDr@7^&N;@7:B/EAB@€ZFN4@Š,N;GKi¹Òm)† ,.N;GíixÒ u—ÒmOTwiíVWN;YR@*A=@78;5*<H143Â>;@7^&N;@*:n/EAB@´L4A=<H^@*AB-Z[N;@
Ì;uXÒ u—Ò.ÌD†Ä Ð Ò.̺†ÄÐ4Tþm
ix–_mDr@7^&N;@7:B/EAB@C1L_1RAn/E<?Az>;@7GvL;N48[/=-P1R8Q/E@7A=<H-RAsZ[N;@rA=1R8;Y9-;ixÒm…†ÄA=1R8;Y9-;ixÒ u—ÒmOTiˆVWN;Y9@7A=@78Wî
57<]143¡@*^L;GH@7@.GH1tAB@*GH1R5*< -9S 8+Z[N;@.@O»W<H:n/E@.@*8Q/EAB@o@*G³A=1R8;Y9-\>;@.N48_1^+10/EAB<H‚ ӎ‘FG]1C>;<H^@*84:=<„-RS 8
>4@,N;G˜ixӓmzƒº@7GM8rN;S ^+@7A=-P>;@C5*-RGHN;^8_1R:f>4@ӎTþm
ix5 mr@7^&N;@7:B/EAB@\Z[N;@tÒ u қ@*:<H8QðR@*A=:B<H–;G?@C:=<Mƒº:_-RS GH-:=<MAE108;Y9-³i¹Òm…†¶ê)‘³@*:>;@*57<HA*‘_:B<Mƒº:_-9S G?-
:B<!G]1R:f57-9GHN4^+8_10:I>;@CÒË:B-98GH<H84@ 1RG?^+@78[/=@o<?8;>;@*L—@*84>;<H@78[/=@*:7T
ix>_mDr@7^&N;@7:B/EAB@oZ[N;@r@7GXL4A=-9–;G?@*^+1>;@o57N_1R>;A=1R>;-9:z^°øS 8;<H^-9:6Ò.̺†Äùf/E<?@*8;@.:=-9G?N;5*< -9S 8±N4S 8;<H5*1
:B<IƒÂ:_-9S G?-€:=<IG]10:“5*-9G?N;^+8;1R:“>;@DÒ:=-98üGH<?8;@ 1RG?^+@78Q/E@Ž<?8;>;@7LX@78;>;<H@78Q/E@*:7‘sƒÂZ[N;@@*8Á/21RG
5*1R:=-G]1ÃN;S 84<H5 1&:B-9GHN45*<„-RS 8º@*: ÌD †·i¹Òu—Òrm ¼4½ Ò uvù&iˆ1“G]1&^+10/=A=<H‚o
Ò M†·i¹Òu—Òrm ¼4½ Ò u):=@CGH1
>4@*8;-9^<H8;1P^+10/=A=<H‚oL;:=@7N;>;-9<?8[ðR@*AB:E1+>4@tÒrmOTzs-9^L;A=N4@*–—@\10>;@*^Ã1RS :IZ[N;@t
Ò ¹Ò±†¶¥ e T
|9| T.f1RG?5*N;GH1RA*‘W:=<M@•»4<?:B/=@R‘³Ò  L;1RAE1“5 10>_1^+10/=A=<H‚oҟ>;@*GM@KJB@*A=57<H57<H- U„¾ T
|R™ T.VW@*1ºÓŒ†bŐû ½ È Û Û Û È=û h ɬ¯cdIeŽN;8_1¬–_1R:B@“>;@*G¡:=N;–—@*:BL_1R57<H-º®¶ƒ°:=@ 1¬ÒbaücdIe[g0hÄG]1Ž^+10/=A=<H‚
Z[N;@/E<?@*8;@CL—-9AIïxîR@*S :B<H^+1&5*-9G?N;^+8;1“1Pû íT…r@*^&N;@7:B/=A=@CZ[N;@}‰†±Ò“i¹Òu—Òrm ¼4½ ÒuD†±Ò.
Ò ¦@7:.GH1
^+10/EAB<H‚r>4@tL4A=-„ƒ9@75*5*< -9S 8:=-9–;AB@t®.T
£
6¿ ^L;GH@*1R8;>;-º@7:B/E@“AB@*:BN;G?/E1R>;-;‘Mò_10GHGH@\G]1Ž^+10/=A=<H‚\>;@“L;AB-ƒR@*575*<„-RS 8Š:B-9–;A=@“@7G :=N4–X@7:=L_1R57<H-D®Ÿ>;@7G
@KJB@*A=57<H57<H- ¾ iˆ1Qm•T
| £ T.1RGHGH1RAsGH1PAB@*5O/21“Z[N;@1„JBN;:B/E1^@lJB-9A 1“G?-9:I:=<?Y9N;<H@78Q/E@*: L;N48[/=-9:IƒºY9A=10ÿ_5 10AI@*85 1R>_1P5 10:=-;3
iˆ19m´ixÐ4È U m•‘!i U È U mO‘Mi | È | mzƒ´i ™ È | mOT
ix–_m´i U È U m•‘!i U È | mzƒ´i U È § U mOT
|9© Tr@7^+-9:n/EA=1RAzZ[N;@o>_1R>;-9:zGH-9:žL;N;8Q/=-9:Ci¹Ì ½ È  ½ m•È Û Û Û Èi¹Ì e È  e m•‘WGH1\A=@757/E1Z[N;@r^@lJB-9Až:=@1„JBN;:B/E1&1
@7GHGH-R:f@*:€N;S 8;<?5 1“:=<’ƒº:;-9S GH-“:B<!LX-9AIG?-“^+@78;-9: >;-9: >;@CG?-9:IL;N;8Q/E-R: /=<H@78;@*81R–;:B5*<H:=1R: >;<?«¹@7A=@78[/=@*:7T
|R¾ T !1“:=<?Y9N;<H@78Q/E@/21R–4G]1&^&N;@7:B/=AE14‘_L_10AE11RGHY9N48;-9:I<H84:B/2108[/=@*: •‘_G]1“L—-9:=<?5*< -9S 8º^+@7>;<H>;1 !i Bmf>;@CN;8
^Ã-RS ðF<HGXZFN4@\:B@t>4@*:=L4G]1R‚*1@78G?<H8;@*1&A=@*5O/214T
/\ix:B@*Y;Tþm U | ™ £ ©
Li¹^Tþm © TþÐ Ñ U Ð4T £ ™ U„© T Þ £ |WU T ¾9™ | Ñ T £ Þ
iˆ19mVFN;LX-R8;<H@78;>;-rZ[N;@fGH1 ðR@*G?-W57<H>_1R>\>;@*GW^-0S ðW<?GQ@*: 57-98;:n/21R8Q/E@0‘9ò_1RG?GH@žG]1I«¹N;8;57<„-9S 8&>4@sL—-9:=<?5*<„-RS 8
!i BmfZ[N;@1„JBN;:B/E1^@lJB-9Af1PG?-9:I>_10/=-9:Iƒº@*:n/E<?^+@GH1ð9@*G?-W57<H>_10>’T
ix–_mVFN;LX-R8;<H@78;>;-¬10ò;-9AE1“Z[N;@\G]1P1R5*@7GH@7AE1R57<„-9S 8D>;@7G¢^Ã-RS ðF<HGv@*:5*-98;:n/21R8Q/=@R‘—@*8;57N;@*8Q/EAB@&G]1&«¹N;8Fî
57<„-9S 8 ¢i BmfZFN4@1JBN4:B/21“^@lJB-9A 1“G?-9:I>_10/=-9:Iƒº@*:n/E<H^@G]1“105*@*G?@*A=1R5*< -9S 8’T
ix5 mVF<—/=N4ðF<H@*A=1tZ[N;@.@*G?@*Y9<?Až@78Q/EA=@.G]1Cò;<HLI-RS /E@7:=<H:…@O«¹@*57/=N_1R>_1\@*8)ix1Qm ƒPG]1Cò;<HL -0S /E@*:B<H:ž@7«¹@757/EN;1R>_1
@78Âi¹–³mO‘³pML—-9AI5*N 10S GM:=@t<H8;57GH<H8;1RA øS 1Q{Dp!L—-9AIZ[Nz@„S {
| Ñ T1R>4-9:’GH-R:’L;N;8Q/E-R:siäÌ ½ È  ½ mOÈ Û Û*Û ÈiäÌ e È  e mO‘„>;@78;-9^<H8;@7^+-9:73 ÄÌD†  e"! ½ Ì K#‘ ±Ì ‹ $†  e"! ½ Ì ‹ ‘
 “†  e%! ½ &˜‘  Ì'“†  e%! ½ Ì(#˜T¦r@*^&N4@*:B/=A=@IZ[N;@ G]1.A=@*5O/21“† ö Î*) Ì@*:…G]1oZ[N;@I^@lJB-RA
1JBN4:B/211“GH-9:IL4N;8Q/E-9:>_1R>;-R:I:=<Mƒº:_-RS GH-“:=< ö ƒ ) ð9@7A=<?ÿ;5 1R8
¸ ö +Î )-, Ì † , 
ö , Ì +Î ), Ì ‹ † , 'Ì  Û
<]ms¿68;5*N;@78Q/EA=@t«0-9S A=^N;G]1R:sL_1RAE1 ö ƒ ) T
<?<ämos-R^+L;ABN;@*–—@&Z[N;@PG]1+A=@*5O/21ºZ[N;@P^+@KJB-9Ar1„JBN;:B/E11ºG?-9:oL;N48[/=-9:C>_1R>;-R::=<H@7^+L4A=@&L_10:E1ºLX-RA
@7GML;N;8Q/E-+}Ÿ/ † . 1e 0 È 35e 2 4 T

©
  
         
 !#"$
 !%&'

()+*!,.-/,&0213546,!78 :<9 ; ,= >6, ;?: = =23A@8635,&4B-/,= =DC4E-/0 : 46=2FGC<0D1 : 7&35C46,&= ; 3A46, :<; ,=&H
I :JLK HNMOQPQR MOS K IDT UWVXU P/Y[Z J \ UWV]_^`U P )
Iba JLK HNMOQPQR MOcP S K IDT UWV U P/Y[Z JX\ T ( ]$U P UWV YdZ )
I 7 JeK Hgf O R f O S K Ibh J \ h SW7&C46=23A>6,0 : 4i>6C : f Okj 0D351,0DCl7C1mCkf Oon ,&= j : 7&35Clp,7-DC0D3 :<;rq
; 86,&@<Cm7C1mClMO n ,= j : 73ACsp,7-DC0D3 :<; )
I > JLK HNMOctvuwt R MO S K IGx J \zy|{ Ibx J )
^ )}o, : 4l~ q€ ,&= j : 7&35C= p,7-/C<0D3 :w; ,&=) *!,&1mC=-/0 : 0ƒ‚v86, K HN~„R  ,=X864 : -/0 : 46=2FGC021 : 7&3…C<9 4
; 3A46, :<; =23 q =†C 9 ; C€=D3
K Ib‡ˆVD‰<VŠ]_‡ P ‰ P JX\ ‡ˆV K Ib‰<V J ]‹‡ P K Ib‰ P J Œ ‡VŽ‡ P qŒ ‰V&/‰ P+ ~ 
=23A,&4i>6C  \ MO‘CE9 f O S6=2,&@Q869 4’7&C020D,= j C46> : )
“i)+*!,1C=-/0 : 0”‚v86,•7 : > : 864 : >6, ;A: = =D35@863A,4B-/,&= -D0 : 46=FGC0D1 : 73AC46,= =DC4 ; 3546, :w; ,&=)
I :JLK H6f OB– R f O t S K IG‰ J \ x—‰ S6=D35,&46>iC x  f O tBu – )
Iba JLK H˜™Rš˜S K I[› J \ I y P ] ( J › )
I 7 JeKgœ Hw TŸž /¡ Y R¢ T£ž¤D¡ Y S >6,¥646V 3A> : j C0 Kgœ I§¦ J \©¨ ¦ S67C4 ¨   TŸž /¡ Y )
I > JLK H< TŸªi ( Y R MO S K I ¨«J \­¬ ® ¨ I y2J|¯By )
I , JeK H< P I MO J Rš I MO J S K I ¨«J \°¨¤± ²³¨ ) I  P I MO J ,&= , ; ,= j : 7&3AC€>6,!FG8i467&35C46,&= >6C<=”p,&7,&=
>i,&0D3dp : a ; ,= 7&C4E7&C<4v-D3A4v8635> : >«) J
I F JLK H6f O tvuwt R f Ov´ u<µ S K Ib¶ J \ x—¶· S67&C4 x  f Ov´ uwt q ·  f O tvu<µ )
¸ )}o, : 4k~mS  qE¹ ,= j : 73AC= p<,&7-DC0D3 :<; ,= =DC a 0D,º, ; 1m3A=21Cs786,&0 j Cm>6,º,&=27 :<;A: 0D,=&)
I :J *!,186,=2-/02,‚v86, ;?: -D0 : 4i=2FGC021 : 73…C9 4_3A>6,4B-/3A> : >¼»„H!~ R ~mS» IG‰ J½\ ‰ S ,&=l864 :
-D0 : 4i=2FGC021 : 73…C9 4 ; 3A46, :<; )
Iba J *!,186,=2-/02,¾‚v86, ;A: -D0 : 4i=2FGC021 : 73…C9 4l4N8 ;?:L¿ >6,¾~À,&4  S†>i,¥†4635> : 7&C1mC ¿ IG‰ J \ ª
j : 0 : -/CN>6C ‰  ~mS6,&=+864 : -/0 : 46=FGC0D1 : 7&3 C9 4 ; 3546, :w; )
I 7 J *!,186,=2-/02,¾‚v86, j : 0 :lÁ ,=D7 :w;?: 0S ;?: -/0 : 46=2FGC<0D1 : 7&3 C9 4 K«Â Ho~™R ~À7&C4 K«Â IG‰ J\ÃÁ ‰ S
,= ; 3A46, :<; )
I > J *!,186,=2-/02,E‚N8i,’=23 K H~ÄR  q³Å H  R ¹ =DC4$-D0 : 46=FGC0D1 : 73AC46,= ; 3546, :w; ,&=
,4B-/C467,&= ;?: 7&C<1 j C=D357&3 C9 4>6,E, ;5;?: =SX,&=€>6,&73A0S Å$Æ'K H”~ÇR ¹ SX- : 1 a 3.,&9 4$,=€864 :
-D0 : 4i=2FGC021 : 73…C9 4 ; 3A46, :<; )
(
Èo)}o, :!K HNMO P R MO P ;?: -D0 : 4i=2FGC021 : 73…C9 4 ; 3A4i, :<; >6,¥†4i3A> : j C0 K IGU J \ x+U ) É : 0 : 7 : > : 8i46Cº>6,
;>6C, =—=27&3A8 @86>635,&0 4B-/>i,C=!>67 ,': =Dp C,&9=0S«-/>6357&,&,&=2=7&H 023 TËa ªÌ: ª @,CS 1³T ( ,9 -/ª 023A7 : S 1T (Í,4B( -/, ;?: TŸ-Dª 0 : ( 46=FG)C0D1 : 73…C9 4 qkʆ:w;A; , ;?: 3A1 : @,4
; : : YdZ Y[Z Y[Z q YdZ
3 J x \ÄÎϪ ª I Ï  MO J 3A3 J x \ÄÎ ª( ² ª( 3A353 J x \ÒÎϪ ª( I Ï  MO J
Ñ
Ï Ð Ð Ð
3dp J x \ Î =D,&4 Ib‡ J 7C= Ib‡ J Ib‡  MO J p J x \ Î ² ( ª pN3 J x \ Î ª( ªª )

7 
C = b
I ‡ Ó
J ² D
= &
, 4 b
I ‡ J ª ² (
Ð Ð Ð
Ôi)}o, :K H!MOÕÖR MO€Õ ;A: -/0 : 46=FGC0D1 : 7&3 C9 4 ; 3A46, :<; >i,¥†4635> : j C<0 K IGU J \ xU )*!,&=D70D3 a :
@,C1³,9 -/023A7 : 1,4B-/, ;A: -D0 : 4i=2FGC021 : 73…C9 4e=23 x ,&=+7 : > : 8i4 : >i, ;A: = =D35@8635,&4B-/,=+1 : -D0D3A7,&=H
( ª ª ( ª ª ² ( ª ª
3 JÑ×ØÙ ª ª ª Ü 3A3 JÑ×ØÙ ( Ü ª ª 3A353 J×ØÙ ² ( ª Ü
ª
ª ª ª’ÚËÛ ª ª ² ( ÚËÛ ª ª ( ÚËÛ
7&C= Ib‡ J ² =2,&4 Iއ J ª 7C= Ib‡ J ² =D,4 Iއ J ª
3dp JÝ×ØÙ =D,4 J 7C= J Ü  MO J p Jß×ØÙ =D,4 Iއ J 7C= Iއ J ª Ü Ib‡  MO J )
Þ
I ‡ b
I ‡ ª Þ
I ‡
ª ª ( ÚÛ ª ª ² ( ÚÛ
à )+á 46786,&4B-/02,4—869 7 ; ,Cs,•351 : @,&4e>6, ;A: = =D35@863A,4B-/,&= -D0 : 46=FGC0D1 : 73AC46,= ; 3546, :w; ,&=H
I :JLK HNMO PQR MO Õ>6,.¥†4635> : j C0 K IG‰ J \ x—‰ 7C4
( ^
x \ ×ØÙ ª ² ( Üsâ
^ ( ÚËÛ
Iba JLK HNMO P u P R MOS K IGx J \ ¶kx!ã 7&C<4 ¶ \ T ( ^ Y , ã \ T ( ² ( Y Z )
I 7 JeK H˜™R MO P S K I[› J \ T ›rIbª J › ± Ibª J Y[Z )
I > JLK H< TŸªi ( Y R¢ TŸªi ( Y S K I ¨gJ I y2J \ I ( ] y P J2¨ I y2J )
I , JeK H<äP I MO J Rӝ I MO J S K I[› J \ › ± )
åi)}o, : ~Ó864z,&= j : 73AC³p,&7.-/C023 :<; >i,Ý>i3A1m,&46=23…C9 4Ì¥†4i35- : 7C4 j 0DCN>6867-DC_354B-/,&0246C6S=D, :³æ 864
=28 a ,= j : 7&3ACl>6,~ q =D, :lçrè H6~ÀR ~ ;?:m: j ; 3A7 : 7&3…C<9 4’‚v86, : =D35@4 :: ‰  ~=D8 j 0DC q ,7&73…C9 4
C0-/C@<C4 :<; =DC a 0D, æ )—*!,&186,=2-D0D,‚v86, çrè ,&=!8i4 : -/0 : 46=FGC0D1 : 7&3 C9 4 ; 3A4i, :<; ) é€ê'8i3.,&9 4i,&=—=2C4
ë 8 I çŠè†J ,ºìí1 I çrèiJ2î
ïi)}o, :EK Hƒf O – R f O t Sr7&C<4 K Ib‰ J!\ x—‰ q x  f O tBu – )L*!,&186,&=-/0D,m‚v86,mìí1 I K—J 7C3A4i7&3A>i,
7C4E7&C ; Ibx J S6, ; ,&= j : 7&35C7C ; 861m4 : >i, x S q ‚v86, ë 8 I K—J 7&C35467&35>6,'7&C<4 ë 8 ; IGx J )X*!,&>686ð7 :m:
j : 02-/350”>6,º,&=-/Cz8 9 ; -D3A1mC‚v86,•=D3 x -/35,&46,'0 : 46@C Ï S†,4v-DC467,&=+>63A1 I ë 8 I K—J2J \­ñò² Ï )
( ª )+á 46786,&4B-/02, a : =D,&= j : 0 : ë 8 I K!J ,ºìí1 I K—J S†,&4’7 : > : 864iC>6, ; C= =D35@863A,4B-/,&= 7 : =DC=H

^
I :JLK >6,¾MOQó+,&4kMOcô>6,¥†4i3A> : j C0 K bI ‰ J \ x—‰ 7&C4
( ^ ( ^ ²“
x \ ×ØØØÙ ¸“ Ôå ¸È ² (( ² ^( Ú Û â
² ^ ² ¸ ² “ “ ² È Ü ÛÛ
Iba JLK H6f O ՔR f O P…S K IDTŸhõVXh P h Õ/Y[Z JX\ T£ö÷hõV†]h P ]h Õ ² ^<hõV6]Ñ^wöøh P ]Ñ^wö÷h Õ2YdZ Sv7&C4i=D3A>i,&0 : 46>6C
f O Õ f O P j 0D3A1m,&02C 7&C<1Cf O n ,&= j : 7&35C=p<,&7-DC0D3 :<; ,= q; 86,&@C7C1mCÝMO n ,= j : 7&3AC<=
p<: ,&7.-/C023q:<; : ,&=&)
(()}o, :½K Hˆ~ùR  864 : -/0 : 46=2FGC021 : 7&3…C<9 4 ; 3A46, :<;ƒq =D, : 4 æ™qÑæ ± =D8 a ,&= j : 73AC=¾>6,E~ qß
02,&= j ,&7.-/3dp : 1,4v-D,<)*!,&186,&=-/0D, ; Cm=23A@8i3A,&4B-D,<H
I :JLK I æ J ,&=+=D8 a ,= j : 73ACm>6,  )
Iba JLK'ú V I æ ± J \„û ‰  ~cü K IG‰ J  æ ±?ý ,&= =D8 a ,= j : 73ACm>6,¾~)
I 7 J }N3 æÌ\ @,&4 û ‰<V â&â ⠍/‰õþ ý ,&4B-DC467&,= K I æ J\ @,4 û…K Ib‰<V J  â â ⠍ K IG‰`þ J ý )
( ^ )+á 46786,&4B-/02, a : =D,&= j : 0 : ë 8 I K!J ,ºìí1 I K—J ,4k7 : > : 8646Cm>6, ; C= =23A@8635,&4B-/,= 7 : =2C=&H
I :JLK H˜ Õ R MO P…S K Iÿ› JX\ T ›rIÞª J ›rI ( J Y[Z )
Iba JLK H˜ P R MOQՅS K Iÿ› JX\ T ›rI y ® J › ± I y ® J › ± I y ® J Y[Z S†7&C4 y ®  MO—)
I 7 JeK H !P u P”R MOQP u P…S K IG¶ J \ x—¶ ² ¶ x S67&C<4
x \ Î ^ ( ² (( â
Ð
(…“i)}o, :ÑK HQ~¢R  864 : -/0 : 46=2FGC021 : 7&3…C<9 4 ; 3A4i, :<; )*!,-D,&021354 : 078 :<9 ; ,=€>6, ;?: =s=23A@8635,&4B-/,=
j 02C j C=D357&3AC<46,&= =2C4’p<,&02> : >6,0 : =S 28i=2-/3d¥†7 : 46>6C ;?: 0D,= j 8i,&=2- : )
I :J }N3 û ‰V& â&â ⠍/‰õþ ý ,&= ; ) >«) ,4B-/C467,&= û…K Ib‰<V J  â&â ⠍ K IG‰`þ J ý ,&= ; )Ë>«)
Iba J }N3 û…K IG‰V J  â â&⠍ K Ib‰õþ J ý ,= ; )Ë>«) ,&4B-/C<467&,= û ‰<V& â â ⠍/‰õþ ý ,&= ; )Ë>«)
I 7 J }N3 û ‰V& â&â ⠍/‰õþ ý ,&= ; ) 3Þ)ƒ,&4B-/C<467&,= û…K Ib‰<V J  â â ⠍ K Ib‰õþ J ý ,&= ; ) 3Þ)
I > J }N3 û…K IG‰V J  â â&⠍ K Ib‰õþ J ý ,= ; )Ë3ø)ƒ,4v-DC467,&= û ‰V& â&â ⠍/‰õþ ý ,&= ; ) 3Þ)
I , J }N3 ë 8 I K!J+\òû ª ý ,&4B-DC467&,= û ‰<V â&â ⠍/‰õþ ý ,= ; )Ë3ø)º=D3 q =†C 9 ; Ce=D3 û…K IG‰V J  â â ⠍ K Ib‰õþ J ý ,=
; ) 3Þ)
I F J }N3 û ‰<V& â â ⠍/‰ t ý ,=Q864 : a : =2,!>6,'~ qEû…K IG‰V J  â â&⠍ K Ib‰ t J ý ,= ; )Ë3ø)r,&4B-/C<467&,= ë 8 I K!JX\
ûªý)
( ¸ )}o, :LK Hg~ R  864 : -/0 : 46=2FGC<0D1 : 7&3 C9 4 ; 3546, :w; )¾*!,&7&35>63A0=D3 ;A: =!=D35@8635,&4B-/,= j 0DC j C=23A7&35C46,=
=2C4’p<,&02> : >6,0 : =+CsF :<; = : =&S 28i=2-/3d¥†7 : 46>6C ;?: 0D,= j 8i,&=2- : )
I :J }N3 ë 8 I K—J \Ãû ª ý S ;?: ,7&8 : 73…C9 4 K Ib‰ J \ -D3A,46, :m; Cs=D861mC€864 : =DC ; 867&3 C9 4«)
“
Iba JŠ: ,&78 : 7&3 C9 4 K Ib‰ J \ =D35,&1 j 0D,!-D3A,46,º=DC ; 867&3 C9 4e=D3 ë 8 I K—JX\„û ª ý )
I 7 J ìí1 I K—J'\  q ë 8 I K!J'\ û ª ý =23 q =6C 9 ; CE=23 ;A: ,&7&8 : 7&3 C9 4 K Ib‰ J'\  -D3A,46,=2C ; 8i7&3…C<9 4
869 4i3A7 : j : 0 : 7 : > :
   )
I > J Š: -/0 : 46=2FGC<0D1 : 7&3 C9 4 K ,=+354Bp,&02=D3 a ; ,º=D3 q =†C 9 ; Cs=D3äìí1 I K—J \° q ë 8 I K!J \„û ª ý )
I , J }N3r~ \­ S ë 8 I K—J \Ãû ª ý =23 q =†C 9 ; C€=D3äìí1 I K—J \ ~m)
I F J }N3ä>63A1 I ~ JX\ >63A1 I „J \ S ë 8 I K—J \ û ª ý =D3 q =†C 9 ; Cs=D3Šìí1 I K—J \° )
I @ J }N3†>63A1 I ~ JX\ >6351 I „J \ ,&4B-/C<467&,= K ,&=X3A4 q ,7-D35p : =D3 q =6C 9 ; C'=23 K ,&=X=2C a 0D, q ,7-/3dp : )
I ʆJ }N3ä>63A1 I ~ J >63A1 I „J S K 46C j 86,&>6,•=D,0 3A4 q ,7-/3dp : )
I 3 J }N3ä>63A1 I ~ J  >63A1 I „J S K 46C j 86,&>6,•=D,0 =DC a 02, q ,&7-D35p : )
I  J }N3~š,&=’>6,ß>i3A1m,&46=23…C9 4¼¥†463d- :$q K ,&= a 3 q ,&7-D35p : S  ,=½>i,ß>6351,46=D3 C9 4¼¥†4635- :³q
>i3A1 I ~ J \ >63A1 I  J )
(õÈo)}o, : 4 x  f O tvu – q ·  f Ov´ uwtN)c*!,186,=2-/02, ; C€=D35@863A,4B-/,<H
I :J 7C ; Ib·x J  7C ; Ib· J )
Iba J 7C ; Ib·x JX\ 7&C ; IÞ· J =23ä0 : 46@<C Ibx JX\ )
I 7 J ë 8 ; IGx J  ë 8 ; Ib·x J )
I > J ë 8 ; IGx JX\ ë 8 ; Ib·x J =23ä0 : 4i@C IÞ· J \ )
I , J 0 : 46@C IÞ·x J  1m3A4 I 0 : 46@C IÞ· J  0 : 4i@C IGx JDJ )
I F J }N3ä0 : 46@<C Ibx J \ ,&4B-/C4i7&,&=+0 : 46@C Ib·x J \ 0 : 46@C IÞ· J )
I @ J }N3ä0 : 46@<C IÞ· J \ ,4B-/C467,&=+0 : 4i@C IÞ·¾x J \ 0 : 46@C Ibx J )
}o8i@,&02,&4673 : H7&C46=23A>6,0D, ;?: =ƒ-D0 : 46=FGC0D1 : 73AC46,= ; 3A4i, :<; ,&= K Ib‰ J \ x‰ qÅ I  J \ ·  q -/,46@ :
,4½786,&4B- :m;?: j 023A1m,&0 j : 02-D,•>i, ; , 2,0D73A7&35Cmïi)
(…Ôi) QC4i=D3A>i,&0D, ;A: -/0 : 46=FGC0D1 : 7&3 C9 4 ; 3A46, :<;+K Hf O t R f O t >6,¥6463A> : j C0 K IG‰ J½\ x‰ 7C4
x  f O tBuwt )X*!,186,=2-/02,º‚N8i, ;A: = =D35@863A,4B-/,&= j 02C j C=D357&3AC<46,&= =2C4E,&‚v8635p :<; ,&4B-/,=&H
I :JLK ,&= 3A4Bp,0D=23 a ; ,<)
Iba J áo3A=-/, x ú V )
I 7 J á ; =23A=2-D,&1 : >6,º,&7&8 : 7&35C46,&= ; 3546, :w;gÊ C1mC@,9 46,&C x+U \ ª -/35,&46,º=2C ; 8673…C9 4°869 4i3A7 : )
I > J á ; =23A=2-D,&1 : >6,º,&7&8 : 7&35C46,&= ; 3546, :w; ,&= xU \ ¡ -/35,&46,'=2C ; 8673…C9 4 j : 0 : 7 : > : ¡  f O t )
I , J >i,- IGx J \  ª )
( à )+á 46786,&4B-/02, ;?: 354Bp,&02= : >6, ;?: -D0 : 46=FGC0D1 : 73…C9 4 K HNMO Õ R MO Õ S K I2T£UWV U P U Õ/Y Z JX\ T UWV ² U P ]
U Õ ^`UWVEUWV]$U P/Y Z )
(…åi)+á 46786,&4B-/02, ;A: 3A4Bp,0D= : >6, ;?: -D0 : 46=FGC0D1 : 73…C9 4e>6,.¥†4635> : ,&4’, ; , 2,0D73A7&35C à >«)
(…ïi)+á 46786,&4B-/02, ;A: 3A4Bp,0D= : >6, ;?: -D0 : 46=FGC0D1 : 73…C9 4e>6,.¥†4635> : ,&4’, ; , 2,0D73A7&35CE( ^wa )
¸
^<ª )}o8 j C4i@ : ‚N8i, K H«~ R  qÅ H  R ¹ =2C4-/0 : 46V =2FGC021 : V 7&3AC<46,&=V ; 3A46, :<; ,=—3A4Bp<,&0D=23 a ; ,&=)
*!,186,&=-/02,•‚v86, ÅkƔK ,= 3A4Bp,0D=D3 a ; , q ‚N8i, I ÅkƔK!J/ú \¼K'ú Æ+Åcú )
^ ()}o,X>6357&, ‚v86,X864 : -/0 : 46=FGV C0D1 : 7&3 C9 4 ; 3A46, :<;wK HB~™R  ,=r8i4•35=DC1mC0¥†=D1mC =D3 K ,&= a 3 q ,&7.-/35p : S
,=!>6,&73A0S«=D3, o3A=-/, K'ú )'ÉrC0!C<-D0 : j : 02-D,<S«=2,>63A7,‚v86,€~ò,='3A=2C1C<02FGC :E =23ˆ,i35=2-D,864
35=DC1mC02¥6=D1mC K >6,¾~À,&4  )
I :J É 02C a : 0 ‚v86,~ ,= 3A=DC<1C0FGC : = 9 1m3A=D1mC6)
Iba J É 02C a : 0 ‚v86,•=D3r~À,&= 3A=2C1mC02FGC : ,4B-/C467,&=  ,&= 35=DC1mC02FGC : ~€)
I 7 J É 02C a : 0 ‚v86,'=23Š~,&= 35=DC1mC02FGC :€ qe ,= 3A=2C1C<02FGC :€¹ ,4B-/C467,&=—~™,&= 35=DC1mC02FGC
:¹ )
I > J }N, : ~Ò864_,&= j : 7&35CÖp,7-DC0D3 :<; =DC a 0D,  I  \ MO C³9 f O Jq =2, : · \šû ‰V â â â /‰ t ý
8i4 : a : =2,C02>6,&4 : > : >i,s~m)+*!,186,&=-/02,‚v86, ;A: -D0 : 4i=2FGC021 : 73…C9 "4 !#‹Hi~ÀR  t ‚v86,
:!#=D3AIb@<‰ 4 :e: 7 : t > S6: ,&=+‰ 864E3A~ =2C1=D8ÑC<027&¥†C<=20D10D,C= j I Cp46, >635,&, 4B-/,$,s2,&p<02,&7&73A7-D3ACC 0º^<>6ª ,s>67&, CNC02>6) Ér,&4 )r: ( > :) =',&4 ;A: a : =D, · S
J  : ; ;&% J
I , J *!,186,=2-/02,'‚v86,'=D3Š~ qE =DC4e>6,'>i3A1m,&46=23…C9 4m¥†4635- : Si,4B-/C467,&=~™,= 3A=DC<1C0FGC :€
=23 q =†C 9 ; Cs=D3 ;A: >i3A1m,&46=23…C9 4e>6,¾~À,&= 35@8 :<;ä:s;A: >i3A1m,&46=23…C9 4e>6,  )
^^ )}o, :LK H¤˜ P R ˜ Õ - :<; ‚N8i, j : 0 : 7 : > : ›  ˜ P S K Iÿ› J+\ y ›m] û › ² ›rIÞª J ý ü y )áX4i7&86,4v-D0D,
T K Y #('ˆS ;A: 0D, j 0D,&=2,&4B- : 7&3…C<9 4m1 : -D0D3A73 :<; >6, K 0D,= j ,&7.-/C•>6, ;?: = a : =D,= C0D>i,&4 : > : =&H · \ û ( ( ]
y ( 7&] 8 y , ] 86y ,P @ý C q )„I \Ã^ û] ( “ ( ²y P ( >6]_,º^ >6y C] = FGy CP 021 ( ² = “ >iy 3A=2] -D3A“4By - P =&²ß) y Õ ý )
:<; ; ; K ² y²ßy J : :
^ “i)}o, :K Ho˜ t R ˜ t S K Iÿ› J”\ › ± ) * :<;5; , ;?: 0D, j 0D,=D,4v- : 7&3 C9 41 : -/023A7&3 :<; >6, K 0D,&= j ,7-DCL>6, ;A:
a : =D,º7 : 4 C9 463A7 : >6,'˜ t )
^ ¸ +) * :<;A; , ;?: 0D, j 0D,=D,4v- : 7&3 C9 41 : -/023A73 :<; >6, ;?: -/0 : 46=2FGC<0D1 : 7&3 C9 4 K >6, ; $, 2,&027&3A73AC à a SŠ02,&= j ,&7.-/C
>6, ;A: = 7&C020D,= j C46>635,&4B-/,= a : =D,&=+7 : 4 C9 46357 : =)
^ Èo)}o, : 4 K H«~ R  qÑÅ H  R ¹ -/0 : 46=2FGC021 : 7&3AC<46,&= ; 3A46, :<; ,= q =2, : 4 · S )q-, a : =2,&=
C02>6,&4 : > : = >6,¾~S  q’¹ S†02,&= j ,&7.-/3dp : 1,4v-D,<)É 0D86, a ,•‚v86,
T ÅkƔK Y #(. \ T Å Y '/. T K Y #(' â
^ Ôi)}o8 j C4i@ : ‚N8i, x 0D, j 0D,=D,4v- : :e;?: -/0 : 46=FGC0D1 : 7&3 C9 4 ; 3546, :w;K H«~ R ~ 02,&= j ,&7.-/C½>6,€864 :
a : =D,ºC0D>i,&4 : > : · )X*!,186,=2-/02,¾‚v86, x ´ 02, j 02,&=2,&4B- ::sK ´ S j : 0 : Ï  1» 0ÝS 0D,&= j ,7-DC>6, · )
^ à )}o, :€K Ho~ R  864 : -D0 : 46=FGC0D1 : 73…C9 4 ; 3546, :w; Si=2, : 4 · "q ) a : =D,=—C02>6,&4 : > : =+>6,¾~ q½
02,&= j ,&7.-/3dp : 1,4v-D,<S q =2, : x ;A: 02, j 02,&=D,4B- : 73…C9 4E1 : -/023A7&3 :<; >6, K 02,&= j ,7-/Cs>i, ;?: = a : =2,&= · q
) )X*!,&18i,&=2-D0D, ; C€=23A@8635,&4B-/,wH
I :J ‰  ë 8 I K—J =D3 q =†C 9 ; Cs=D32!# Ib‰ J  ë 8 ; IGx J )
Iba 3J   ìí1 I K!J =23 q =†C 9 ; C€=D23 !' I !J  7&C ; IGx J )
È
I 7 JeK &, = 3A4Bp,0D=23 a ; ,•=2V 3 q =†C 9 ; C€=D3 x 46C€,&=+=D3546@8 ;A: 0&)4—>6,1Ì: 9 =S x ú V ,&= ;?: 02, j 02,&=D,4B- : 73…C9 4
1 : -D0D3A73 :<; >6, K'ú 02,&= j ,7-/C>6, ;?: = a : =D,= )q · )
^ åi)}o, : ~Ò8i4_,= j : 73ACp,7-DC0D3 :<; 7&C<4 · \Çû ‰V5/‰ P /‰ Õ ý 864 : a : =2,<)‹}o, :ÑK Hc~¢R ~ù864 :
-D0 : 46=FGC0D1 : 73…C9 4 ; 3A46, :<; - :<; ‚v86,
{ ² ( {
T K Y # \ ×ØÙ ( { ( Ü
{ ² ( ^ { ÚÛ
I :J }N8 j C<463A,46>6C‚v86,!~ ,&=c864L,= j : 73AC•p,&7.-/C023 :<; 0D, :<; S ʆ:w;A;?: 0 ; C=Xp :<; C02,&=X>6, {  MO j : 0 :
‚v86, K 4iC=2, : =DC a 02, q ,&7-D35p : )
Iba J ìí>i,&1 I :BJ j ,0DC¾=D8 j C4635,&46>6C‚N8i,'~ ,=Q864l,&= j : 7&35C¾p,7-DC0D3 :<; 7C1 j ; $, 2C q ‚v86, {  f O )
I 7 J É : 0 :'; C=p :<; C0D,= ʆ:<;5;?: >6C<=,4 IGa J S<,&=2-D86>63 : 0ƒ=D3 ‰ \™² ‰V<]löø‰ P ]Ibö ² ( J ‰ Õ j ,02-D,&46,7&, :
;A: 351 : @,&4€>6, K ) á 4l7 : =DC : ¥†0D1 : -/3dpC6S Ê6:<;A;A: 0ˆ-/CN>6C= ; C=   ~°- :<; ,&=X‚v86, K 6I  J \ ‰ )
^ ïi)}o, :+K HNMO Õ RӘ P - :<; ‚v86, j : 0 : 7&35,&02-DC= ‡c8 7  MOƒS K IDT (s(m( Y Z JX\ 9^ 7]‡ y S K IDTŸª ² (s( Y Z JX\
‡ y :] 7 y P qLK IDTŸªÍª ( Y Z J \ 7e]­Iއ ² ( Jíy )
I :J * :<;A;A: 0 ‡ q 7 j : 0 : ‚v86, K 4iC=2, : 3A4 q ,7-/3dp : )
Iba J * :<;A;A: 0 a : =D,&=+>6, ë 8 I K—Jcq >6,ºìí1 I K—J ,&4EFG864673…C9 4E>6, ‡ q 7 )
I 7 J * :<;A;A: 0 ;A: 351 : @,&4e>6, ; =D8 a ,&= j : 73A<C ; \„û U  MO ՗H UWVr]³U Õ \ U P ]³U Õ \ ª ý S†=D,@”869 4
; C=”p :<; C<0D,&= >i, ‡ q 7 )
“ ª +) * :<;A; , ;?: 0D, j 0D,=D,4v- : 7&3 C9 4l1 : -D0D3A73 :<; Sw02,&= j ,7-/C¾>6, ;?: a : =2,+7 : 4 C9 4i3A7 : SN>i, ;A: -/0 : 46=2FGC021 : 7&3…C<9 4
; 3A46, :<;WK H6f O P R f O P ‚N8i,•p<,&0D3d¥†7 : H K I2T ( ]³ö ( ² ö Y Z JX\ T ( ö Y Z qLK IDT ( ö Y Z JX\ T ² ( ª Y Z )
“i(+) * :<;A; , ;?: 0D, j 0D,=D,4v- : 7&3 C9 4l1 : -D0D3A73 :<; Sw02,&= j ,7-/C¾>6, ;?: a : =2,+7 : 4 C9 4i3A7 : SN>i, ;A: -/0 : 46=2FGC021 : 7&3…C<9 4
K=23A,&Ho4iMO>6C Õ R MO Õ U ‚v86,•MO = : Õº-D3AH =2F UW: 7&V, ]_; C€U =D35@86U 35,&4B-/,wH ª K ) Ib‰ I JXQ\C46^w=D35‰ >6,&=D023 ,‰ ,  j æ 02Coqe>i86K 7Ib-D‰ CLJX3A\™4B-/,² 0D‰46C=D3 7 ‰ 4  C9 46 æ 3A7= C S
>6,¾MOcÕ ) J æz\Àû  P² Õ\ ý ; :
“ ^ )}o, : 4 · \Óû ‰<5V /‰ P /‰ Õ ý 8i4 : a : =D,’C0D>i,&4 : > : >6,k~ >q )¢\¢9û  ?V  P  Õ  ô ý 864 : a : =2,
C02>6,&4 : > : >6,  )@QC46=D35>6,&02, ;?: -/0 : 46=2FGC<0D1 : 7&3 C9 4 ; 3A46, :<; K H!~ R  ‚N8i, = : -D3A=F : 7&,wH
K Ib‰<V JX\  VN]  P ]  Õ A²  ô S K Ib‰ P JX\  V A²  P ]k^  Õ ] “  ô qK IG‰ Õ JX\ ^  Vo] “  Õ ]k^  ô )
I :J á 46786,&4B-D0D, a : =D,= j : 0 : ë 8 I K—J ,'ìí1 I K—J )
Iba J * :<;A; ,—-/CN>6C= ; C= ‰  ~- :<; ,= ‚N8i, K Ib‰ J \ ^  P ²B Õ ²C ô )
I 7 J á 46786,&4B-D0D, ;?: 0D, j 0D,=D,4v- : 7&3 C9 41 : -/023A73 :<; >6, K 02,&= j ,7-/C'>i, ;A: = a : =D,= · ± \„û ‰<5V ^w‰ P ]
‰ Õ /‰ P ]³‰ Õ ý q ) ± \ 9û  ?V  P  Õ ]  ô  Õ :²  ô ý I éÉC0 ‚v8c, 9 · ± q ) ± =DC4 a : =2,&= îõJ )
““i)}o, : · \ û ‰?V D‰ P D‰ Õ ý 864 : a : =D,C<02-/C<46C0D1 :<; >6,ÑMOmÕe7&C<46=D35>6,&0 : 46>6C$, ; j )Ë3ø)Ã7 : 4 C9 46357&C6)
á 46786,&4B-/02, ;A: 1 : -D0D35ð!>6, ;?: -/0 : 46=FGC0D1 : 7&3 C9 4 ; 3546, :w;WK HNMOcՔR MOQÕ0D,&= j ,7-DC>6, ;A: a : =D, ·
,4½7 : > : 8646C€>6, ; C= =23A@8i3A,&4B-D,&= 7 : =2C=&H
Ô
I :JLK ,&= ;?: 02,?D†,i3 C9 4’0D,&= j ,7-DCl>6, ; j ;?: 4iC æ_\ @,&4 û ‰<V?/‰ P ý
Iba JLK ,&= ;?: 02,?D†,i3 C9 4’0D,&= j ,7-DCl>6, ;?: 02,&7.- :sæÌ\ @,&4 û ‰ Õ ý
I 7 JeK ,&= ;A: 0DCw- : 73…C9 4 :w; 0D,>6,&>iC0!>6, ; ,2, æ°\ @,4 û ‰ Õ ý ,4Ñ864 : 9 46@8 ; C ‡ S«7&C4i=D3A>i,&0 : 46>6C
‚v86,•, ; =2,&4B-/35>6Cm>6,º0DC<- : 7&3 C9 4 j C<=D35-D35p<Cm,= >6, ‰V ʆ: 73 : ‰ P )
“ ¸ )+* :<;A; , ;?: 0D, j 0D,=D,4v- : 7&3 C9 4k1 : -D0D3A73 :<; 0D,= j ,&7-DCl>6, ;?: a : =D,•7 : 4 C9 463A7 : >i, ;A: -/0 : 46=2FGC021 : 7&3…C<9 4
Kªi HoUWMO V ՔR U MO ՗^`‚vU 86,•,&= ª ;?: ) 0D,D†,o3 C9 4k02,&= j ,&7.-/C>6, ;A: 0D,&7.- :€æ¼\™û U  MO ՗H UWVˆ]³U P ]‹U Õ \
² P² Õ\ ý
“BÈo+) * :<;A; , ;?: 0D, j 0D,=D,4v- : 7&3 C9 4k1 : -D0D3A73 :<; 0D,= j ,&7-DCl>6, ;?: a : =D,•7 : 4 C9 463A7 : >i, ;A: -/0 : 46=2FGC021 : 7&3…C<9 4
K>6,€HN>6MO C<Õ =ºR FGC<0DMO 1 Õ ‚v='86>6,!3A=,-/= 354v;A:- 02=&?, HºD†3, i3 1C9 4L,>6023 ,&= 4Bj -D,&,s7.-/86C4 >i, 1 ; j ;?-D: 0D4i35ðC >6æÍE, \„*Cû 8iU =D, MCO Õ >6,H 0&UWSrV 3A3 ]k,&U 1 P ]kj ,U Õ 46\ >6CEª ý, S
$, 2,&0D73A73ACm““i)
: : J : : : Ê ; J ;: ;
“Ôi+) * :<;A; , ;?: 0D, j 0D,=D,4v- : 7&3 C9 4k1 : -D0D3A73 :<; 0D,= j ,&7-DCl>6, ;?: a : =D,•7 : 4 C9 463A7 : >i, ;A: -/0 : 46=2FGC021 : 7&3…C<9 4
K HŠMO—UÕ¾R MO MO—Õ+Õ H Sr‚vUW86V,m]$,&U =º86]$4 : U 0DC<- : ªo7&3 C9 UW4ÑV >6, U ¸ GÈ ]ÍFº,&^`4ÝU =D,4v-Dª 3A>6C ) : 4B-/3 Ê C0 : 0D3AC :<; 02,&>6,>6C0º>6, ; ,$2,
æ_\ û  P Õ\ ² P Õ\ ý
“ à )+á 46786,&4B-/02,k864 : -/0 : 46=FGC0D1 : 7&3 C9 4 ; 3546, :<;cK H”MOՒR MOՅS K \I »†Sc- ‚N8i, IGUŠ
I¯ K I§U J  æ”JXŒ U  MOcÕ S†=D3 æÍ\ @<,&4 û T (Ì(Ì( Y[Z ý ) I QC<46=D35>6,&02,•, ; j  )Ë3ø) H 7 : 4 C9 :<46; 357&C6) J ¯ æ Je\
“åi)}o, : 4³~ q  n ,&= j : 7&35C=¾p<,&7.-/C023 :<; ,&= I  \ MO Cß9 f O J >i,l>6351,46=D35C46,&= ñÓqJ 02,&= j ,&7 n
-D35p : 1m,&4B-/,w)L}o, :’K Hr~‘R  ; 3546, :w; )€*!,&186,&=-/0D, ; C’=D35@8635,&4B-/,wH¾=23 ·!V q · P =DC4 a : =2,&=
C02>6,&4 : > : =m>6,½~ >q ) V Kq ) P =DC4 a : =2,&=lC<0D>6,4 : > : =m>6,  SQ,4v-DC467,&=l0 : 46@C T K Y #(LM'NL \
0 : 46@C T K Y #PO$'QO&)
“ïi)}o, :€K HNMOQÕQR MO ô S K IGU JX\ x+U 7C4
( ^ ²(
x \ ×ØØØÙ ª“ ªÔ ² “( ÚËÛ â
² ( ² ^ ª Ü ÛÛ
á 46786,&4B-/02,864 j : 0+>i, a : =D,=C02>6,&4 : > : = · q · ± - :<; ,&= ‚v86,
( ª ª
T K Y #(#/R \ ×ØØØÙ ªª ª( ªª ÚËÛ â
ª ª ª Ü ÛÛ
éSá o35=2-/,4 a : =D,&= C02>6,&4 : > : = · q · ± - :<; ,&= ‚v86,
( ª ª
T K Y #(#TR \ ×ØØØÙ ªª ª( ª( Ú Û î
ª ª ª Ü ÛÛ
éÉC0 ‚v8c, 9 î
à
¸ ª )QC4i=D3A>i,&0D, ;A: = 1 : -/0D357&,=
x \ Î ² (( ² (( ² ^^ · \ Î “Ô ²² ^( ¸^
Ð Ð
I :J á 46786,&4B-D0D,1 : -D0D3A7,&= 354vp<,&02=D3 a ; ,&=VU V q ~ V - :w; ,&= ‚v86,
U VDx ~ V \ Î ª ª ª
( ª ª
Ðâ
Iba J ìí>i,&1 I :J j ,0DCm7&C4 · ,&4 ; 86@ : 0 >6, x )
I 7 J á 46786,&4B-D0D,1 : -D0D3A7,&= 354vp<,&02=D3 a ; ,&= )™3q , - :<; ,= ‚N8i, ) x ,ò\ · )
¸ ()}o, :€K HNMOQÕQR MOcP…S K IGU JX\ x+U 7C4
x \ Î ² ^( ² (( ² ^^ â
Ð
á 46786,&4B-/02, a : =D,&= · q ) >6,MOÕ q MOcP…S60D,= j ,&7.-/35p : 1m,&4B-D,<S†- :<; ,&= ‚v86,
T K Y #(' \ Î ª^ (( ^^ â
Ð
¸ ^ )}o3~ qÝ =2C4Ý,&= j : 7&35C=•p<,&7.-/C023 :<; ,&=º=DC a 0D,  I  \ MO CÑ9 f O J Sr>6,&4iC<- : 1mC=W I ~   J':<;
7C42864B-/Cm>6,'-/CN> : = ;A: =”-/0 : 46=FGC0D1 : 7&35C46,= ; 3A4i, :<; ,&=”>6,¾~À,&4  )
QC4i=D3A>i,&0D,º,X 4 W I ~   JQ;A: = =D35@8635,&4B-/,=+C j ,0 : 73AC46,=&H
I }o861 :BJ * : >6C= K¼qÅ  W I ~  „J S K ] Å ,&= ;?: -/0 : 46=2FGC<0D1 : 7&3 C9 4€>6,¥†4i3A> : >6, ;A: =23A@8i3A,&4B-D,
1 : 46,0 : H I K ] ÅJ Ib‰ J \zK Ib‰ J ] Å IG‰ J Œ ‰  ~)
I ÉX02Co>i867-DC j C0s864³,=D7 :w;?: 0 J * : >6C K  W I ~  ÅXJ¾q ‡ „ S>6,¥6463A1mC= ‡ K 1m,&>63 : 4B-/,<H
Ib‡ K—J IG‰ Jˆ\ ‡ K Ib‰ JXŒ ‰  ~)
I :J *!,186,=2-/02,‚v86,s7&C<4 ;?: =C j ,&0 : 7&3AC<46,&=>6,¥†4i3A> : =YS W I ~  „J ,&=!8i4,&= j : 7&35Clp,7-/C<0D3 :w;
=2C a 02,  )
Iba J }N8 j C<46@ : ‚v86, >6351 I ~ J \ S>6351 I „J \zñ q ‚v86, · q ) =2C4 a : =2,&= C02>6,&4 : > : =ƒ>i, ~
qs=28½ 02, j0D,&02= ,&j =2,&,4B7-D- 35p 7: 3…C1m9 4’,&4B1 -/,w) -/02É 3A7&0D3 86, a 0D, ,&= ‚vj 86,, 7-D;AC:—>6: j, ; 3A7 := 7&a 3 C9 4=2,&‚v= 86· , : =23A@4 S :—T : #(7 : ' > :—K – uwW tiISi~ ,&= „864 J
35=DC1mC0¥†=D1mC6:)ˆéZQ8 :<9 ; : ,&= ;A: >6:<3A; 1m,&4i=D3…C<9 4>6,W ;A:I ~  „: J2î á 3qÊ 3 )a : 86K 4 Y : a : Ñ=2,  >6,W I ~  „J )

å
Práctica 4 (2da Parte)- Ecuaciones Diferenciales Lineales

Rb
1. Hallar Re(f ), Im(f ), f ′ y a f (t) dt en cada caso.
1
a) f (t) = (2 + i)t + (1 − i) sen t, b) f (t) = (1 + 3it)3 , c) f (t) = 1+it .

2. Suponiendo que f, g : I → C, con I ⊂ R un intervalo, son derivables en t ∈ I, probar que


valen las reglas de derivación:

a) (cf + dg)′ (t) = cf ′ (t) + dg ′ (t) (c, d ∈ C);


b) (f g)′ (t) = f ′ (t)g(t) + f (t)g ′ (t).
 ′ ′ (t)g ′ (t)
c) fg (t) = f (t)g(t)−f g 2 (t)
si g(t) 6= 0.
d ) (f n )′ (t) = nf n−1 (t)f ′ (t) para todo n ∈ Z − {0} (f (t) 6= 0 si n < 0).

3. Se dice que F : I → C, con I ⊂ R un intervalo, es una primitiva de f : I → C si


F ′ (t) = f (t) para todo t ∈ I. Pruebe lo siguiente.

a) Dos primitivas de f difieren en una constante compleja.


Rb
b) Si F es una primitiva de f y f es continua, entonces a f (t) dt = F (b) − F (a). (Regla
de Barrow.)

4. Para z ∈ C, con z = x + iy, x, y ∈ R, se define ez = ex (cos y + i sen y). Probar que vale lo
siguiente:
′ ′
a) ez+z = ez ez para todo z, z ′ ∈ C;
b) ez = 1 si y sólo si z = 2kπi con k ∈ Z;
c) |ez | = ex , con lo cual |eit | = 1 si t ∈ R;
1
d ) para todo z ∈ C, ez 6= 0 y ez = e−z ;
e) ez = ez+2kπi si y sólo si k es entero.
eit +e−it eit −e−it
f ) cos t = 2 y sen t = 2i (t ∈ R).

Hallar todos los z ∈ C tales que ez = 1, −1, i, −i. Hallar los z tales que ez ∈ R y los z
tales que ez ∈ iR.
′
5. Sea P : I → C, derivable en t ∈ I. Probar que eP (t) = P ′ (t)eP (t) . (En particular
′
ect = cect para toda constante c ∈ C.)

6. Calcular empleando
R π 2t la regla deR πBarrow la integral 0 e(2+i)t dt y, a partir de ella, obtener
los valores de 0 e sen t dt y 0 e2t cos t dt.

7. Hallar la solución general de las siguientes ecuaciones:


i) y ′ − 2y = 0; ii) y ′ = y + (1 + i)x; iii) y ′ + 2xy = y; iv) ty ′ + y = 3t3 − 1 (t > 0).

8. Resolver los siguientes problemas a valores iniciales:


i)xy ′ = (1+x)y; y(1) = 3 ii) y ′ +y = sen x; y(0) = 0 iii) (x+1)y ′ +x2 y = e−x/2 ; y(0) = 1.

1
9. Halle la solución general de las siguientes ecuaciones resolviendo la ecuación homogénea y
hallando una solución particular por el método de los coeficientes indeterminados:
i) y ′ − 2y = 1 − x; ii) y ′ − y = (1 + x)ex ; iii) y ′ + y = 2et ; iv) y ′ = iy + teit ;
v) y ′ + iy = 3 cos 2x; vi) 2y ′ + 6y = ex cos x.

10. Hallar bases de los espacios de soluciones de las siguientes ecuaciones. (En todos los casos
la base debe estar formada por funciones reales.)
a) y ′′ + 3y ′ + 2y = 0 b) 2y ′′ − 18y = 0 c) y ′′ − 8y ′ + 16y = 0 d) y ′′ + 9y = 0
e) 2y ′′ + 2y ′ + 2y = 0 f) y ′′ − k 2 y = 0, k ∈ R g) 2y ′′ + 10y ′ + 25y = 0
h) y ′′ + 2y ′ + (ω 2 + 1)y = 0 (ω ≥ 0).

11. Encuentre la solución general de las siguientes ecuaciones empleando el método de coefi-
cientes indeterminados para hallar una solución particular:
a) y ′′ + y ′ = 3x2 b) y ′′ + 6y ′ + 9y = 18 cos 3x c) y ′′ − 4y = e2x
d) y ′′ − y ′ − 2y = ex + x e) y ′′ + y = cos ωx, (ω > 0) f) y ′′ − ω 2 y = A sen ω0 t, (ω, ω0 > 0).

12. Resolver los siguientes problemas a valores iniciales.


a) y ′′ − y ′ = 3e2x , y(0) = 0, y ′ (0) = −2
b) y ′′ − y ′ − 2y = 10 sen x, y( 12 π) = −3, y ′ ( 21 π) = −1
c) y ′′ + 4y ′ + 4y = 9 cosh x, y(0) = 0, y ′ (0) = 0.

13. Resolver los siguientes problemas con valores de frontera:


a) y ′′ − 9y = 0, y(−4) = y(4) = cosh 12 b) y ′′ − 2y ′ = 0, y(0) = −1, y(0,5) = e − 2 c)
y ′′ + y = 0, y(0) = 0, y(π) = 1 d) y ′′ + 4y = 0, y(0) = 1, y(π) = 1 .
Teniendo en cuenta los resultados obtenidos: ¿un problema con valores de frontera siempre
tiene solución?, ¿si tiene al menos una solución, ésta es única?

14. Hallar la solución general de las siguientes ecuaciones mediante el método de variación de
parámetros:
i) y ′′ − 2y ′ + y = x3/2 ex ii) y ′′ − y = 2ex /(1 − ex ) + e2x .

Problemas opcionales

1. Suponga que el ritmo al que se enfrı́a un cuerpo caliente es proporcional a la diferencia


entre su temperatura y la del ambiente que lo rodea. Un cuerpo se calienta a 100 C y se
expone a una temperatura ambiente de 10 C. Luego de 30 minutos su temperatura es de
50 C. ¿Cuánto deberá esperarse para que la temperatura llegue a los 15 C?

2. Si la resistencia del aire que actúa sobre un cuerpo de masa m en caı́da libre ejerce una
fuerza proporcional a la velocidad del mismo pero de sentido contrario (= −kv), la ecuación
de movimiento es
v ′ = g − (k/m)v.
Supongamos v(0) = 0, hallar lı́mt→+∞ v(t).

2
R i(t) L

v(t)

Figura 1:

3. Considere el siguiente circuito RL:

a) Suponiendo V (t) = V0 e i(0) = i0 , calcular i(t) para t > 0. ¿Qué sucede con i(t) a
medida que t aumenta?
b) Idem (a) pero con V (t) = V0 sen ω0 t.

4. Considere el siguiente circuito RC:


R C

q(t)

v(t)

Figura 2:

a) Suponiendo V (t) = V0 y q(0) = q0 , calcular q(t) e i(t) para t > 0. ¿Qué sucede con
q(t) e i(t) a medida que t aumenta?.
b) Idem (a) pero con V (t) = V0 sen ω0 t.

5. La ecuación y ′ + α(x)y = β(x)y k con k ∈ R, k 6= 0, 1, se denomina ecuación de Bernoulli.

a) Demostrar que mediante el cambio de variable z = y 1−k , la ecuación se transforma


en la ecuación lineal
z ′ + (1 − k)α(x)z = (1 − k)β(x).
b) Resolver las ecuaciones:
a) xy ′ + y = x4 y 3 b) xy 2 y ′ + y 3 = x cos x.

6. Si y = y(t) denota el número de habitantes de una población en función del tiempo, se


denomina tasa de crecimiento de la población a la función y ′ /y.
a) Graficar y(t) en el caso en que la tasa de crecimiento sea constante.
b) Supongamos que una población tiene tasa de crecimiento constante. En un instante
determinado la cantidad de individuos es de 1000 mientras que seis meses después es de
1010. ¿Cuántos individuos habrá después de 10 años?.
c) Estudiar las poblaciones cuyas tasas de crecimiento son de la forma r − cy con r > 0,
c > 0 e y(0) = y0 > 0. Calcular lı́mt→+∞ y(t).

3
7. Considere el siguiente sistema mecánico compuesto por un cuerpo de masa m sometido a
una fuerza f (t) y un resorte cuya constante elástica es k. Denomine x(t) al desplazamiento
m
k
f(t)

0 x(t) x

Figura 3:

de la masa respecto de la posición de equilibrio en el instante t. Suponiendo f (t) ≡ 0,


demuestre lo siguiente: p
(i) x(t) = A cos ω0 t + B sen ω0 t, con ω0 = k/m, A = x(0) y B = x′ (0)/ω0 . Con lo cual el
cuerpo oscila con una frecuencia
√ de ω0 /2π ciclos por segundo.
(ii) Si se considera C = A2 + B 2 y ϕ = tan−1 (A/B), entonces x(t) = C sen(ω0 t + ϕ).
Por lo tanto C es la amplitud de la oscilación y ϕ es el ángulo de fase.

8. Considere un sistema mecánico como el del problema anterior (con f (t) ≡ 0) pero ahora
sumergido en un medio viscoso. Entonces, además de la fuerza elástica producto de la
interacción del resorte con el cuerpo, deberá considerarse una fuerza de rozamiento F ,
producto de la interacción del cuerpo con el medio, que puede suponerse proporcional a
la velocidad del cuerpo pero de sentido contrario a ésta, en otras palabras, F (t) = −cx′ (t)
con c una constante positiva (c es la constante de amortiguación).
Demuestre lo siguiente: √
(i) Si c2 > 4mk, x(t) = c1 e−(α−β)t + c2 e−(α+β)t , con α = c/2m, β = c2 − 4mk/2m,
(observe que 0 < α < β ). Este caso se denomina sobreamortiguado debido a que no hay
oscilaciones.
(ii) Si c2 = 4mk, x(t) = (c1 + c2 t)e−αt . Este caso se denomina de√amortiguamiento crı́tico.
(iii) Si c2 < 4mk, x(t) = e−αt (A cos√ ω ∗ t + B sen ω ∗ t) con ω ∗ = 4mk − c2 /2m. También
x(t) = C sen(ω ∗ t+ϕ)e−αt con C = A2 + B 2 y ϕ = tan−1 (A/B). En este caso se producen
oscilaciones amortiguadas de frecuencia ω ∗ /2π, y se denomina subamortiguado.
9. Considere el sistema masa-resorte del problema 7, suponiendo que sobre el cuerpo se aplica
una fuerza f (t) = F0 sen ωt. Demuestre lo siguiente:

a) Si ω 2 6= ω02 = k/m,

x(t) = A(ω) sen ωt + c1 sen ω0 t + c2 cos ω0 t

con A(ω) = F0 [m(ω02 − ω 2 )]−1 y c1 y c2 constantes arbitrarias.


b) Si ω 2 = ω02 , entonces
F0
x(t) = − t cos(ω0 t) + c1 sen ω0 t + c2 cos ω0 t
2mω0
con c1 y c2 constantes arbitrarias. Observe que en este caso la amplitud de la oscilación
no es acotada (resonancia).

4
10. Considere el sistema masa-resorte amortiguado del problema 8 y suponga que sobre el
cuerpo se aplica una fuerza f (t) = F0 sen ωt.

a) Demuestre que
x(t) = A(ω) sen(ωt + ϕ) + xh (t)
 
con A(ω) = F0 / m2 (ω02 − ω 2 )2 + ω 2 c2 , ϕ = tan−1 m(ωωc
p
2 −ω 2 ) y xh (t) una solución
0
arbitraria de la ecuación homogénea mx′′ + cx′ + kx = 0.
b) Demostrar que lı́mt→+∞ [x(t) − A(ω) sen(ωt + ϕ) = 0], con lo cual, para t >> 1,
x(t) ≈ A(ω) sen(ωt + ϕ), independientemente de las condiciones iniciales.
c) Hallar el valor de ω que maximiza la amplitud A(ω).
d ) Encontrar el potencial electroestático v(r) entre dos esferas concéntricas de radios
r1 = 4 cm y r2 = 8 cm, sabiendo que el potencial de la esfera interna es de 110 volts
y el de la externa es de 0 volts. (Sugerencia: v(r) satisface la ecuación diferencial
rv ′′ + 2v ′ = 0)

11. La ecuación x2 y ′′ + axy ′ + by = 0 con a y b constantes y x > 0 (x < 0), se denomina


ecuación de Euler-Cauchy.

a) Demostrar que mediante el cambio de variable x = et (x = −et ), la ecuación se


transforma en la ecuación lineal a coeficientes constantes
d2 y dy
2
+ (a − 1) + by = 0.
dt dt

b) Resolver en (0, +∞) las siguientes ecuaciones de Euler-Cauchy:


a) x2 y ′′ + 2xy ′ − 6y = 0 b) x2 y ′′ + 3xy ′ + y = 0 c) x2 y ′′ + xy ′ + y = 0
d) x2 y ′′ − 2xy ′ + 2y = 0

5
Ejercicios adicionales a las guı́as 1 a 4

Producto interno
1. Se ha definido en |R2 un producto interno tal que (v1 , v1 ) = 1, (v1 , v2 ) = 0 y (v2 , v2 ) = 2, con
v1 = [1 1]T y v2 = [1 2]T . Se pide:
(a) hallar (x, y) con x = [x1 x2 ]T , y = [y1 y2 ]T .
(b) Descomponer v = [2 2]T como suma de un vector paralelo a u = [1 0]T y otro ortogonal
(según el p.i. definido) a u.
, e3 } la base canónica de |R3 y sea (·, ·) un producto interno en |R3 tal que ke1 k = ke2 k =
2. Sea {e1 , e2√
1, ke3 k = 2, (e1 , e2 ) = 0 y u = 2e2 − e3 es ortogonal a e1 y a e3 . Halle (x, y) para x, y ∈ |R3 y
calcule la proyección ortogonal de v = [1 1 2] sobre el subespacio S = {x ∈ |R3 : x1 = 0}.
3. Sea B = {v1 ; v2 ; v3 } una base de un |R-espacio vectorial V . Hallar los valores de α para los cuales
existe un producto interno en V que verifica:

kv1 k = kv2 k = kv3 k = 2, (v1 , v3 ) = 0, (v1 + v2 , v1 + v3 ) = 4 + α y (v1 − v2 , v1 − v3 ) = 4 − α.

Considere α = 1 y halle una base ortogonal de S ⊥ siendo S = gen{v1 + v3 }.


4. Demuestre que (p, q) = p(0)q(0) + p(1)q(1) + p(−1)q(−1) es producto interno en P2 pero no en
P3 . Halle los valores de α y β para que p = αt + β(t2 + 1) se encuentre lo más cerca posible de
q = t2 .
5. Demostrar que para α > 1, (x, y) = x1 y1 + x1 y2 + x2 y1 + αx2 y2 es producto interno en |R2 . ¿Cúal
propiedad de producto interno no se cumple si α = 1?.
Considere α = 2 y halle los v de |R2 tales que su proyección ortogonal a S = {x ∈ |R2 : x1 −x2 = 0}
sea u = [1 1]T y d(v, S) = 1.
6. Sea {e1 , e2 } la base canónica de |R2 . Demuestre que no existe un producto interno en |R2 tal que
ke1 k = 1, ke2 k = 2 y (e1 , e2 ) = 2.
7. Considere en |R2×2 el producto interno (M, N ) = m11 n11 +m12 n12 +m21 n21 +m22 n22 . Demuestre
que si S = {A ∈ |R2×2 : A = AT } entonces S ⊥ = {A ∈ |R2×2 : AT = −A}.(Sugerencia: halle
primero una base de S).

8. Considere en C([−π, π]) el producto interno (f, g) = −π f g dx. Hallar los valores de α, β y γ
que minimizan el valor de la integral
Z π
[|x| − α − β cos x − γ sen x]2 dx.
−π

9. Sabiendo que u = [2 2 2]T es el elemento del subespacio S ⊂ |R3 más cercano a v = [1 2 3]T .
a) Hallar los posibles subespacios S.
b) Hallar una base ortonormal de S sabiendo que dim(S) = 2.
Matrices de proyección, QR, cuadrados mı́nimos
 
1/3 1/3 1/3
 
10. Comprobar que P =  1/3 5/6 −1/6  es una matriz de proyección y hallar Q ∈ |R3×k , con
1/3 −1/6 5/6
k a determinar, tal que las columnas de Q sean ortonormales y P = QQT .

1
11. Dadas A ∈ |Rn×m y B ∈ |Rn×k demostrar que col(A) ⊥ col(B) si y sólo si AT B = 0. (Nota:
considere el p.i. canónico de |Rn ).

12. Encuentre A ∈ |R3×3


" √ tal que [1 − 1 0]A = 0 y A admita una descomposición QR normalizada
√ √ #
2 2 √2 2√2
A = QR con R = . ¿Es única?.
0 3 2 3
Para la matriz A hallada, encuentre la matriz de proyección sobre (col(A))⊥ .

13. Sea A ∈ |Rn×m una matriz de rango m y A = QR una descomposición QR normalizada de A.


¿Qué dimensión y qué rango tiene R?. Deducir que AT A = RT R y que AT A es inversible.
 
1 −1 " √ √ #
√ √ √
  2 2√2 √ 0
14. Sabiendo que A =  0 1  y b = [ 2 3 2]T , hallar la matriz de
0 3 − 3
1 1
proyección a col(A) y todos los x ∈ |R3 que minimicen kAx − bk.
 
1 1 2
 
15. Dada la matriz A =  1 r 1 + r , hallar los valores de r para que A admita una descom-
1 r 1+r
posición QR normalizada tal que rango(QQT ) = 1. Encontrar la matriz de proyección a col(A)
para cada valor de r hallado.

16. Sea P ∈ |Rn×n una matriz de proyección. Demuestre que si P = QR es una descomposición QR
normalizada de P entonces Q = RT .

17. Sean P1 ∈ |Rn×n y P2 ∈ |Rn×n , matrices de proyección. Demostrar que si P1 P2 = 0, entonces


P2 P1 = 0 y P1 + P2 es matriz de proyección. Hallar la relación entre los rangos de P1 , P2 y
P1 + P2 .

18. Dada A ∈ |Rn×m , demuestre que Nul(AT ) = col(A)⊥ y de allı́ deduzca que Nul(A)⊥ = col(AT ).
(Considere el p.i. canónico.)

19. Suponga que A ∈ |Rn×m (m ≥ n) y que Nul(A) tiene dimensión m−n. Demuestre que si A = QR
es una descomposición QR normalizada entonces Q es inversible.

20. Sean P1 ∈ |Rn×n y P2 ∈ |Rn×n matrices de proyección. Demostrar que col(P1 ) ⊆ col(P2 ) si y sólo
si P2 P1 = P1 .

21. Suponiendo que A ∈ |Rn×m tiene rango m y b ∈ |Rn , explicar por qué x̂ = (AT A)−1 AT b minimiza
kAx − bk.

22. Determine, justificando su respuesta, si las siguientes afirmaciones son verdaderas o falsas:
(a) Si x̂ es una solución por cuadrados mı́nimos de la ecuación Ax = b entonces kAx̂k ≤ kbk.
(b) x̂ = 0 es una solución por cuadrados mı́nimos de la ecuación Ax = b si y sólo si b es ortogonal
a col(A).

23. Explicar, sin resolver la ecuación, por T


  qué u = [1 0] no puede ser la solución por cuadrados
1 1
 
mı́nimos de Ax = b con A =  1 0  y b = [1 1 0]T .
2 0

2
 
α 0
 
24. Sabiendo que A ∈ |R3×2 admite una descomposición QR normalizada A = QR con Q =  β 1 
γ 0
y que para cierto b ∈ |R3 la solución x̂ por cuadrados mı́nimos de la ecuación Ax = b verifica
b − Ax̂ = [1 0 − 1]T , hallar los posibles valores de α, β y γ.
 √ √ 
1/ 2 1/√3
 
25. Sabiendo que A ∈ |R3×3 admite una descomposición QR normalizada con Q =  √0 1/√3 ,
1/ 2 −1/ 3
T
hallar bases ortonormales de col(A) y Nul(A ) y las matrices de proyección a esos subespacios.
Transformaciones lineales
26. Defina una transformación lineal T : |R3 → |R3 , tal que T (x) = 2x si x ∈ S = {x ∈ |R3 : x1 − x3 =
0} y d(x, S) = d(T (x), S) para todo x ∈ |R3 . Halle la representación matricial de T en la base
canónica de |R3 .
 
1 1 1
 
27. Sea T ∈ L(P2 ) tal que [T ]BC =  1 −1 3  con B = {1; 1 + t; t + t2 } y C = {1; t − t2 ; t2 }.
2 1 3
(a) Hallar bases de Nu(T ) e Im(T ) y determinar si existe p ∈ P2 tal que T (p) = 2
t . 
1 0 0
 
(b) Determinar, justificando, si existen bases B 0 y C 0 de P2 tales que [T ]B 0 C 0 =  1 1 2 . En
1 1 3
caso de existir hallar tales bases.

28. Sea V = {f : |R → |R : f = c1 f1 + c2 f2 + c3 f3 } con f1 (t) = 1 + t + 2t2 , f2 (t) = 1 + αt + 2t2 y


f3 (t) = 1 + 2t + αt2 . Se pide:
(a) Determinar todos los valores de α para los que está bien definida T ∈ L(V, |R 3 ) tal que
T (f1 ) = [1 1 1]T , T (f2 ) = [1 − 1 1]T , T (f3 ) = [2 0 2]T . Para esos valores de α hallar bases de
Nu(T ) e Im(T ).
(b) Para la transformación lineal T de (a) con  α = 3, decidir para qué valores de λ existirán
1 3 0
 
bases B de V y B 0 de |R3 tales que [T ]BB 0 =  1 2 0 . En caso de existir halle tales
1 λ+2 λ−1
bases.
 
1 4 0
| 3 , P2 ) tal que [T ]EB
 
29. Sea T ∈ L(R =  −2 1 3  con E la base canónica de |R 3 y
λ2 4λ + 3 1
B = {1; t + 1; t2 + t}. Se pide:
i) Hallar los valores de λ para que T sea inversible.
ii) Para λ = −1, justificar que T es biyectiva, explicar cómo se obtiene [T −1 ]E 0 E a partir de
[T ]EB (con E 0 la base canónica de P2 ) y calcular T −1 (p) para p = t2 + 4t + 3.
iii) Hallar los valores de λ para que existan al menos dos x ∈ |R3 distintos tales que T (x) =
t2 − t − 1.

30. Sean B = {v1 ; v2 ; v3 } y C = {w1 ; w2 ; w3 } bases de los espacios vectoriales V y W respectiva-


mente.
(a) Justificar la existencia de una única transformación lineal T : V → W que verifica T (2v1 +
v2 ) = w1 − w2 + w3 , T (v1 − v2 ) = w1 + 2w2 + w3 y T (v1 + v3 ) = 2w1 + w2 + 2w3 y encontrar

3
bases de Nu(T ) y de Im(T ).
(b) Encontrar bases D de V y E de W tales que [T ]DE tenga tantas columnas y filas nulas como
sea posible.

31. Sea T ∈ L(V, W ) y (·, ·)W un producto interno en W . Demostrar que (x, y)V = (T (x), T (y))W ,
es un producto interno en V si y sólo si T es inyectiva.
 
1 −1
 
32. Demostrar que (x, y) = xT AT Ay con A =  0 2  es producto interno en |R2 . (Sugerencia:
1 2
use el ejercicio anterior).

33. Sea T ∈ L(P2 , |R"2×2 ) definida


# por T (a0 + a1 t + a2 t2 ) = a0 B + a1 BA + a2 BA2 con B ∈ |R2×2
1 α
inversible y A = .
α 1
(a) Hallar los valores de α para los cuales dim(Nu(T )) = 1.
(b) Considerando B = I, α = −1" y el producto
# interno en |R2×2 , (M, N ) = m11 n11 + m12 n12 +
2 −2
m21 n21 + m22 n22 , expresar C = como C = C1 + C2 , con C1 ∈ Im(T ) y C2 ⊥ Im(T ).
0 0

34. Sabiendo que T ∈ L(R | 3 ) es la reflexión respecto de cierto plano S y que T ([1 1 1]T ) =
T
[−1 − 1 − 1] se pide:
(a) Determinar el plano S y hallar la representación matricial de T en una base ortogonal
B = {v1 ; v2 ; v3 } tal que v1 = [1 1 1]T .
(b) Demostrar que T n = Id si n es par y T n = T si n es impar.

35. Sean V = {f : |R → |R : f (x) = (a0 + a1 x + a2 x2 )ex } y T ∈ L(V, P2 ) definida por T (f ) =


(f 0 − f )e−x .
(a) Demuestre que dim(V ) = 3 y halle bases de Nu(T ) e Im(T ).
(b) Halle los f ∈RV que minimizan kp − T (f )k con p(x) = x2 + x, considerando en P2 el producto
1
interno (p, g) = −1 p(x)g(x)dx.

36. Definir T : |R3 → |R3 de modo tal que T (S) ⊂ S siendo

S = gen{[1 − 1 1]T ; [1 0 − 1]T },

PS ⊥ (x) = PS ⊥ (T (x)) y dim(Nu(T )) = 1. Encontrar [T 2 ]E con E la base canónica de |R3 .


 
2 −3
 
37. Considere la transformación lineal T : |R2 → |R3 , T (x) = Ax con A =  1 0 . Hallar todos
0 2
los b ∈ R que cumplan simultáneamente: i) kbk = 5, ii) d(T (x), b) es mı́nima para x̂ = [2 1]T .
| 3

(Considere el p.i. canónico.)

38. Sea T ∈ L(R | 3 , V ) tal que para la base canónica E de |R 3 y B = {v1 ; v2 ; v3 } una base de V ,
[T ]EB = A es triangular superior y a11 = −1, a12 = 1, a13 = 2, a22 = 2, a23 = 0 y a33 = −1.
Justificar la existencia de T −1 y demostrar que (x, y) = (T −1 (x))T T −1 (y) es un producto interno
en V . Hallar la proyección ortogonal de v1 + v2 + v3 sobre gen{v1 + v2 }.

4
  
    ! "  
 #
 $
    #  % 
&
'()+*,
-..
+ * /  0 1 
 %! 32   * 54  76  8
'9* 
:
*   %

;=<.> ?A@'BAC?EDGFHCIKJ=L+MNBODGJQPRMNIKJ=FSC'LTUMNBODGJQPNC@'DSJ=FSC'LWVACX@MNVYMZBO?YM[VOC\I]MNL)LS^K_=BA^]C'?EDGCL)`9MRDGFH^]@'CLa
@'J=?ALS^KVACFSMN?AVAJ+bAFH^]`+CFSJdceBAC$C'If@BAC'FSbgJ9VAC$C'LS@ MRIhMNFHCL.CL8ijkTlI]BOC_=J)ceBACCL%m j <
p p p x x
^hno ru;qtp r.stv ^]^hn$o ;; r1r1;;Wv ^]^K^wno r1x; ; v ^KPOnfyz{ x xx x;  POn yzzz{ x ; ;qxp r8sx ~| }
x x r1; r1;W|~} xx xx rut t  }}
p€<.C'`BACL‚DGFHCIKJdLS^K_=BA^]C'?EDGCNƒ
^hn…„†VAC$‡Zˆ‰‡[C'L LS^]?O_=BAIhMRFŠLS^‹TlLAJ=Œ I]J)LH^Žl x CL.MNBODSJqPRMNIKJ=F<
^K^wnu‘^ FGMN?A_NJY’“„n•”—–˜‡5a5— x C'L$BA?—MRBODGJQPRMNI]JNFVACd„™VAC)`BAIšDG^]bOI]^]@'^]VYMRV‰_=C'J=`›C'Œ DGFH^]@M
‡lrU”œ<
O<8OMRžA^]C'?AVAJ)ceBAC Ÿ¡;¢r›p xR£¥¤ CL.BA?\MNBODGJQPNC@'DSJ=F.VAC
p p r8§
„¦ yz{ p r; r8 d¨
rup r; ;9|~}
C'?A@BAC'?EDGFSCuI]J=L…MNBODSJqPRMNIKJ=FSC'LVACu„1a€LHBAL©`BOIKDG^KbAI]^K@^KVYMNVAC'L MNI]_NCžAFSMN^]@MNL T)_=CJ=`ªC'Œ DSFS^K@ MNLTdD«MN?‘¬
DSJ=L.MNBODGJQPNC@'DSJ=FSC'L8IK^]?ACMNI]`+C?EDSC^]?AVOCbgC?AVA^KC?EDGC'L @J=`+J)LSC M)bgJ=LS^KžAI]CR<
sA<.> ?A@'BAC?EDGFHCZIKJ=LlMNB€DGJQPRMNI]J=FHCL VOC­„auLHBALX`BAIšDG^]bOI]^]@'^]VYMRVACL+MNIK_=CžAFSMN^]@MNLWT®_=CJ=`ªC'Œ DSFS^K@ MNLWT
DGMN?EDGJ=L.MNBODSJQP=C@¯DGJ=FHCL.I]^K?AC MNIK`9C'?EDGC^K?AVACbgC?OVA^]C'?eDSCL @JN`9J)LSCM+bgJ=LH^]žAIKCNaALH^]C?OVAJ
; p
„°±yz{ r²;
; r²; x§ d|~} ³
´€<.C'`BACL‚DGFHC1ceBAC1I]J=L MNB€DGJQPRMNI]J=FHCL VACBA?YM+`WMRDSFS^Kµ¶·DGFH^hMN?A_NBAIhMNF%’“^]?€¸¹CFS^KJ=F J+LSBAbgCFH^]J=FGn LHJ=?
IKJ=LCI]C'`9C'?EDGJ=L…VAC8I]MVO^hMN_=JN?YMNIº<5> ?A@BOC?EDGFHC%I]J=L7MNBODSJQP=C@¯DGJ=FHCL MRLSJ‘@^hMRVAJ=L…M1@ MRVYMMNB€DGJQPRMNI]J=F
bYMRFGMdCIf@MNLSJ+C?\ceBAC¶g»¼»…¾½ ¶O¿º¿8LS^fÀ.3½ ÁY<
§O<8€BObœJ=?O_EMdc‘BOC1„ÂVAC$‡[ˆ‡[MNVA`+^KDSCIhMdbYMRFHDG^K@^ J=Œ ?lC?\žAI]J‘ceBACL
„¦"o „%ÃÄÃń%„uÆÄúÆÆv
x
@'J=?‰„ÃÄà VOC”lˆÇ”œaY„úÆ8VOC”lˆ›’¹‡lrU”An…Tl„uÆÄÆ8VAC9’“‡lrU”AnŠˆ›’¹‡lr›”AnÈ<
;
ɒ M=nXC'`BAC'LHDGFHC©ceBACŠC'IebœJ=IK^]?AJN`9^KJŠ@ MNFSMN@'DSCFʌ LHDS^]@'JuVAC…„3C'LCI‘bAFHJ€VABO@'DGJ8VAC©IKJ=LËbœJNI]^]?OJ=`9^KJ=L
@MNFGMR@'DGC'FS^KLHDG^K@J=L VAC$„ÃÄà Tl„8ÆÄÆQ<
’“žYnXC'`BAC'LHDGFHC1ceBAC1C'L8MRBODGJQPRMNI]JNFŠVAC$„†LS^‹TlLAJ=Œ I]J)LS^Ž\CL.MNBODSJqPRMNIKJ=F VAC$„%ÃÄÃ\J)Œ „uÆÄÆQ<
’“@ n‰‘^Ž„ÃºÆ  x+Ì ceB…C$Œ FHCIhMR@^QJNŒ ?\bOBACVACCL‚D«MNžOI]C@'CF.C?EDGFHC$I]J=L.MNBODSJQP=C@¯DGJ=FHCL.VAC„%ÃÄÊTX„uÆÄÆ
T I]J=L MNB€DGJQP=C'@'DGJNFSCL8VAC$„%Í
<.> ?A@'BAC?EDGFHCZIKJ=LlMNB€DGJQPRMNI]J=FHCL VOC­„auLHBALX`BAIšDG^]bOI]^]@'^]VYMRVACL+MNIK_=CžAFSMN^]@MNLWT®_=CJ=`ªC'Œ DSFS^K@ MNLWT
² DGMN?EDGJ=L.MNBODSJQP=C@¯DGJ=FHCL.I]^K?AC MNIK`9C'?EDGC^K?AVACbgC?OVA^]C'?eDSCL @JN`9J)LSCM+bgJ=LH^]žAIKCNaALH^]C?OVAJ
p p r8§ x x
yzzz p r; r8 x x
„¦ zz rup r; ; x x | }}} ³
z{ x x x ; r;  }}
x x x ; r; }
tO<8Î%C?AC'FGMNIK^]µ'CCIfCÄÏHCFS@'^]@'^]J+§)MNIf@ MNLHJ9C'?‰ceBAC
„ÃÄÃ"„%ÃºÆ ÑQÑQÑ·„%ÃÉÒ
„Џ yzzzz{ x << „uÆÄÆ << ÑQ< ÑQ< < Ñ·„uÆÓÒ << |~}} ¨
}
x x ³ ³ ³ „uÔGÔ }
@'J=?‰„ÃÄà ¨ ³ ³ ³ ¨ „8ÔGÔ%`WMÕDGFS^K@C'L @BYMNVOFGMNVYMRL<
ÖO<.C'`BACL‚DGFHCceBAC%IhM`BAIKDS^]bAIK^]@'^]VYMNV)_=C'J=`›C'Œ DGFH^]@M1×­VAC'I‹MNBODSJQPNMRI]J=F©\VACu„·CLŠ`+C?OJ=F©Jd^K_=BYMNI
ceBAC$IhMd`BOIKDG^KbAI]^K@^KVYMNV9MNI]_=C'žAFGMR^]@ MØÙVACIf`+^]LH`9JMNBODGJQPRMNIKJ=FaAVOCI]MLS^K_=BA^]C'?EDGC¸¹J=FH`WMOƒ
’ÉM=nXC'`BAC'LHDGFHC©ceBAC…C¯Ú€^]L‚DGC©BO?YM8`9MRDGFH^]µ ?AJ8LS^K?A_=BAI]MNFŽÛ®D«MNIeceBAC©LHBALbOFS^]`+CFSMNLË×9@'J=I]BO`9?YMRL
LHJ=?\BA?YM)žYMNLSC$bYMRFGM)CIËMNBODSJ€C'LSbYMR@^]J+MNLHJ€@'^hMNVOJ)MNIfMNà BODSJQPNMRI]J=F.f<
’“žYnXC'`BAC'LHDGFHC1ceBAC$bYMRFGMD«MNIËÛ)aAI]M`WMRDSFS^Kµ„8ÜO°ÛÝ „uÛÞCL VAC$IhM¸¹J=FH`WM
„ Ü  o gßÈà á v ³
x â
’“@ n\C'`BAC'LHDGFHCdceBACdCI5bœJ=IK^]?AJN`9^KJd@ MNFSMN@'DSCFʌ LHDS^]@JWVAC„Þ@'J=^]?A@'^]VAC1@J=?­C'IbgJ=I]^K?AJ=`+^]J)@ Mq¬
FSMN@'DSCFʌ LHDS^]@'J+VOC„.Üw<
’“VYnXC'VABAµ'@ M+MbYMNFHDS^]F…VACI‹bOBAàN?Eæ DGJ+MN?EDGC'FS^]JNFŠceBAC%CI‹bgJ=IK^]?AJ=`+^]J@ MRFGMN@¯DGCFʌ L‚DG^]@'JVAC%„ÂC'L VAC
I]M¸¹J=FS`9Mu㎒¹äHnå’¹ä#rªœn ’¹äHn…TlceBACbgJ=F I]JDGMN?EDGJ+×[çèØ­<
; x <.> ?A@'BAC?EDGFHClbYMNFSM@ MNVAM­”3é†m j IKJ=L1MNBODGJQPRMNIKJ=FSC'LVAC9„a#LHBAL`BAIšDG^KbAI]^K@^]VAMNVACL$MRI]_=C'žAFGMN^K@ MNL
TÇ_=C'J=`›C'Œ DGFH^]@MNLT­DGMN?EDGJ=L1MNBODSJQP=C@¯DGJ=FHCLIK^]?ACMNI]`+C?EDSCd^]?AVAC'bœC'?AVA^KC?EDGC'L@J=`+J\LSC M‰bœJ=LH^]žAIKCNa
LH^]C?OVAJ p ” ;
„°±yz{ x p ;  ³
x x ;9|~}
p
;=;=< Ì$ê B MNŒ ?EDSJ=LŠMNB€DGJQPRMNI]J=FHCL…VA^K¸¹C'FSC?EDSCL©VOC@C'FSJDS^]C?OC8@'J=`9J1`kMqŒ ÚO^K`9JBA?YM1`WMRDSFS^Kµ.„¦VACu‡ ˆ9‡
LH^fFGMN?A_NJY’“„n·” –k‡fÍ
;qp€<.> ?A@'BAC?EDGFHCIKJ=L MNBODSJQPNMRI]J=FHCLŠTXMNB€DGJQP=C'@'DGJNFSCL8VAC$IhM`9MRDGFH^]µ
; r1; p
„¦ yz{ ; r1; p 
; r1; p | }
LH^]?XëYMNIKIhMNFŠC'IfbœJNI]^]?OJ=`9^KJ1@MNFGMR@'DGC'F ʌ LHDG^K@JA<
;QO<.C'`BACL‚DGFHCceBACLS^Ž\CL.MNBODSJqPRMNIKJ=F VAC$„ÂC?EDSJ=?A@C'Lƒ
’ÉM=nlìNÔ CL.MNB€DGJQPRMNI]J=F©VAC$ìR„1Ô <
’“žYn\ CL.MNBODSJQPNMRI]J=FŠVOC„ à LS^5” CL BA?\C?EDGC'FSJ9bœJ=à LH^KDS^KP=JO<
’“@ n‰‘^f„˜C'L ^]?EP=C'FSLS^KžAI]CRag Ý CL.MNB€DGJQPRMNI]J=F©VAC$„ Ý <
’“VYn\dí›ìCL.MNB€DGJQPRMNI]J=F©VAC$„®í›ìNßY<
; sA<.> ?A@'BAC?EDGFHCIKJ=L MNBODSJQPNMRI]J=FHCLŠTXMNB€DGJQP=C'@'DGJNFSCL8VAC$IhM`9MRDGFH^]µ‡[ˆ‡
;7íUì ; ; ÑQÑQÑ ;
yzzz ; ;…í›ì ; ÑQÑQÑ ;
zzz ; < ; < ;©í›< ì ÑQ< ÑQ< Ñ ; < |~}}} ³
z{ < < < < <  }}}
; ; ; ÑQÑQÑ;…íUì }
€BO_=CFHC?A@'^hMOƒ5C?A@'BAC?EDGFHC8bOFS^]`+CFHJ%I]J=L#MNBODGJQPRMNIKJ=FSC'L#TdMNBODSJQP=C@¯DGJ=FHCL7bYMNFSM$C'IY@ MNLHJ$bYMNFHDS^]@'BAIhMNF
ì$ x <
;q´€<î’ÉM=nXC'`BAC'LHDGFHCWceBAC C'IbœJ=IK^]?AJN`9^KJl@ MNFSMN@'DSCFʌ LHDS^]@J\VAC9„ï@'J=^]?A@'^]VAC+@JN?CIVAC9„ ¤ TÇceBAC
bgJ=F I]JDGMN?EDGJ)„ÂTl„ ¤ DG^KC?AC'?\IKJ=L `+^]LS`+J=L MRBODGJQPRMNI]JNFSCL'<
’“žYnXC'`BAC'LHDGFHC—ceBAC[IhMªVA^K`9C'?ALS^ J=Œ ?¾VACIMNBODSJ€C'LSbYMR@^]J3MNLHJ€@'^hMNVAJUMNIMNB€DGJQPRMNI]J=FlÂVOCZI]M
`9MRDSFS^]µ„U@'J=^]?O@^]VOC @J=?IhM VA^K`9C'?ALS^ J=Œ ?%VACIeMRBODGJ‘CLHbYMN@^KJ.MNLSJ‘@^]MNVAJ.MNI=`+^]LH`9J MNB€DGJQPRMNI]J=F
VOC\I]M­`WMRDSFS^Kµ „ ¤ aTªceBACXbœJNFdI]J­D«MN?EDSJ[I]M,`BAIšDG^]bOI]^]@'^]VYMRV—_=C'J=`›C¯Œ DGFH^]@ M,VOCU@J=`+J
MRBODGJQPRMNI]JNFuVACd„Þ@'J=^]?O@^]VOC@J=?­I]MW`BAIšDG^KbAI]^K@^]VAMNVX_=CJ=`ªC'Œ DSFS^K@ MWVAC+Ç@J=`+JlMNB€DGJQPRMNI]J=F
VOC1„ ¤ <
’“@ n Ìlð B…C+Œ FSC'IhMN@'^QJ=Œ ?ÇbABOCVAC+CL‚D«MNžAIKC@'CFSLHCWC'?eDSFSC9CIbœJNI]^]?OJ=`9^KJW@MNFGMN@¯DGC'F ʌ L‚DG^K@J\VAC9„TÇC'I
VOC1„uñ©Í=a Ì T C?EDGFHC1I]MNL VA^]`+C?OLS^]JN?ACL VAC$IKJ=L MNBODSJ€C'LSbYMN@'^]J=L VOC„ÂTlIKJ=L VAC$„ñ…Í
;Q§O<8€^…㎒¹äHn, òeÔ Ô'ä Ô í†ò‘Ô Ý Ã‚Ô ä Ô Ã Ý Ã í•ÑQÑQÑAí˜ò€Ã‚äŠí˜òeólTk„ôC'LXBA?YMª`WMÕDGFS^Kµ‰‡¾ˆk‡5a8VOC'õY?A^K`9JNL
㎒¹„%nŠ·òeÔ„ íUò‘Ô Ý ÃH„ Ý í¦ÑQÑQÑÕí3ò€ÃS„®í3òeóÈßY< C`BAC'LHDSFSCceBAC1LS^­C'LMNB€DGJQPRMNI]J=F VAC1„
C'LHDGJN?A@C'L7㎒ɍœnŠC'L8MRBODGJQPRMNI]JNFŠVAC ㎒“„n¯<
; <.æC'`BACL‚DGFHCUceBACULS^ã˒¹äHnè æ ’wäHn‚ìO’¹äHn,@J=? æ ’¹äHn‰TÂìO’¹äHnbgJ=I]^K?AJ=`+^]J=L\C?EDSJ=?A@C'L‰ãŽ’¹„%nè
² ’“„n‚ìO’“„%nÈ<

;QtO<8€^K_EMI]JNL bYMNLSJNÒeL.öRÒVOCI]MdVAC`+J=LHDSFGMNÔ @'^QJ=Œ ?lVOCIËLS^K_=BA^]C'?EDGC%FSC'LSBAIšD«MNVOJAƒ
€CMN?„éUm j T)㎒wäHn©ÂòeÔ'ä í¦ÑQÑQÑqí3òeó@'J=?òeÔl ½ x < > ?EDGJN?A@C'LbYMNFSM9@ MRVYMWMNB€DGJQPRMNI]J=F
‰VOC ã˒“„%nŠCÈÚO^KLHDSCBO?MNB€DGJQPRMNI]J=FŠ÷\VAC$„ÂD«MRI‹ceBAC.ã˒É÷Yn°fø
T1ÏHBOLHDG^šõYceBAC$IhMNL MÕõYFS`9MN@^KJ=?AC'L LSCe?Yù MRIhMNVYMRL<
C'`9J=L‚DGæ FSMN@^ J=Œ ?‹ƒ ‘BAbœJN?A_EMN`+J=L.c‘BOC),CLBA?,MNBODGJQPRMNIKJ=F VAC8㎒¹„%n T‰@'J=?ALH^]VAC'FSC`+J=L8CIŽbœJ=IK^š¬
?AJN`9^KJ ’wäHn U㎒¹äHnrªË< æ
’ÓúRnŠ> ?eDSJ=?A@'CLuI]M`WMÕDGFS^Kµ ’“„%n©C'L LS^]?O_=BAIhMRF<
û J=F©JNDSFGMbYMNFHDSCNa‘bœJNF…C'IœüŽCJ=FHC`9M¸¹BAæ ?OVYMN`+C?ED«MNIgVAC'IMNŒ IK_=CžOFGM æ ’¹äHn Ðò‘Ô=’¹ägr,÷NënýÑQÑQÑ ’wägr,÷RÔQn
LH^]C?OVAJ+÷=à ¨ ÷ÕÆ ¨ ³ æ ³ ³ ¨ ÷RÔ%I]MNL FGMOʌ @C'L VAC ’wäHn©@J=?ED«MRVYMNL.@J=?XLSB‰`BAIšDG^]bOI]^]@'^]VYMRV‹<
’ÓúRnŠ> ?eDSJ=?A@'æ CL ’“„n ¦òeÔ=’¹„¾rª÷=ÃGߑn ³ ³ ³ ’“„¾rª÷ÕÔ ß‘n¯<
’ÓúRn ê J=`+J ’“„n©CL.LS^K?A_=BAI]MNFa€bYMNFGM)MNIK_ŠBAŒ ? Àºþ aŽ’“„èr›÷Õ»¥ÿG߀n©C'L8LH^]?A_NBAIhMNF'<
’ÓúRn û J=F IKJD«MN?EDGJ9÷Õ»¥ÿ.CL BAæ?‰MNBODSJQPNMRI]J=FŠVOC„1<
’ÓúRn ê J=`+J9÷q»šÿ.C'L FGMAʌ µ%VAC ’wäHn¯aN㎒“÷Õ»šÿÈn ·XT õY?YMNIK^]µMIhMdVAC`+J=L‚DGFGMR@^QJNŒ ?‹<
;QÖO< 8MNI]IKCI]J=L MRBODGJQPRMNI]JNFSCL VOC„ 5í®pR„ Æ r›N„UíUß9LH^]C?OVAJ9„ÂI]Md`WMRDSFS^KµVACIfCÄÏHCFH@^]@'^]JdsA<
p x < Ì û MNFSM‰ceB…C9Œ PRMNIKJ=FSC'L1VOCWì­é¦ij FHCLSBOIKD«M\^]?EP=C'FSLH^]žAIKC9I]M\`9MRDGFH^]µ)„ …í¾ìR„ Æ rkß a5LS^„ C'LI]M
`9MRDGFH^]µVOCIfC ÏHC'FS@'^]@^KJ+EÍ
p€;=<8€^f„ÂC'L.DGMNI‹ceBAC„ Æ Ð„ Ì©ê B MNŒ I]C'L.LHJ=?\LSBAL.bœJNLS^]žOI]CL MRBODGJQPRMNI]JNFSCLGÍ
p=p€<.> ?A@'BAC?EDGFHCŠBA?YM.¸“MN@'DSJ=FS^Kµ MN@'^QJ=Œ ?„Џ°Û ŠÛÝ Ã @'J=? [VA^hMN_NJ=?YMNIEbYMRFGM8I]MNLŽLS^]_NBA^]C'?eDSCLŽ`WMÕDGFS^K@C'L
„1ƒ p ;
^hn…„¦"o ; p v ^]^hn…„¦¢o p; r1s; v ^]^K^wn7„Џ¢o ruN; À"r; À v <
pNO < ÄVAC'` C ÏHC'FS@'^]@^KJ+MN?EDGCFH^]J=F bgCFHJ)@J=?\IhMd`9MRDGFH^]µ„ÂVAC'IfC ÏHC'FS@^K@^KJ\;$^K^]^wnÈ<
pRsA<.C¯DGCFH`9^K?ACLH^ŽI]M`WMÕDGFS^Kµ
„¦ o p p; v
bABOCVACVA^hMR_=J=?YMNIK^]µMNFSLHCNø‘LS^fCL.MNLʌ ë MNŒ _EMRI]JAa€LS^f?AxJ+C¯Ú€bAIK^]ceBAC$bœJNF c‘B…CŒ ?OJ9C'L.bgJ=LH^]žAIKCN<
p=´€<.C¯DGCFH`9^K?ACLH^‹IhMNL©`WMRDSFS^K@CL VOC%I]à J=LŠCÄÏHCFH@^]@'^]J=L ceBACM@'J=?EDG^]?eBYMR@^QJNŒ ?\LHC%C?eBA`+CFSMN?MRVA`9^šDGC'?
BA?AMd¸“MR@'DGJNFS^]µMN@^ J=Œ ? „¦¦Û ŠÛ Ý @JN
? ®VA^hMN_NJ=?YMNIÉ< €^ýDGMNIý¸“MN@¯DGJ=FH^]µMN@^ J=Œ ? C¯Ú€^KLHDGC%ëAMNI]IKCuBA?YM€a
LH^f?AJ)C¯Ú€^]L‚DGCC¯Ú€bAI]^KceBAC%bœJNF.ceB…CRŒ <
MEnŠ€< ž n;=<~^KP @ n%;N<~P V nŠsA<
pN§O<8€CM+„ÂI]Md`WMRDSFS^KµVACbgC?OVA^]C'?eDSC1VOCIfbYMNFAMNŒ `+C'DGFHJdFSCMN I …ƒ
p lí›s ;  r ru p ­r Æ
„Џ yz{ x s$r x Æ ³
x x s%r | }
’ÉM=
n žODSC?AC'F8IKJ=LŠPRMNI]JNFSCL VOC bAMNFGMdI]JNL c‘BOC1„ÂC'L8VA^]MN_=J=?AMNI]^Kµ MNžAIKCN<
’“žYnX^]MN_=J=?AMNI]^Kµ MNF7„†bYMNFGM Ǐå;%TlbAMNFGM [°p€<
s
p < Ì  CFSVAMNVACFHJ9Jd¸“MNIKLSJeÍ
² ’ÉM=n\‘^fIhM`9MRDGFH^]µ„ÂVAC$‡­ˆ‰‡[bgJ=LHCC$‡ÇMNBODSJqPRMNIKJ=FSC'L VA^]L‚DG^K?eDSJ=L C?EDSJ=?A@C'L8„ÂC'L VA^hMN_NJ=?YMNIK^š¬
µMNžAIKCN<
’“žYn\‘^f„°¦Û©Û Ý Ã  Ý Ã @'J=?èVO^hMN_=JN?YMNIºa‘C?EDSJ=?A@C'LZÐ¶<
’“@ n‰‘^ „•C'L$VA^hMN_NJ=?YMNIK^]µ MRžAI]CC?EDGJN?A@C'L.㎒¹„%nuC'L1VA^]MN_=J=?AMNI]^Kµ MNžAIKC1@'BYMNIKc‘BO^]CFSM LSCM\C'I bœJ=IK^š¬
?OJ=`9^KJ㎒¹äHnÈ<
’“VYn\‘^‹ãŽ’“„n¯af@JN?Xã˒¹äHnuBA?[bœJNI]^]?OJ=`9^KJ9?OJ\@'J=?AL‚D«MN?EDGCRaËCL$VA^hMR_=J=?YMNIK^]µMNžAI]CC?EDSJ=?A@C'L1„C'L
VO^hMN_=JN?YMNI]^Kµ MNžOI]CN<
’“C n‰‘^ýIhM1`WMRDSFS^Kµ8‡‰ˆl‡W㎒“„n¯a€@J=?㎒wäHn…BA?lbgJ=I]^K?AJ=`+^]JAaNbœJ=LHCC‡,MNBODGJQPRMNIKJ=FSC'L©VA^š¸¹CFSC'?EDGCL'a
C'?EDGJ=?A@'CL8„˜C'L VA^hMN_NJ=?YMNIK^]µ MRžAI]CR<
pNtO<8€BObœJ=?O_EMdc‘BOC„†é  ÒEöRÒ ’ ُ·ij•JdŒ m j n©C'L.BO?YMd`WMRDSFS^KµuD«MNIýc‘BOC„ Æ Ð„·T VAC'`BACL‚DGFHC1IKJ
LH^]_=BA^KC?EDGCRƒ
’ÉM=n\‘^ ,é,@'J=IĒ¹„%n©C?EDSJ=?A@C'LuÒ „ X! Ë<
’“žYnl@'J=IĒ¹„%#n "%$.BAIĒ¹„%n&  Ò <
’“@ n\>ڀ^]L‚DGCBA?YMdžYMNLHC1VOC  @J=`+bABAC'LHD«M)bgJ=F MNBODSJqPNC@¯DGJ=FHCLuVOC„1<
Ô
’“VYnX>ڀ^]L‚DGC©Ûé  ÒeöRÒ ^K?EP=CFHLS^KžAI]CD«MNIEceBAC…„¦¦Û ŠÛ Ý Ã @'J='? U¦VO^hMN_ ’ ( ; ¨ ³ )+³* ³ ¨ ;, ¨ x ¨ ³ ³³ ¨ x n
TX”9¦FSMN?A_=JA’“„n¯<
’“C n‰‘^MNVAC`®MNŒ L$„ ¤ Ò „-T  ïij J\Œ „ñ°™„-T   m j afC?EDGJ=?O@CL1C¯Ú€^]L‚DGC)BA?YMXžYMNLHC
JNFHDGJN?AJ=FS`9MNIœVA'C  @J=`+bABAC'LHD«M)bgJ=F MNBODSJqPNC@¯DGJ=FHCLuVOC„1<
pNÖO<8€BObœJ=?O_EM+ceBAC%¶Âé .%0’ /n©
T /CL.VAC$VA^K`9C'?ALS^ J=Œ ? õY?A^šD«MO<7C'DSCFH`9^K?ACNaÕÏHBOLHDG^šõY@ MR?AVAJAaY@'B MNŒ I]C'L
VAC$I]MNL LS^K_=BA^]C'?EDGCL.MRõAFS`9MN@^KJ=?ACL©LSJ=?\@^KCFHDGMNL<
’ÉM=n\‘^œáÂT â LHJ=?WžYMRLSCL7VAC /·C?EDGJ=?O@CLŸ ¶ £21 DG^KC?AC IKJ=L `9^KLS`+J=L MNB€DGJQP=C'@'DGJNFSCL…ceBACŸ ¶ £43 <
’“žYn\‘^fáÞT â LHJ=?XžYMNLHCL VAC /C?EDSJ=?A@C'L1Ÿ ¶ 2£ 1 DS^]C?OCI]J=L©`9^KLS`+J=LŠMNB€DGJQPRMNI]J=FHCLŠceBAC+Ÿ ¶ 4£ 3 <
’“@ n\>ڀ^]L‚DGCdBA?AM žYMNLSCdVOC /ï@JN`9bABOCLHDGM bœJNF%MNBODSJqPNC@¯DGJ=FHCL$VAC¶˜LS^5TLAJ=Œ I]Jǟ ¶ £41 C'L%VA^hMÕ¬
_NJ=?YMNIK^]µ MRžAI]C8bYMNFGM)@BAMNI]ceBA^KCF žYMNLHCá•VAC /)<
’“VYn\‘^¶DS^]C?OC‡›MNBODSJqPRMNIKJ=FSC'LVA^KLHDG^K?EDGJ=LC'?EDGJ=?A@'CLC¯Ú€^]L‚DGCdBA?AMXžAMNLSC)á D«MRIceBAC‰Ÿ ¶ £21 C'L
VO^hMN_=JN?YMNIº<
 x < û MNFGM@MNVYMBA?AMVACIhMRL©LS^K_=BA^KC?EDGC'L©DGFSMN?ALH¸¹JNFS`9MN@^KJ=?ACL7I]^K?AC MRI]CL7C?O@BAC'?eDSFSC$I]JNLŠMNBODSJQPNMRI]J=FHCLa
LHBAL5`BAIKDS^]bAIK^]@^KVYMNVAC'L‹MNIK_=CžOFGMN^K@ MNL‹T%_NCJ=`›C'Œ DSFS^]@MNLfT%žYMNLHCL5bYMNFGM I]JNLËMNBODGJ‘CLHbYMN@'^]J=LŽMNLHJ€@'^hMNVAJNL
M @ MNVAM8MNB€DGJQPRMNI]J=F'<5€^eCLŽbœJ=LH^]žAIKC C?A@'BAC?EDSFSCŠBA?AM.žYMRLSC7J=FSVAC'?YMNVYM D«MNIEceBAC7IhM FHCbAFHCLSC'?ED«MN@'^QJ=Œ ?
`9MRDGFH^]@'^hMNI VAC%I]MDGFGMR?ALH¸¹J=FH`WMR@^QJNŒ ? FSC'LSbgC@'DSJ)VAC%CIKIhMLHC M)BA?YM`9MRDGFH^]µVA^]MN_=J=?AMNIº<€^ýDGMNI‹žYMNLHC
?AJ)CÈÚO^KLHDSC1CÈÚObOI]^]ceBAC%bgJ=F ceB…CRŒ <
’ÉM=n ¶·ƒ‘ij 65 ij  aA¶7’ œnŸ œÃ5í Æ  ƅ8r   œÃŽí  Æí   8r Æ œÃŽí  Æí   £ ¤ <
’“žYn ¶·ƒ:9 Æ 5 9 ÆQaA¶’“òí<;«äËí<=Èä n7¦pRò%í>=Èä‹í°’ r;Šr='n ä <
´
’“@ nl¶·ƒ?/ 5 /D«MNIfceBAC$C'?‰I]M)žAMNLSCáåA@CBNÃøDBRÆFE
Ÿ ¶ £21 ¢o r1x; ; v ¨
x
@'J=?ALH^]VAC'FGMN?AVOJªbAFS^K`9C'FSJ[c‘BOC-/"C'L BA?kC'LSbYMN@'^]J—PNC@¯DGJ=FH^hMNI.FSCMNI T®IKBAC_=JZceBAC,CL9BA?
C'LSbYMR@^]J)P=C'@'DSJ=FS^]MNIf@J=`+bAIKC ÏHJA<
’“VYn ¶·ƒ:9  5 9  aA¶’¥ã n ›ãAÜ ‘r,ãf<
O;=<8€CM$¶°ƒ?/ 5 /·BA?YM%DGFSMN?ALH¸¹JNFS`9MN@^ J=Œ ?dI]^K?AC MNIOc‘BOCNaeFSC'LSbgC@'DSJVAC.BA?YM$žYMNLHC.áG @CBNÃ'Dø BÕÆ Dø B  E
DS^]C?OC$MNLSJ‘@^]MNVYMdIhM`9MRDGFH^]µ
;©<í r
„¦ yz{ p.<í r ,rè;  ’ éÇijŠn ³
p r; x | }
’ÉM=
n žODSC?AC'F8IKJ=L MNBODSJQPNMRI]J=FHCL VAC%¶aA@JN`9bAFHJ=žYMN?OVAJ)c‘BOC1?AJ)VAC'bœC'?AVAC'?VAC …<
’“žY
n žODSC?AC'FI]JNLuMNB€DGJ‘CLSbAMN@^KJ=LuMRLSJ‘@^hMRVAJ=L8M9@ MRVYM MNBODSJQPNMRI]J=F T\CL‚DGBAVA^]MNFuLH^Ž¶†C'LuVA^]MN_=JR¬
?AMNI]^Kµ MNžAIKCN<
’“@ n ê BAMN?AVAJ)¶·LHC M)VA^hMN_NJ=?YMNIK^]µ MRžAI]C.C?A@'BAC?EDSFSCBO?YMdžYMNLSC$J=FHVAC?AMNVYM+áDGMNIfceBAC9Ÿ ¶ 2£ 1 LHC M
VO^hMN_=JN?YMNIº<
Ep€<8€BObœJ=?O_EMdc‘BOC¶°Hé .$0’ /1n¯aY@'J=H? /ÞVAC$VA^K`9C'?ALS^ J=Œ ? õY?A^šD«MOaOCL D«MNIfceBAC%¶ Æ °ßY<
’ÉM=nX> ?A@'BAC?EDSFSCIKJ=L bgJ=LS^KžAI]C'L MNBODGJQPRMNIKJ=FSC'L VAC%¶1<
’“žYnXC'`BAC'LHDGFHC9ceBAC+LS^ I5Ã$J  @CBZKé /ôƒ¶L’ B€n%M  BNE+T IËÆ1J  @CB—Oé /¶7’ B€n%!r BNEea
I56à PQIËÆXR  @ x EX8T I5é!í IËÆXS  /9<¾’ɐ€BA_=C'FSC?O@^hM€ƒ CLH@FS^KžY-M B›T
 B=é&í BRÆ+@JN<? BNà
7’ B%íª¶7’ B€nSVn UNpWT BRÆ©7’ Br¶L’ B€nSXn U=p€< n
’“@ n\C'`BAC'LHDGFHC1ceBAC%¶°CL.VA^hMN_NJ=?YMNIK^]µ MRžAI]CR<
’“VYn\‘^.VO^]`\0’ /nW¢´­TVAC'D՟ ¶ 4£ 1 ±r;‰bYMRFGMÇMNI]_NBA?YM­žYMNLHCá¢VAC /9a Ì @'B MNŒ IKCL)LSJ=?UIhMRL
bgJ=LH^]žAIKCL VA^K`9C'?ALS^KJ=?ACL VOC I5à WT IËƯÍ
=O< ê J=?OLS^]VOCFSC+IhM DSFGMN?AL‚¸¹J=FS`9MN@'^QJ=Œ ?­IK^]?ACMNI¶ ƒfij  5 ij  aËVAC¯õY?A^KVYM\VOC9I]MXLH^]_=BO^]C?EDSC+`9MN?AC'FGMOƒ
¶L’ B€n ! Br›Z YY[\Y]\ Y B9@J=
? ^låŸ¥;¢rk;›p £ <> ?A@'BAC?EDGFHCIKJ=L MNBODSJQPNMRI]J=FHCLŠ¶`< _u?YMNIK^]@'CLS^7Ÿ ¶ £41
C'L.VO^hMN_=JN?YMNI]^Kµ MNžOI]C.bYMNFGMd@'BYMNI]ceBA^KCF žYMNLHCá™VOCiaj Q<
NsA< b CLHBACIšPNM)CIýC ÏHCFH@^K@^]J+MN?EDSCFS^KJ=F @JN? ^‰éÇi6j .BA?XP=C@¯DGJ=F ?AJ+?eBAI]J+MNFHžA^KDSFGMNFH^]JA<
E´€< 8MNI]IKCdBA?YMX`WMRDSFS^Kµd„ é¾ij  ö  DGMNI ceBACNƒ I叕_=C'c? @€Ÿ¡; r°; xR£ ¤ E LSCM‰^K?ePRMNFH^hMN?EDSCdbœJ=F$„1a
 °puLSC MMNBODSJQPNMRI]J=FŽ'T I6dXLHC M$CIYMRBODGJ‘CLHbYMN@^KJ$MNLSJ‘@^]MNVAJ%M%C'LSC.MNBODSJqPRMNIKJ=FaÕTVAC¯DՒ“„n å;qp€<
=§O<8€CM /"BO?kCLHbYMN@^KJ—P=C'@'DSJ=FS^]MNI8FHC MNI VACVA^]`+C?ALH^QJ=Œ ?3[TULHC Mªá  @CB=ÃVø BRÆ Vø B  EÇBO?YM—žYMNLHC
VAC /)<U€C M­¶¢Aé .$0’ /1n1D«MNIŠceBACRƒ‰^hn€e^ BUô § BNéíÂfp BRƊíÂg´ B  C?EDSJ=?A@C'L+¶7’ BOn+T  B a^]^wn
IÙh  @CB°i é / ƒ Ÿ p ;=; r = 7’ BOnl E­C'LWCI.MNB€DGJ‘CLSbAMN@^KJMNLHJ€@'^hMNVAJZMÇBA?AJVAC\I]JNL
MNB€DGJQPRMNI]J=FHCL VAC%¶aOTl^K^]^hn7IhMDSFG² MN£ µM+1 VOCWŸ ¶ 4£ 1 x CL ^]_NBYMNIfM+´€<
€C$bA^KVACNƒ
MEn 8MNI]I]MNF©IKJ=L MNBODSJQPNMRI]J=FHCL VAC%¶<ž n uMNIKIhMNFŸ ¶ 2£ 1 <
§
 .< C'`BACL‚DGFHCceBACIhMRL `WMRDSFS^K@CL©„†Tlá•?AJ)LHJ=?‰LHC`+C ÏSMN?EDSCL.LS^
²
„¦¢o p; r1s‰; v T á叢o p; pX; v ³
=tO< ê J=`+bAFHBACžgC$c‘BOCI]MNL `9MRDGFH^]@C'L
„¦"o p; ³ ´´ p; ³ ´´Xv T áå"o r; rupp\v
³Æ ³
LHJ=?‰LHC`+C ÏSMN?EDSCLj< uMRI]I]C UéÇij ^K?ePNCFHLS^]žOI]C%D«MNIýc‘BOC„¦  ká  Ý Ã <
¯
Æ N
ö
=ÖO<.> ?A@'BAC?EDGFHC„ Ô bYMNFGM+”lé­ß lÅLH^ºƒ
MEn6_—o p; rs; v žYn6_%—o p; pX; v <
s x < 8MNI]I]MNF©BA?AM+`9MRDGFH^]µ„†éÇij  ö  DGMNI‹ceBAC„ Æ ¦á™@J=?
rup ru 
áå yz{  s ru  ³
r8 ru s |~}
Ì >L[BAŒ ?A^K@ MEÍ Æ¯öNÆ
sA;=< 8MNI]I]MNF©„†éÇij DGMNI‹ceBAC„ Æ rªN„Uí®pRßd¦áWaY@'J=?
áå"o   v ¨
TlVOC'Dq’¹„%nårup€<
sep€<.C¯DGCFH`9^K?AC$I]J=L.bgJ=LS^KžAI]C'L.PRMNIKJ=FSC'L8VAC1I]^K`nÔ m oNp­„ Ô BgagLS^KC?AVOJ „åI]M9`9MRDGFH^]µ%VAC'I5CÄÏHCFH@^]@'^]JW=Ö
ž nÈ<.’º€BO_=CFHC?A@'^hMOƒCL‚DGBAVO^]C$bAFS^K`9C'FSJdC'If@ MNLHJ9C'?‰ceBAC BWC'L8MRBODGJQP=C'@'DSJ=F8VOC„1<
sEO<.%MNVAM)I]M`WMÕDGFS^Kµ ; x
x
„¦ yz{ xà x ; 
Æ rp qÆ |~}
C'?A@J=?EDSFGMNF BA?dLHBAžgCLSbAMN@^KrJ Iª^]?EPRMNFS^]MN?EDGC…bœJ=F5„®D«MRIOceBAC I]^K`nÔ m p­„ B x bYMNFSMu@ MNVArM B sé I8<
s=sA<.C'`BACL‚DGFHC c‘BOC7bAMNFGMŠBA?AM `9Ò MRDSFS^]µ5„†é—m j ÒeöRÒ aqCIN@J=?ÕÏHBO?eDSJI3A@ft+é—m j Ò ƒ1IK^]`dÔnm oNp,„ Ô t1
E C'L#BA?dLHBAžœC'LSbYMR@^]J%VOC8m j a=^K?EPNMRFS^hMR?eDSC©bgJ=F„1< Ì ‘^€„kC'L#VA^]MN_=J=?YMRI]^]µMNžAIKC c‘B…CŠŒ LSBAžgCLHbYMN@'^]J
FHx CLSBOIKD«M IŠÍ
se´€<.C'`BACL‚DGFHC‰ceBAC‰LS^ ㎒¹äHndCL+CI bgJ=I]^K?AJ=`+^]J,@MNFGMR@'DGC'F ʌ LHDG^K@J[VAC\„ T›„!CL9VA^hMR_=J=?YMNIK^]µMNžAI]CRa
C'?eDSJ=?A@'CLf㎒“„n <Š’É> LHDSC.FHCLHBAIKDGMNVAJAa=@'J=?AJ‘@^KVAJ%@J=`+JC'IAü5C'J=FSC'`WMVOC MQT‘IKC'TE+¬ 8MN`9^KIKDSJ=?‹a
PRMNIKC%C?\_=C?AC'FGMNIÉaACx L VAC'@^KF.LH^]?X?AC'@CLH^]VYMRV‰VACLSBObœJ=?OCF ceBAC$„˜C'L VA^hMN_NJ=?Yê MNIK^]µ MRžAI]CR<¼n

²
Práctica 5 (2da Parte)- Sistemas de Ecuaciones Diferenciales

0
1. Hallar la solución general del sistema
 X = AX, conA:
· ¸ · ¸ 2 2 −6
3 1 1 −2
i) ii) iii) 2 −1 −3 
1 3 1 −2
−2 −1 1
2. Resolver los problemas a valores iniciales:
· ¸
2 −1
a) X 0 = AX, X(0) = [1 − 2]T , con A = .
−1 2
b) X 0 = AX, X(0) = [1 − 2 1]T , con A la matriz del Ejercicio 1.iii.
3. Hallar bases de soluciones reales 
de los sistemas:
· ¸ 2 1 −1
0 1
i) X 0 = X ii) X 0 =  0 0 1  X.
−1 0
0 −2 2
4. Resolver el problema a valores iniciales
· ¸
0 T 0 5
X = AX, X(0) = [1 0] , con A = .
−1 4
5. Hallar la solución general de los siguientes sistemas lineales no homogéneos:
½ 0
y1 = 2y1 − y2 + et
a)
y2 = −y1 + 2y2 − e2t
0
· ¸
0 0 −1
b) Y + AY = F (x), con A = y F (x) = [x − x]T .
−1 0
Sugerencia: haga un cambio de variables adecuado.
6. La solución general de un sistema lineal homogéneo X 0 = AX en R2 es
· ¸
c1 e−t + 5c2 e3t
X=
2c1 e−t + c2 e3t
Hallar A.
7. Considere la ecuación lineal homogénea de 2do orden:
y 00 + ay 0 + by = 0 (1)
2
y el sistema de ecuaciones en R
· ¸ · ¸
0 1 y1
Y0 = Y, con Y = . (2)
−b −a y2
Pruebe lo siguiente:
a) Si Y = [y1 y2 ]T es solución de (2) entonces y = y1 es solución de (1).
b) Si y es solución de (1) entonces Y = [y1 y2 ]T con y1 = y, y2 = y 0 , es solución de (2).
c) El polinomio caracterı́stico de A coincide con el polinomio caracterı́stico de (1).
8. Encontrar la solución X 0 = AX
 general de   con A:   
· ¸ 0 −1 0 1 −2 −2 −1 1 0
1 −4
i) ii)  1 −2 0  iii)  −2 2 3  iv)  −1 0 1 
1 −3
0 0 −1 2 −3 −4 1 0 −2

1
  
   


   ! "  
#%$'&( * ) +  ,
 $-
.  *) $/10 " ,
 $234$,  ) 5 ,$
#
  % 67  %$
8
09  4$

: ;=<?>A@CBDE<F;HGI;KJ LM;KJNBPOQBRQSTUSTU;KJVWJF>XLZYK;4[]\IB J^B TMDIGITM[H\AB_LU;4S;KDW<FRF>=R^TU;AVa`cbMd bfegR^BhAR^BJQBDW<?>iBLAhIR^;HGI\ISj<F;


TUDW<^BR^DA;S>XD ;Kk DATUS;BDlmonp;rk q m nAs
t s#u DIS;KDW<FRF>=ƒ„ R#† Dv‡ \Ak wxBR^† ;KJ yaV{zvŠŒ‹ |r}GIBiw~>XDABRF><F>XL[]\IBX@
„ ˆ ‰ ˆ ˆ y ‹
t e_€‚ … ‡ ˆ ˆ z  J^B>;XRQ<F;XŽK;KD{>XLs Ke#‘’”“x•—–A™ ˜ y z› JQB >\IDITM<F>XR^TU>As
† ‡ †
†‡‡ †ˆˆ } •cš]™ ˜
œs#%Bj<FBRQwxTMDIB%S\ >Xk LUBJ GIBiLž>XJ wx>=<FRQTUSBJ GIBiLU;XJ BŸOQBRQSTMSTU;XJ# |¢¡GIBL¤£4s¦¥Cs{§J^;KD¨;KRQ<^;KŽK;KDI>XLUBJs
© s#%Bw\IBJc<FRQBv[]\IBvLU>XJow~>=<^R^TMSBJoGIBvª;K\IJQB«I;XLUGIBR¬J^;XD~;KRc<F;KŽK;XD{>XLUBJ%`­YKBR¬BŸOQBR^STUSTM;=®¯`°G{eoGIBL
£4s±¥²s³Ke´s
§Is#%Bw\IBJc<FRQBµLU>XJ J^TMŽK\ITMBDW<FBJ#hAR^;KhITMBG{>=GIBJ GIBiLž>=J w~>=<^R^TMSBJ¬\IDITM<F>XR^TU>XJ@
`>Ke¶‘·BJ \IDITM<F>XR^TU>¸ ‘v¹ºBJ#\ADITM<F>XR^TU>¸ ‘ »9BJ\ADITM<F>XR^TU>As
`°¼{e¨½HTN‘¾|¨¿7J^;KDr\IDITZ<?>XRQTž>XJ¬BDW<F;KDASBJi‘%¿ÀBJ#\ADITM<F>XR^TU>As
 œs½œB>¨Á-@Nlm nàlm n¨LU>¢<^RF>XJcÄÅ;KR^wx>XSTÆ;Kk DÇLUTMDIB >=L,GIBÈIDITUG{>¨hÉ;XRiÁ`­ÊÉeµ1€iÊËS;KDˀ'ÌÍlm n]Î=n
;KRc<F;KŽX;KD{>XLs,%Bw\IBJc<FR^BµLU;EJQTUŽK\ATUBDW<^BX@
`>KeÏ`­Á`ÅÊÉejdQÁ`­Ê{ÐZe^e*`­Êgd^Ê{ÐZe¬h{>=RF>E<F;HGI;EÊgd^Ê{ÐÑÌÏlmonAsou D¶hI>XRQ<^TUS\ALž>XRÒ?Á`­ÊÉeÓÒ ÀÒ?Ê Ò%h{>XR^>
<^;œGA;ʶÌÏlmonœs
`°¼{e¯ÊrÔÕÊ Ð JQT|¢J{;Xk LU;J^TÁ`ÅÊÉe*ÔÕÁ`ÅÊ Ð ejs
`°S e9ÖÓ× d Ø ØØdF× nœÙ BJµ¼{>XJQBx;XRQ<F;XDI;KR^wx>XL,GAB~lmnrJ^T,|ÇJ{;Xk LU;rJ^T ÖÆÁ`°× ejdØ Ø ØdQÁ`Å× n e Ù BJµ¼{>XJQB
;XRQ<F• ;XDI;KR^wx>XLÉGIBlm*nœs •
`°G{e¨½HTÚÕBJ \IDÃJQ\I¼³BJ^hI>XSTM;xTMD]Y=>XRQTž>XDW<^B4hÉ;KR_ÁµVIBDW<F;KDISBJ Á`ÅÚ_eہÜÚÜ|¯Á`ÅÚ¬Ýge ÍÚ_Ý,s
¡As ÞPGIBwßBŸOQBR^STMSTM;Ë 9h³BR^;ÏS;XDÕq m n BDpLU\IŽK>XREGIBÃlm n |º‘(Ì3q m nWÎ=n \IDITM<F>XR^TU>BDpLU\IŽW>=REGIB
€Ì9lmÛn]Î=n;KRQ<^;KŽK;KD{>=Làs
á s½œ\AhÉ;KDAŽW>Ë[]\IB9⠁ ÖÓ× ØØ Ø × nHÙ |ºâ ™  ÖÓä Ø Ø Ø ä nHÙ JQ;KDܼ{>XJQBJ¯;XRQ<F;XDI;KR^wx>XLMBJGIB
lm#nå`°R^BJ^hs¨q m nWe´s~• %Bw\IB•JQã <FRQB[Hã \AB~Lž>¯wx>=<^R^TUæEGAB•jSã >Xw¼Iã TM;rGIBx¼I>XJ^Bxç èaé°èœêµBJ"\AD{>¨wx>=<^R^TUæ
;KRc<F;KŽX;KD{>XL#`ÅR^BJQhs¯\IDATM<?>=R^Tž>Kejs¶`ཛྷ\IŽXBR^BDISTU>A@GABw\ÛBk JQ<^R^BLM;¶hIRQTUwBRQ;Ãh{>=RF>rBL*S >XJQ;¶BDË[]\IB
⠙ BJ LU>¼I>XJ^BiS >=D ;Kk DITMS >Asoëv<FR^>>XLM<^BR^DI>=<FTZYX>"BJ#BwhILUB>XR_BL¤BPOQBRQSTUSTU;ìEGIBL¤£4s±¥²sɐKe´s
®As½œB>Á‚@Almon" lm¬n\AD{><^RF>XDAJQÄÅ;KRQw~>XSTÆ;Kk D¢LMTUDIB>XLÉ<F>XL¤[]\IBµh{>XRF>STMBRc<?>E¼{>XJQB";KRc<F;KDA;KR^wx>XLâxV
í Á îžèÏBJ ;KRQ<^;KŽK;KDI>XLàs %Bw\IBJQ<FRQBi[H\ABX@
t
` >Ke í Á îžèIïgBJ#;XRQ<F;XŽK;KD{>XLÑh{>XRF>ES\{>XLU[]\ITMBR ;X<^RF>E¼{>XJQBi;KRQ<^;KDI;KRQw~>XLÑâ Ð s
`°¼{e¯ðo>=LUBDå`°>WePñ´`°Gae¬GIBL¤BŸOQBR^STMSTM;~ Hs
ìAs#ª>XLULMBiÁòÌÏói`àlm ˆ e´V³Á-·ô õµö{Va<?>XLg[]\IBÚ́ÖÆÊåÌlm ˆ ˆ @œÊ ‰ Ê ™ ÷ Ê ˆ ùø Ù JQB >¯TUDWY=>XRQTž>XDW<FB
h³;KR_Á’|XVa>XGIBwº>Xk JV,Ò?Á`ÅÊÉeÓÒ_3Ò?Ê Òh{>XR^>E<^;œGA;ʶÌÇlm s •
t øAs#uNúœhILUTM[]\IB4hÉ;XR [H\ÛBµk LU>XJ_RQ;X<?>XSTU;KDABJ_|¯LU>XJ_JQTUwB<FRûk >XJ¬hIR^BJ^BRcY=>XDÃLM;KJr>Xk DAŽK\ILU;XJ BDW<FRQBiYXBSj<F;KRQBJ
|¢LU>XJ LU;XDIŽKTM<^\IGIBJ_GIBÇBk JQ<^;KJs
tKt s#ª>XLULMBµLM;KJ>X\A<F;ÆY=>XLM;KR^BJ|Ã>X\A<^;ÓYXBSj<F;KRQBJ4GIB"Lž>xwx>=<^R^TUæi;KRc<F;KŽX;KD{>XLgGIBLNBŸOQBR^STUSTM; t svü_>XLMS\ILMB
BL{wº;Kk GA\ILU;4GIB S>XG{>µ>X\A<F;ÆY=>XLM;KRN|EBL{hIR^;HGI\ISj<F;µTUDW<FBR^DI;iBDW<^R^B#GI;KJo>X\A<^;ÓYXBSj<F;KRQBJoS;KRQR^BJQhÉ;XDœñ
GITMBDW<FBJ#>GI;XJ>=\A<F;ÆY=>XLU;XR^BJ GATMÄÅBRQBDW<FBJs
t œs#%Bw\IBJc<FRQBÃ[]\IBÃLU;KJ>X\œ<F;ÆY=>XLU;KRQBJGIB¨\ID{>Ïw~>=<^R^TMæ¢\IDITM<F>XR^TU>J^;KDpGIB¨wº;Kk GA\ILU;Ç\IDI;Ï|ý[]\IB
>X\œ<F;ÆYKBS<F;XR^BJ#S;XR^R^BJ^h³;KDIGITMBDW<FBJv>GITMJQ<FTMDW<F;KJ >X\œ<F;ÆY=>XLU;KRQBJ JQ;KDÃ;XRQ<F;XŽK;KD{>XLMBJs
tÆ© s#%TU>XŽK;KD{>=LUTUSB¬;KRc<F;KŽX;KD{>XLMwxBD]<^B S >XGI>"\ID{>µGIB#Lž>XJ*J^TMŽK\ITMBDW<FBJÛwx>=<FRQTUSBJ*J^TMwþBjk <FR^TMS >XJVWBJoGIBSTURV
B´úAhAR^BJQBS>XG{>E\ID{>GIBiBLMLž>XJ_BDÃLU>ÄÅ;KR^wx>E€µÿ¬€4»V{S;KDÀ;KRc<F;KŽX;KD{>XL³|¢ÿ GITž>XŽX;KD{>XL@
„ ƒ t t © Œ
Š ‹ „ ƒ  © ¡ ø Œ
Š ‹ ƒ„„„  ø t ø Š ‹‹‹
Tže¨“ ©t ©t › TMT­e … t © t  TMTUTže … ‰ © ¡ ‰  © ø  TMYAe øt ø ø øt s ‰ ‰
© t t ø ø © …ø t ø 
t §Is#%TU>XŽK;KD{>=LUTUSB \IDITM<F>XR^TU>XwBDW<FB%S>XG{>\IDI>GABµLU>XJ_JQTUŽK\ITMBDW<FBJ w~> <FR^TMSBJ ªBR^wÃûk <FTMS >XJVœBJ GIBSTURV
B´úAhAR^BJQBS>XG{>E\ID{>GIBiƒ„ BLMLž>XJ_BDÃLU>ŠŒ‹ ÄÅ;KR^wx>x‘ÿ#‘v¹V{S;KDǑ¾\IDITM<F>XR^TU>"|¢ÿÜGITU>XŽK;KDI>XLà@
t  ø
Tže¨“ ø  ø¨ › TUTže … ‰ t ø 
‰ ø ø t
t  œsü¬;KwhIRQ\IB¼³B [H\AB#LU>XJ J^TMŽK\ITMBDW<FBJowx>=<^R^TUSBJ hI\ABGIB J^BRÛGATž>XŽK;XD{>XLUTMæ >XGI>XJ²\ADITM<F>XR^TU>XwxBDW<FBXVK>¬\Ik D
JQTUDrJ^BRª#BRQw¶ûk <FTUS>XJ@ ƒ„ ø t ø Š ‹
Tže¨“ tt ‰ tt › TUT­e … t ‰ ø ø  s
ø ø t
t ¡Asü_>XLMTMÈ{[]\IB4S>XG{>\ID{>EGIBiLž>=J J^TUŽX\ITUBD]<^BJ#>=È{RQw~>=STU;XDIBJ S;Kwx;YXBRQG{>XGIBRF>x;Ä°>XLMJF>A@
`>Ke¨½HT ·Ì9lmÛn]Î=nBJ#GITž>XŽX;KD{>XLMTUæ >=¼ILUB#;KRQ<^;KŽK;KDI>XLUwBDW<FBvBDW<F;KDASBJ ùBJ J^TMwþBjk <FRQTUS >œs
`°¼{e #D{>w~>=<^R^TMæ%J^TMwþBjk <FR^TMS > ùÌÇlmon]Î=n<^TUBDA B å>=\A<F;ÆY=>XLU;XR^BJ RQB >XLMBJ GITMJQ<FTMDW<F;KJs
`°S e
g>XJ~w\ILM<^TUhILMTUSTUG{>XGþ>=LUŽKB¼IRF>XTMS >9| ŽXB;KwþBk <^R^TUS>åGIB¶S >XG{>Ë>X\A<F;ÆY=>XLM;KRxGAB\ID{>Ëwx>=<^R^TUæ
JQTUwþBk <^R^TUS>S;KTMDISTMGIBDÑs
`°G{e¨½HT ·Ìåq m n]Î=nBJ#GATž>XŽK;XD{>XLUTMæ >X¼ALUB#\IDITM<F>XR^TU>XwBDW<FB%BDW<F;KDISB J ¾BJªBRQwÃûk <FTMS >As
`°B eýH T  Ì q m n]Î=nýBJ¶GITU>XŽK;KD{>=LUTUæ>X¼ILMBÃ\IDATM<?>=R^Tž>=wxBD]<^B| J^\IJÃ>X\A<^;ÆYX>=LU;KRQBJrJ^;KDÕR^B>XLUBJ
BDW<F;KDISB J ·BJ#ªBRQwÃûk <FTMS >As

Å` ÄFe¨½HTùÌËq m nWÎ=n BJ GITž>XŽX;KD{>XLMTUæ >=¼ILUB \IDITZ<?>XRQTž>XwBDW<^BBD]<^;KDISBJv<F>Xw¼ITjBk D¢BJ_GITž>=ŽK;KD{> ñ
LMTUæ>X¼ILUBv\IDITM<F>XR^TU>XwBDW<FB=s
`°ŽWe¨½HT ÌÍq m n]Î=n¢BJiGITž>XŽX;KD{>XLMTUæ >=¼ILUBµ\IDITZ<?>XRQTž>XwBDW<FBBDW<F;KDASBJµJ^\ÇTMD]YXBRQJF>AVCBDÇS >=J^;¨GIB
B´úœTUJQ<^TURVA<?>Xw¼ITjBk D¨BJ GITž>XŽX;KD{>XLMTUæ >=¼ILUB\IDITM<F>XR^TU>XwBDW<FB=s
t á s#%Bw\IBJc<FRQB4S;KD¨\IDrBŸOQBwxhILM;E[]\IBiBLhIRQ;œGA\IS<^;GABµwx>=<^R^TUSBJ ª#BRQw¶ûk <FTUS>XJ¬DA;BJ DIBSBJF>=R^Tž>Óñ
wBDW<FBi\IDI>wx>=<^R^TUæ%ªBRQwÃûk <FTMS >As
t ®As½œ\AhÉ;KDAŽW>[]\IB ùÌÇlm n]Î=nBJ J^TMwþBjk <FRQTUS >i|~[]\IBiâÀÌÇlm n]Î #s*%Bw\IBJc<FRQB4[H\ABiâµ» %â~VAâµ»Éâ
|rââµ»9JQ;KDÃJQTUwþBk <^R^TUS>XJs
t ìAs½œBiGITMSB4[]\IBµ\ID{>w~> <FR^TMæ4çÌÇlm n]Î=nEBJv>=D]<^TUJQTUwþBk <FRQTUS>J^Tgç%»¨ ‰ çs
`>Keru DIS\IBDW<^R^BLM;KJ >X\A<^;ÆYX>=LU;KRQBJ_|r>X\œ<F;ÆYKBS<F;XR^BJ#GIBµLž>wx>=<FRQTUæ4>=D]<^TUJQTUwþBk <FRQTUS>
çù “ øt øt › d
‰
|S;KwhIR^\AB¼³B#[]\IB#J^\AJÛ>=\A<F;ÆY=>XLU;XR^BJ JQ;KDTUwx>XŽKTUDI>XR^TM;KJghI\IR^;XJ*|[H\ABBJoGATž>XŽK;XD{>XLUTMæ >X¼ALUB
\ADITM<F>XR^TU>XwxBDW<FBXs
`°¼{e¨ü¬;XDIJ^TMGIBRQB Ձ  ç‚|S;XwxhIRQ\IB¼³B#[H\AB ÍBJ_ª#BRQw¶ûk <FTUS>As,uNúœhILMTU[]\IB#hÉ;KRo[]\ÛB%k >µh{>XRc<FTUR
GAB%BJc<F;Ü\Ik LM<^TUw;iJ^B%GIBGI\ISB4[H\AB4LU;KJÛ>X\A<^;ÓY=>XLM;KR^BJ¬GIB%ç·JQ;KD¢TUwx>XŽKTMD{>XR^TM;KJ hI\IRQ;KJ¬|[]\IB
çBJ GITž>XŽX;KD{>XLMTUæ >=¼ILUB#\IDITZ<?>XRQTž>XwBDW<FB=s
`°S e¨%Bw\IBJQ<FRQB"[]\IBiJQTgçÌÇlm nWÎ=nEBJ#>XDW<FTMJ^TMwþBjk <FR^TMS >BD]<^;KDISBJ ’  çBJvª#BR^wÃûk <^TUS>As
`°G{er%BGI\IæS >"GABLahI\IDW<F;>XDW<FBR^TU;XR [H\AB\IDI>"wx>=<FRQTUæ_RQB >XLa>XDW<^TUJ^TMwþBjk <FRQTUS >v<^TUBDIB#>X\A<F;ÆY=>XLM;KR^BJ
TMw~>=ŽKTUD{>=R^TU;XJ~hI\IRQ;KJ¶; D]\ILU;XJV4[]\IB>=\A<F;ÆYKBS<^;KR^BJ¶S;XR^R^BJ^h³;KDIGITMBDW<FBJ9> >X\A<F;ÆY=>XLM;KR^BJ
GATUJQ<^TUDW<F;XJ J^;KD¨;KRQ<^;KŽK;KDI>XLUBJ_|¢[]\IBiBJ#GITž>XŽX;KD{>XLMTUæ >=¼ILUB#\IDITZ<?>XRQTž>XwBDW<FB=s
`°B e¨%Bw\IBJQ<FRQB"[]\IBi\IDI>wx>=<FRQTUæ4çòÌÏlm nWÎ=n>XDW<FTUJQTUwþBk <^R^TUS>BJ JQTUDIŽK\ALž>XR_JQT ÇBJ TUwh{>XRs
XøAs#ª>XLULMB  Ìplm ˆ Î ˆ JQTUwþBk <FRQTUS>x<?>XLN[H\AB   ‰ t |  ™ À¯J^B>XDÇJ^\IJµ>X\A<^;ÆYX>=LU;KRQBJ%|¶Ú év
ŽKBDÑÖ í t"tit î » Ù s •
 t s#ª>XLULMB\ID{>¢wx>=<FRQTUæªBRQwÃûk <FTMS > 1ÌÜq m ˆ Î ˆ <?ˆ >XL²[H\ABx⠁  ˆ ‰  ™ ÷  ‰ öÃJQB >¢J^TMDIŽK\ILU>XRV
¯’"J^B>E>X\A<F;ÆY=>XLM;KRÛGA;K¼ILUB#|xÚ ¾ÖÆÊÌËq m @iÊ ‰  Ê ™¤÷ Ê ˆ ’ø Ù JQB >TUDWY=>XRQTž>XDW<FB#h³;K R "s
•
u*JÏ\Ik DITMS >  ! JQT¤DI;LU;BJ BDAS\IBD]<^R^B"GI;KJ GITMÄÅBR^BD]<^BJs
Kœs#u DIS\IBDW<FRQBLU>w~ã >=<^R^TMæ%GIBiS >XGI>\IDI>GABµLU>XJ J^TMŽK\ITMBDW<FBJ ÄÅ;XR^wx>XJ S\{>=GIR{>=k <^TUS>XJ BD¶lm ™ s
>We t ø=Ê ™ ‰ ¡=Ê Ê ™ ‰ © Ê ™™ ¼ae   Ê ™ ÷ © Ê Ê ™ S eÛÊ Ê ™ s
 © s#u DIS\IBDW• <FRQBLU>• w~>=<^R^TMæ%GIBiS >XGI>\I• DI>GABµ• LU>XJ J^TMŽK\ITMB• DW<FBJ ÄÅ;XR^wx>XJ S\{>=GIR{>=k <^TUS>XJ BD¶lm ˆ s
>We¤§XÊ Ê ™ ÷ ¡=Ê Ê ˆ ‰ ®=Ê ™ Ê ˆ ¼aeg®=Ê ™ ÷ á Ê ™™ ‰ © Ê ™ˆ ‰ ¡ Ê Ê ™ ÷ §=Ê Ê ˆ ‰  Ê ™ Ê ˆ S e¤Ê ™ ‰ Ê Ê ˆ ÷ Ê ™ˆ s
=§Isü¬LU>XJ^TZÈ{• []\IB4S >XGI• >\AD{>EGIB4Lž>XJ¬J^TUŽX\I• TUBD]<^BJ_ÄÅ;KRQw~>=J_S\{•>=GIR{>=k <^TUS>X• J BDÃlm ™ |~BÄÅBS<Ñ\Ik • Bi\IDr• S>Xw¼ITM;
GIB¨Y=>XR^TU>X¼ILUBJEÊ·(#€ "ý[H\ABÃ<FR^>XDIJcÄÅ;KR^wBÃLU>ÏÄÅ;KRQw~>ÇS\I>XGIR{> k <FTUS>åBDº\ID{>ËJQTUD <œBk RQwxTMDI;ÏGIB
hIRQ;œGA\IS<^;SRQ\Iæ >XGA;IsÇu J^SRQTU¼{>ÃLž>¶D]\IBjYX>¶ÄÅ;XR^wx>ÃS\I>XGIR{> k <FTUS>As $4RF> È{[]\IB¯LU;XJS;XD OQ\IDW<F;KJGIB
DITZYKBLàs
>WegÊ ™ ÷ t ø=Ê Ê ™H÷ Ê ™™ ¼{e © Ê ™ ‰ §XÊ Ê ™H÷ ¡=Ê ™™ SÆe²Ê ™ ÷  Ê Ê ™H÷ Ê ™™ Gae ‰ §XÊ ™ ÷ §XÊ Ê ™ ‰ © Ê ™™ s
• • • • • • • •
©
K œsü¬LU>XJ^TZÈ{[]\IB4S >XGI>\AD{>EGIB4Lž>XJ¬J^TUŽX\ITUBD]<^BJ_ÄÅ;KRQw~>=J_S\{>=GIR{>=k <^TUS>XJ BDÃlm ˆ |~BÄÅBS<Ñ\Ik Bi\IDrS>Xw¼ITM;
GIB¨Y=>XR^TU>X¼ILUBJEÊ·(#€ "ý[H\ABÃ<FR^>XDIJcÄÅ;KR^wBÃLU>ÏÄÅ;KRQw~>ÇS\I>XGIR{> k <FTUS>åBDº\ID{>ËJQTUD <œBk RQwxTMDI;ÏGIB
hIRQ;œGA\IS<^;SR^\Iæ>XGI;Isou J^SR^TM¼{>LU>EDH\ABY=>ÄÅ;XR^wx>S\{>XGAR{>=k <^TUS >œs
>We  ÓÊ ™ ™ ÷ ¡=Ê ™™™ ÷ á Ê ™ˆ™ ÷ §XÊ Ê ™ ‰ §XÊ ™ Ê ˆ
¼ae © Ê • ÷  Ê ™ ÷  Ê ˆ ÷  Ê • Ê ™N÷  Ê Ê ˆ,÷ §XÊ ™ Ê ˆ
S e_Ê Ê • ™N÷ Ê ™ Ê ˆ,÷ Ê Ê ˆ s • •
X¡As#4>XGI• >™ Lž>"ÄÅ;XR^wx>S\{>=GI• R{>=k™ <^TUS&> %@]lm ™  lm'V ™%x`ÅÊÉe* ‰)(* Ê ™ ÷ ‡* Ê Ê ™g÷ (* Ê ™™ VIGABw\IBJQ<^R^B4[]\IB
‰ Ò?Ê Ò ™ + %x`ÅÊÉe + Ò?Ê Ò ™ h{>XRF><^;œGI;EʶÌÇlm s • •
 á s#u L,JQTUŽK\ITMBDW<FBBŸOQBRQSTMSTU;¢™ w™ \IBJc<FRF>¯S;Kk w;¢SLž>=J^TMÈIS >XR%\AD{>~ÄÅ;KR^wx>~S\{>XGIRI>=k <FTMS ,> %x`ÅÊÉevòÊI-» #ÊgV
GI;XDIGI#B Ձ`°y e_ÌÇlm Î V{J^TUDrBDIS;KDW<^RF>XR#LU;KJ_>X\A<F;ÆY=>XLM;KR^BJ GI#B so%Bw;KJQ<^RF>XR@
`>Ke %BJGABÈ{/˜ DI. TMG{>EhÉ;XJ^TM<^TMY=>J^T|¯J{;Kk LM;JQT¤y ø|¢GAB< 1` 4e øAs
`°¼{e %BJGABÈ{DITMG{>EDIBŽK>=<FTZYX>EJQT|¢J{;Kk LM;EJ^T¤y •P• 0 øE|¯GIB<Æ3` 4e 0 øœs
`°S e %BJTMDIGIBjÈ{DITUGI>J^T¤|¢J{;Xk LU;EJ^T¤GAB< 1` 4e •P• øA2s 0

`½œ\IŽKBR^BDASTž>œ@ãCBDIŽK>åBDpS\IBDW<?>å[]\IB¨J^ T  2 4|  ™ JQ;KD LM;KJ>X\A<^;ÓY=>XLM;KR^BJGIB V_BD]<^;KDISBJ


<^RF>Xæ>{3` %e 5  ÷  ™ |¢GIBj< 3` %e 5   ™ e´s •
X®As#%Bw\IBJc<FRQB[]\I• BLž>ÏBjúœhIRQBJ^T ;Kk D·`ÅÊg6d "I• er Ê ¹ %#"þS;K7D % Ì q m n]Î=n ªBRQwÃûk <FTMS >ÇGIBÈ{DAB\ID
hIRQ;œGA\IS<^;TMDW<FBRQDI;BDÇq m nJ^T|¢JI;Kk LU;EJQ8T %BJ#GIBÈIDITUG{>Eh³;KJ^TZ<FTZYX>œs
XìAs#%Bj<FBRQwxTMDIB S\ >=k LUBJ*GIBLž>XJ*JQTUŽK\ITMBDW<FBJ*BjúœhIR^BJ^TM;KDIBJ¬GIB<^BR^wTUDI>XDxhAR^;HGI\IS<^;KJoTUDW<FBR^DI;KJoBD¯lm ™
;xk lm ˆ s
>WeÛÊ " Ê " Ê " © Ê " ¼ae¬Ê " Ê " Ê " © Ê "
S e á Ê • " • ÷ ‰ §X• Ê ™²" ÷ ™N÷ ™ §Xʕ ÷ " ˆ ‰ ™ §=™Ê ™ " ÷   • Ê ™ • " ÷™²÷ • §XÊ ™²ˆ ÷ " ÷ ™ ì=• Ê ÷ ˆ " ˆ s ™ ™
¥²>XRF>• LU;K•J¬[H\AB"RQ• BJ^\ALM<FBDÕ hAR^;HGI\IS<^;KJ • TUDW<FBR^DI;XJV{ŽKR^>=È{S >=R • Lž>E¼É;XLž>\IDITZ<?>XRQTž>As
© øAsü¬;KwhIˆ RQ\IB¼³B%[]\IB`ÅÊg6d "Ie ‚ Ê " ÷  Ê " ™ ‰  Ê ™ " ÷ Ê ™ " ™÷ Ê ˆ " ˆ BJ \ID¢hIR^;HGI\ISj<F;ETMD]<^BRQDI;
BDÏq m s • • • •
©At s#%Bw\IBJc<FRQBÛ[]\IBÛJ^THâ3Ì9l9m  Î=nAV=BDW<F;KDISBJ :ùÍâµ»ÉâÕBJNJQBwTUGIBjÈ{DITUGI>#h³;KJ^TZ<FTZYX>œsgu*DAS\IBD]<^R^B
LU>~S;KDAGITUSTÆ;Kk D9[]\IBGIB¼ÉBES\IwxhALUTURvâ h{>XRF>¯[]\I)B :-RQBJQ\ILM<^BEGIBÈ{DATUG{>¢h³;KJ^TZ<FTMY=>Asx1` ;<:-J^BLU>
GIBDI;KwTUD{>wx>=<^R^TUævGI B =?>A@CBòGIBµâ~s¦e
© œs#u DIS\IBDW<FRQB4BL³wº>Ók úATMwx;4|BL³wÃûk DITMwx;4GIBLž>iÄÅ;KR^wx>XJ*S\{>=GIR{>=k <^TUS>XJ¬GIBLU;KJoBŸOQBR^STUSTM;KJ¬=§"|~K œV
JQ\ OQB<^;KJ#>LU>R^BJc<FRQTUSSTÆ;Kk D¢Ê » Êr t s
©K© s#4>XGI>#LU> ÄÅ;KRQw~> S\{>XGIRI>=k <FTMS > %@Hlm ˆ  lm,CV %x`ÅÊÉe* ‰  Ê ™ ÷ ÓÊ Ê ™ ‰  Ê ™™™ ‰ t ø=™Ê ™ˆ V=GAB<F™ BR^wTUD{>=R
LM;KJvY=>XLU;XR^BJvwº> k úœTMwx;|wÃûk DITMwx;xGIDB %~`­ÊÉevJ^\ OQBj<F;¨>¢Lž>~• RQBJc<FR^TM• SSTÆ;Kk DÊ ÷ Ê ™ ÷ ì=Ê ˆ 7ìAVÑ|
LM;KJoY=>XLM;KR^BJÛGABvÊrh{>XR^>"LU;XJ¬S\I>XLUBJ¬J^Bv>XLUS>XDIæ>XD¯BJQ;KJ_B´úœ<^R^Bwx;KJsž`½œ\IŽKBR^• BDISTU>A@ S;KDAJ^TUGABR^B\ID
S>Xw¼ITM;GIB#YX>=R^Tž>=¼ILUB []\IBv<^RF>XDIJcÄÅ;KR^wBvLU>"RQBJQ<^R^TMSST ;Kk D~GI>XG{>BD¢\ID{>GIBLž>iÄÅ;KR^wx>µÊI»aÊ¢ t s¦e
© §Is#u DIS\IBDW<FRQB%BLaw ™ > k úœTUw;4|BL³w¶™ ûk DITUw;%GIBLž>4ÄÅ;KR^wx>4S\{>XGAR{>=k <^TUS E> %~`­ÊÉe* Ê ™ ÷ Ê ™™ JQ\ OQB<^;E>"LU>
RQBJQ<^R^TMSST ;Kk Dr Ê ‰  Ê Ê ™¤÷ ÓÊ ™ Í§As*ªv>XLMLUB#LU;KJoÊÃhI>XRF>LM;KJ¬S\{>XLUBJ_J^B%>XLUS•>XDIæ >"BLÑB´úH<FR^Bwx;As
• •
§
©  œs#u DIS\IBDW<FRQBLM;KJ_hI\IDW<^;KJGABµLU>S\IRcY=>
 Ê ™ ÷ ¡=Ê ™™ ‰ GF  ÓÊ Ê ™  t
w >Xk J SBR^S>XDI;KJ>XL;KRQTUŽKBDrGIBiGI;KJ ÄÅ;K• R^wx>XJ_GATMÄÅBRQBDW<FBJ• @
`>K e H¨TUDITMwxTMæ >XDIGA;¶Ò?Ê Ò%S;KD¨Ê¶J^\ OQBj<F;x>E\ID{>R^BJc<FRQTUSSTÆ;Kk D¨>XGABS\I>XG{>As
`°¼{erª>XSTMBDIGA;9\IDþS >Xw¼ATU;¶GAB¯YX>=R^Tž>=¼ILUBJ>XGABS\I>XGI;9|ËRQBJQ;KLMYHTUBDIGI;9BL¬hIRQ;K¼ILUBw~>ÃBDýLž>=J
D]\IBjY=>XJ#Y=>XRQTž>X¼ALUBJs
© ¡As#u DIS\IBDW<FRQB~\IDI>rGABJ^S;KwhÉ;KJQTUSTÆ;Kk DÇBDÇY=>XLU;XR^BJ4JQTUDIŽX\ILž>XRQBJ`° ð4½Ie%GIBS >XG{>r\ID{>¢GIBLž>=J%J^T ñ
ŽK\ATUBDW<^BJ w~> <FR^TMSBJ ƒ„ á t ŠŒ‹
>WeÓ ‰ ø ø › ¼aeÓ  t › S e“  ‰  › G{e … ø ø  BÆeÓ © ©  › s
© ø t   t
‰     ‰
© á sü¬;KDAJ^TUGABR^BiLU>E<^RF>XDAJQÄÅ;KRQw~>XSTÆ;Kk D~LUTUDAB >XLÑÁ’@Hlm ™  lm ˆ GABÈ{DITMG{>EhÉ;XR Á`­ÊÉe I ʶS;KD
ƒ„ t t ŠŒ‹
’
… ©t ©t  Ø
‰
u DIS\IBDW<FRQBGIBµBDW<FRQBµ<^;œGA;KJLM;KJ_YKBS<^;KR^BJ%GIBµDI;KR^wx> t Va\IDI;<?>XL¤[]\IB¨Ò?Á`­ÊÉeÓÒvJ^B >wº>Ók úATMw~>
|r;X<^R^;<?>XLg[]\IB¨Ò?Á`­ÊÉeÓÒvJ^B >xw¶ûk DITUwx>As &J \¬B"k RQBLU>XST ;Kk DÃBDIS\ABDW<FR^>~BDW<FRQBLM;x[]\IB";K¼A<F\œYK;x|
LM;KJ_Y=>XLU;KRQBJ JQTUDIŽK\ALž>XRQBJ GI#B  !
© ®As½œ\AhÉ;KDAŽW>[]\I B ùÌÇlmonWÎ  | Ձù‘ K ¿i»BJ \ID{>E ði½¯GI B "Ms LX\IJQ<^TMÈ{[]\IB4S>XG{>ER^BJ^hI\IBJQ<F>As
`>Ke¨½H\IhÉ;XDIŽW>[H\A B ùBJ#S\{>XGIR^>XG{>BiTUDWYKBR^J^TM¼ILUB=s*u DIS\IBDW<FR^B"\ID{> ð4½¯GI#B  šA• s
`°¼{er%Bw\IBJQ<FRQBx[]\IBJ^TNÀBJiS\{>XGIR^>XG{>AVO GIBj<Ó`14ePO³BJ"TMŽK\{>XLN>XL hIR^;HGI\ISj<F;¨GIBLU;KJ%Y=>XLM;KR^BJ
JQTUDIŽX\ILž>XRQBJ GI#B "s
`°S e¨%Bw\IBJQ<FRQB [H\ABLU>XJ,S;XLU\IwD{>XJ,GABv¿‚JQ;KD~>=\A<F;ÆYKBS<^;KR^BJÛGIQB  » º|E[]\IB Lž>XJ,S;KLU\AwxD{>=J
GAB‘ùJQ;KDÃ>X\A<F;ÆYXBS<^;KR^BJGA B  #»²s
`°G{er%Bw\IBJQ<FRQBx[]\IBLU;XJiY=>XLM;KR^BJ4J^TMDIŽK\ILU>XR^BJµDA;ÃD]\ILM;KJiGIDB 1S;KTUDISTUGIBDÇS;KDåLU;KJ%Y=>XLM;KR^BJ
JQTUDIŽX\ILž>XRQBJCDI;#DH\ALU;KJCGIB  »E|iGIBÛ>XLULàûk GIBGI\IæS>v[]\IB¬BLœR^>XDIŽK;#GIMB  |iGAB #»ES;KTMDISTMGIBDs
`°B e¨%Bw\IBJQ<FRQBv[]\IBJ^RT ÕBJ S)r|GIBÈIDITUG{>µhÉ;XJ^TM<^TMY=>AVWBD]<^;KDISBJ_LM;KJ YX>=LU;KRQBJ*J^TMDIŽK\ILU>XR^BJ
|¯LU;KJ >X\œ<F;ÆY=>XLU;KRQBJ GI#B ùS;XTUDISTUGIBDs
`ÅÄFerª>XLULMBr\ID{> ði½þGITB :ßU   » 2|pGABw\IBJQ<^R^B[]\IB=@9T­eLU;KJRF>=DIŽK;KJGIVB 2| GITB :
S;KTUDASTUGABDVaTUTž?e :7BJvTMDWYKBRQJ^TM¼ILUBµJ^T¤|rJ{;Xk LU;J^TCBLgRF>XDAŽK;xGAB ·BJ W9s4`àü¬;Kwh{>XRQB"S;KD¶BL
BPOQBRQSTUSTU;x t GIBL¤£4s¦¥Cs © ejs
`°ŽWer%Bw\IBJQ<FRQB"[]\IBµJ^Tg€òBJ#;KRQ<^;KŽK;KD{>=L#€ ùX| -1` %€òX| 4e¬<FTMBDIBDÃLM;KJ wxTMJ^w;KJ¬Y=>XLM;KR^BJ
JQTUDIŽX\ILž>XRQBJs
 
© ìAs; h{>=RQ<FTMRGIBrLž>=JEGIBJQS;KwhÉ;XJ^TUSTU;KDABJEBDþY=>XLU;XR^BJEJQTUDIŽX\ILž>XRQBJ«I>XLULU>XG{>XJBDpBL BŸOQBR^STMSTM; © ¡
BDIS\IBDW<FR^B GABJ^S;KwhÉ;KJQTUSTU;KDIBJ²BDY=>XLU;XR^BJCJ^TMDIŽK\ILU>XR^BJ²GAB¬Lž> TUDWYXBR^J^>%GIB¬LU>#wx>=<^R^TUæ*GIBLœhA\IDW<F;
S=s*|¯GIBiLž><FR^>XDIJQhI\IBJc<?>GIBiLž>wx>=<FRQTUæ%GIBL¤hI\IDW<F;xBXs
§WøAs#ª>XLULMB¨LU>åhIJQB\IGA;KTUDWYKBR^J^>GITB V S;KYD  Lž>Ïw~>=<^R^TMæ¨GABL#BŸOQBRQSTMSTU; © ¡åB=sÕ| BDAS\IBD]<^R^B9LU>
JQ;KLU\ISTÆ;Kk D hÉ;KR¢S\{>XGIR^>XGI;KJ¢wÃûk DATUw;KJxGAB9DI;KR^wx>ËwÃûk DITMw~>ÏGIB9Lž>åBS\I>XST ;Kk 7D ʁ zÓVS;KD
z  í t ‰ t î » s
§ t s#%Bw\IBJc<FRQB[]\IBEBDÇBL²S >XJQ;¢BD9[]\I&B 3Ìþlm n]Î  <FTMBDIBR^>XDIŽK; W9VLž>~hAJ^B\AGI;KTUDWYXBR^J^>¨GAB 
GIBjÈ{DITMG{>¯>~h{>=RQ<FTMRGAB\ID{,> Z¢E¿ [ºGIB ò|ÃLž>J^B\IGI;KTMD]YXBRQJF>¢GIBÈIDITUG{>xBD9BL²BŸOQBR^STMSTM;¢ t GÑs
GIBL¤£4s¦¥Cs © V{S;XTUDISTUGIBDs
§]œsü¬;KwhIRQ\IB¼³Bi[H\A B Ձù‘ K ¿ » S;KD
ƒ„„ † ™ ™ ø ‰ † ™ ™ ŠŒ‹‹ ƒ„  ø ø ŠŒ‹ ƒ„„ † ˆ ˆ † ™ ™ † ‡ ‡ ŠŒ‹‹
‘́ … † ø t † ø  d Kp … ø øœØ   ø  d ¿ù … † ˆ ˆ ø ‰ † ˆ ‡ 
™™ ø ™™ ø ø ø †ˆˆ ‰ †™™ †‡‡
BJ²\IDI>#GIBJ^S;Kwxh³;KJQTUST ;Kk DµBD"Y=>XLM;KR^BJ¤J^TMDIŽK\ILU>XR^BJCGA\B ý|KVX>#hI>XRQ<^TUR¤GIBoBLMLž>AVÆS>XLUS\ILUB*RF>=DIŽK;{1` 4e´V
LU>XJvwx>=<FRQTUSBJvGABhIR^;Æ|XˆBSSTÆ;Kk DÇ>¯S;KLP1` ;4e_|:#\ILP1` 4e |¶LU>~J^B\IGI;KTMD]YXBRQJF>¢GIDB HÃ;H;KRQBjñŸ¥CBDIRQ;KJ^B=s
u DIS\IBDW<FRQBLM;KJ¬Ê9ÌÇlm \ADITM<F>XR^TM;KJ_[]\IBiw~>ÓúATMwxTMæ >XDå]Ò Ê ÒÓs
§ © s½A>=¼ITUBDIGI;[]\IB
ƒ„ t ‰  ø ŠŒ‹ ƒ„  ø ŠŒ‹ Pt ^ F  Pt ^ F 
Ձ
… tt tt ‰ tt  … øø ø  “ tP^ F  ‰ tP^ F  › d
;K¼œ<FBDABR,\IDI>4 ð4½"GI?B ºJQTUDS>XLUS\ILž>=RNhIRQBYHTž>XwBDW<^B  |XVW>%h{>XRc<FTURCGIB_BLULU>AV=«I>XLULU>XR²\AD{>% ði½
GI B #»¶|rS >XLMS\ILU>XR Lž>=J#wx>=<^R^TUSBJ GIBµhIRQ;Ó|XBSSTÆ;Xk D>S;KL—3` ; _Ée¬|¨:#\IL—1` #»Ñe_|¢Lž>J^B\IGI;KTMDWYKBRQJF>
GIEB HÃ;H;KRQBjñŸ¥CBDIRQ;KJ^BµGI#B #»Cs
§K§Is#%BSTUGATUR_JQT¤J^;KDrYKBR^G{>XGABRF>=JLU>XJ J^TMŽK\ITUBDW<FBJ#>=ÈIR^wx>XSTM;KDIBJ
`>Ke¨½HT |Çâ J^;KDÏw~> <FR^TMSBJ%J^BwxBPO^>XDW<FBJi;KRc<F;KŽX;KD{>XLMwxBD]<^BBDW<^;KDISBEJ |9â <^TUBDIBDÇLU;XJ
wTUJQwx;XJ¬Y=>XLU;KRQBJ JQTUDIŽK\ALž>XRQBJs
`°¼{e¨½HT ·Ì9lm n|râ7ÌÏlm n<FTMBDIBDÃLM;KJ_wTUJQwx;KJ_>X\A<F;ÆY=>XLM;KR^BJVIBD]<^;KDISBJ ù|¯âÀ<^TUBDIBDrLU;XJ
wTUJQwx;XJ¬Y=>XLU;KRQBJ JQTUDIŽK\ALž>XRQBJs
`°S eýH`T ÀÌ lm n~BJ%JQTUwþBk <FRQTUS>AVÑBD]<^;KDISBJiLU;KJ#Y=>XLU;KRQBJJ^TUDAŽK\ILž>=R^BJvGIB JQ;KDÇTUŽK\{>=LUBJ>¯LU;XJ
>=\A<F;ÆY=>XLU;XR^BJ GAEB "s
`°G{e¨½H`T ÀÌ lm n BJ%JQTUwþBk <FRQTUS>AVÑBD]<^;KDISBJiLU;KJ#Y=>XLU;KRQBJJ^TUDAŽK\ILž>=R^BJvGIB JQ;KDÇTUŽK\{>=LUBJ>¯LU;XJ
w ;Kk GI\ALU;KJ GIBiLM;KJ >X\A<^;ÓY=>XLM;KR^BJ GI#B s
§] œs½œB,> 1Ìplm n¯|JQB >XbD a  V| aRc2BL,wÃûk DITMwx;|BL,wº>Ók úATMwx;Y=>XLU;KRvJ^TUDAŽK\ILž>=RvGI&B 7RQBJ^h³BSj<FT ñ
Y=>XwBDW<FB=sE%Bw\IBJc<FR^B[]\IBJ^MT ÏBJi\IDå>X\A<F;ÆY=>XLM;KRvRQB >XL GI&B ÀBD]<^;KDISB J a  + O/`O + aRcxs
`½œ\IŽKBR^BDASTž>œ@ S;KDIJQTUGIBR^BµBL¤hIRQ;œGA\IS<^;×K» ×~S;XDÃׯ\ADÃ>=\A<F;ÆYKBS<^;KR\ADITM<F>XR^TM;EGI#B s¦e
¡
  
           !#"  $#&%'    (
)*$+
,
$+-./ 0 1 
!+

2'3!4&57685:9 57;<5:=?><@*;?ACBD@'E EGFDH'=<FDA.>JI8A)E K7A.9LI8@'6MA.E NPOQ9 57BD9.=<BDASRTN R)U VRTN RLWX


Y[Z]\_^a`cb O Y[ZedSfg\_h7di`cb O YjZk\l^m^a`cb O Ynd\od 2 \ 2 `pb 3
Z 3(q685er<st=?AuBv5w9)Fv6M9.=<>?xyA)6GA)><9.Fv5z=<>?FDIT5e6GF{5z|}U~€‚N„ƒ…†‡#XRTN R~ˆ27‰9)@'><EGFD;<A.6G57><;<@zB{5eE
><@76GŠ*57E2'O Z AC‹Œ3
h 3!(57BvBv576(BD@'EŽ<=<>JIG@'E;<A*|~€‚N_ƒ‘…† ‡X’N“U ”mN ‡ ~ f ‰*s•=<A:EMA–A)><9.=?A.>JI86G57>—Š‘57˜ E+9)A.6M9 5™;<A
Y 2m2 ` b OQ9.@7><EGFD;<A.6G57><;<@–B{57E >?@'6GŠ*57ER›š<R)U)OœR›š<R ‡ VR›š<RLW3
 3!ž+A)Š=<A.EŸI86MACs•=<A™R›š?R)U V R¡š<RLW¢EG@'>£><@'6MŠ¤5eE.3
^ 3(¥¡@'Š–Ž<6M=<A.¦§A—s•=<A—B{57E™><@76GŠ*57EoRš R)U™VRš¨RLW©A.>¢…† ‡ª><@lK'A.6MFDrQ9.57>ŒBv5lFD;<A.>JIGFv;Q57;g;<A)B
ŽQ5e6857BDA.Bv@7H'6857Š–@*«yK7A 5¬A.B“A®­MA.6M9.FD9.Fv@ ^ «¯9‚°;<A.B“±3²4³3 Z °¨V¤Ž“@76¡Bv@1IT57>JI8@><@¬EG@'>™FD><;<=<9)Fv;Q57E›Ž“@76
=<>£Ž<6M@´;<=?9)I8@:FD>•IGA.6M><@<3›µ³¶ E 9.FDA.6ŸIT5¬A.EŸIT5–5erQ6GŠ*579)F‚@'˜ >A)>…†·–¸
¹?3(º´=?Ž“@'>?HJ51s•=<A–RPšeR A)E¡=<>Q5C><@'6MŠ¤5»A.>™…†&· V:s•=<A(¼„ƒ#…†·•½e· A.EFv>JK'A)6GEMFv¦<BDA73¨ž+A.Š=<A.EŸI86GA!s•=<A
EMF};<A)rQ>?A£RTN R¿¾£~R¿¼(N RÀOR›š<R¿¾A.E =<>Q5:><@'6MŠ¤5´3
Á 3!¶ Š–Ž<BvA.57><;<@:A.B}A­MA.6M9.Fv9)Fv@:¹:9.@'Š–Ž<6M=<A.¦§As•=<A1Bv57E EGFDH'=<FDA.>JI8A)E!EM@'>£><@'6GŠ*57E)X
5J°wRTN R~ à « Z N“U³”_N ‡ ° ‡ ”]«yN“U \ N ‡ ° ‡ NăŅ† ‡ O«¯¦’°zRTN R ~ˆŠ¤5ÀÆ}« Z§Ç N“U Ê ÇÉÈ7Ç h N“U \¢Z N ‡ Ç °
Nªƒw…†¡‡ O³«y9‚°RTN R ~ Ç N“U³”_N ‡ \ N’Ê Ç ” Ç N“U \oZ N ‡ Ç ” Ç h N“UP”_N ‡ \ N’Ê Ç Nƒw…† 3
Ë?3!4&57685SBv5¤EGFDH'=<FvA)>JI8A.EÌEG=<9)A.EGFD@'><A)EC;?AK'A.9LI8@'6MA.EÍ · OÎA.>?9.=<A)>•IG6GA:A.B&BÏИ Š*FpI8AÍPWA.>wB{5eEÌ><@'6MŠ¤5eE
R›š<R)U)ORš<R ‡ V R¡Ê š<RLW3
‡Ï·‚· Ô Ô U Ü[Ý Õ•· U Ü[Ý Õ•· ”lU â Ü[Ý
«¯5J°›Í · ~ˆÑÒÓ · ·eÖÂÕtÔ<‡ ‡ Þ «¯¦’°›Í · ~ ÑÒÓ U ß· · ÔÔ<‡ U Þ «¯9‚°¡Í · ~ÑÒÓß U ·‚· ÖÏÔ Ô<‡ U Þ
×vØyÙ · Ú ·ÀÛ Ú Õ Ûáà · Ú ·‚Ô Û Ú Õ ÛáෂÚÖ · Ô Û
ã?3ä«i5'°£ºtA 5*å=?>uA)EGŽQ579)Fv@:K'A)9)I8@76GF{5eBÎ9.@'>£><@'6MŠ¤5R›š<RÀ3¨ž+A.Š=?A.EMIG6GA»s•=<A
Ç R¿æ R \ R¿ç}R Ǖè R¿æ \ ç}R ŽQ57685I8@t;<@»æ*V:çA.>¤å¨é
«¯¦Q°™¶ Š–Ž<BvA.57><;<@êA.B»Ž<=<>JIG@57>JI8A)6GFv@76£;?A.Š=<A)EMIG6GA s•=<A—EGFCæ · 9.@'>JK'A)6GH'A5læ§WA)>•IG@'><9)A.E
BDFvŠ · ë WmR¿æ · R ~R¿æ“W–RÀ3µCì57BvABv56GA)9'И Ž?6G@t9 5J¸
2 f 3!ž+A)Š=<A.EŸI86MACs•=<AC9 5e;Q5¬=<>Q5:;<A»B{57E EMFvH'=<FDA.>JI8A)E EG@'>£><@'6MŠ¤57E A)>£í ‡
«i5'°RÂîœR ~gŠ*5aÆtï{ðeñ òGó U¯ô Ç î³«yõM° Ç 3
«¯¦Q°RÂîœR ~gŠ*5aƧ€ Ç îP« f ° ÇÉÈ7Ç î³«Ÿ2Àö Z ° DzÈ7Ç î³«®2À° Ç ‰•3
2
÷¥ 57BD9.=<BDASRÀ2”_õ}”_õ ‡ RÌA.>£9 57;Q5:=<>Q5¬;<A»A)E857E!><@'6MŠ¤57E)3
2'2'3(º´A.5wøù=?>Q5¦Q57EMA£@76G;<A)>Q57;Q5#;<A)B …†»úÏA.EMŽQ579.FD@K'A)9)I8@76GF{5eB åä;<A;<FDŠ*A)><EGF @'˜ >âœO¨VEMA 5wûLügA)B
FDEG@'Š–@'6Mr<EGŠ–@;<AC9.@t@'6G;<A)>Q57;Q5eE.3
«i5'°£ºt=<Ž“@7><HJ5»st=?A:R“š RA.E¨=<>Q5»><@'6MŠ¤5+A)>S…† ·?3œž+A.Š=?A.EMIG6GA s•=<A:Rýš RLþœO•;<ALrQ><FD;Q5Š*A);<F{57>JIGA
R¿ç}RLþª~ÿRLû)ü¨«yç´°ÀRÀO<A)E(=<><5–><@76GŠ*5A.>ªå*3
«¯¦Q°£ºt=<Ž“@7><HJ5*s•=<A™R›š’RLþ—A.E!=<>Q5:U ><@'6GŠ*5:A.>#å:3÷ž+A)Š=<A.EŸI86MA1U s•=<A™R÷š<RÌA.E!=<>Q5*>?@'6GŠ*5¬A.>
…† ·*EGFEMA;<A)r<><AR¿æ R»~ RLû üÕ «¯æ“°‚RLþ3–«  A.9.=?A.6G;?A¬st=?A:û üÕ «yæý° A.E+A)B*=<˜ ><FD9.@S窃_åÄIT57B
s•=<A1ûLü›«yç´° ~gæP3É°
2 Z 3(º´A.5™å~ í ‡ VuEGA.5£ø~Ā•2  2÷”êõ Gõ&”êõ®‡ ‰–=<>Q5S¦Q57EMA¬@'6GÊ;?A.>Q57;<5™;?A:å*3C¶ ><9.=<A)>JI86GA–ŽQ576G5
9.57;Q5=<>Q5;?ASB{57E1><@'6MŠ¤57EuR̚}R)U)O+R̚ÎR ‡ VŒR(šPRLWÅA)>ꅆ O =<>Q5ªxe@'˜ 6MŠ=<B{5£ŽQ576G5B{5ª><@76GŠ*5
R + ” TõP”lû¿õ ‡ RLþzA.>ªås•=<A»EMA1;<ALrQ><F @–˜ A)>«¯5J°¡;<A)B³A­MA.6M9.Fv9)Fv@2'273
2 h 3(º´A.57>ªå Vøÿ9)@'Š*@¬A)>uA)B}A®­MA)6G9.FD9.FD@*5e>•IGA.6MFv@'6)O?VEGA.5wR›š<RLþ—Bv5><@'6MŠ¤5¬;<ALrQ><Fv;<5:Ž§@'6
U U 
RÂîRLþª ~ 
ò yõM° ‡ õ  ‡ é
¶ ><9)=<A.>JI86MA¤=<><5£xe@'˜ 6MŠ=<B{5SŽ<57685™B{5><@'6MŠ¤5R Y ¢û `pb RA.>o…† Ê s•=<A–EGA–;<A)r<><F‚@£˜ A.>m«¯¦’°+;<A)B
A­MA.6G9)Fv9)Fv@™2'2'3
2  3(º´A.5lR!šýR¤X³…† ‡  …† ;<ALrQ><Fv;<5£Ž§@'6™RTN R:~Ċ*5aƧ€ Ç Z N“U ÇÉÈ7Ç h N ‡ Ç ‰•3ž+A.Š=?A.EMIG6GA–s•=<A—R!šýR¬A)E
=<><5™>?@'6GŠ*5¤V#H'6G5erQs•=<A:B{5S¦§@'B{5S=?><FDI8576GFv5?34 6G=?A.¦§A–s•=<A¬I857B><@'6MŠ¤5A)ECA)s•=<FDKe57BDA.>JI8A:5B{5eE
><@76GŠ*57E2'O’‹ V Z 3
2 ^ 3 Â;<A)Š 57>JI8A)6GFD@'6 Ž“A)6G@–A.>ª…† Ê 9.@'>oRTN R ~ Z§Ç N“U Ç ” h“Ç N ‡ Ç ” Ç N’Ê Ç 3
2‚¹?3!ž+ALI8A.6MŠ*FD>Q576 EGF³B{5eE!>?@'6GŠ*57E!;<ABD@'E(;?@'EÌA­MA.6G9)Fv9)Fv@'E(57>JI8A)6GFv@76GA.E!EG@7>><@'6MŠ¤57E!Fv>?;<=<9.FD;Q57E!Ž“@76
=<>£ŽÎ3[F 3
2 Á 3!ž+A)Š=<A.EŸI86MAzs•=<ARTN R~ à N ‡U ”_N“UMN ‡ ”_N ‡‡ A.E=?>Q5l>?@'6GŠ*5A.>]…† ‡.3q»6G5erQs•=<AwB{5¦§@'Bv5
=<>?FDIT5e6GF{5+V–Ž<6M=<A.¦§AÌs•=<AÌI857B’><@'6MŠ¤5»A.E÷A)s•=<FDKe57BDA.>JI8A(51Bv51><@'6MŠ¤5 Z «iº´=<H'A)6GA)><9.Fv5?XA.>?9.=<A)>•IG6GA
=<>£ŽÎ3[F 3I857BÎs•=<AR¡š<RÌEGA.5–Bv5><@'6GŠ*5¬Fv><;?=<9.FD;Q5?3²°
2‚Ë?3!ž+A)Š*@'EŸI86G576*s•=<AlR Y N“U N ‡ `cb Rz~ à  N ‡U ” Z JN“UMN ‡ ”  N ‡‡ A.E¤><@'6GŠ*5—A)>k…† ‡SEMF(VlEQ@'˜ BD@ EMF
\   m 3
2‚ã?3ä«i5'°£ºtA 5  X•å   =?>Q51I86G57><EŸxy@'6GŠ*579.F @'˜ >¤BDFv><A.57B’Fv>JV'A)9)IGFDKe51VzRœšJR  =<>Q5>?@'6GŠ*51A)> 3
4 6M=<A.¦§A+s•=<AÌEMF“EMAÌ;<ALrQ><A¤R¿ç}RLþªŠ–A.;?F{57>JI8A:R¿ç}RLþ~ÿ R «yç?°‚R ¤OPR&šeRLþªA.E›><@'6GŠ*5»A.>™å:3
«¯¦Q!° !EGAuA.B!Ž<=<>JI8@ 57>JI8A.6MFv@'6–ŽQ576G5z;?A.Š–@'EMIG68576–s•=<AªBv57E:EGFvH7=<FvA)>•IGA.E–ALÆ´Ž<6MA.EGFD@'><A)ES;<A)r<><A.>
>?@'6GŠ*57E÷A)>™í ‡ «yA.E ;<A)9.FD6.O<FD;<A.>JI8FprQs•=<"A  O?Bv51I86G57><EMxy@76GŠ*579.F @'˜ #> gVSB{51><@'6GŠ*5uR¨š•R –°
F{°1RÂîR÷~ Ç î³« Uf ° Ç ” Ç îP«Ÿ2À° Ç ” Ç î³« Z ° Ç O FvF{°CRÂîR÷~]Š*5aƒ€ Ç î³« f ° DzÈ7Ç îP« \ 2À° DzÈ7Ç î³«®2À° Ç ‰•O
FDFvF{°»RÂîR÷~%$ ò õ Ç î³« õM° Ç õ¿3
Z
Ejercicios Adicionales a las Prácticas 5 a 7

1. Dada T ∈ L(V ) definida por T (v1 ) = 7v1 − 4v2 y T (v2 ) = 2v1 + v2 , con B = {v1 ; v2 } una base de V ,
se pide:
(a) encontrar, si existe, una base C de V tal que la representación matricial de T en esa base sea
diagonal.
(b) Calcular [T k ]B para k ≥ 1.

2. (a) Defina una transformación lineal T : |R3 → |R3 tal que T 6= ±I, S = {x ∈ |R3 : x1 + x2 − x3 = 0}
sea invariante por T y (T (x), T (y)) = (x, y) para todo x, y ∈ |R3 .
(b) Hallar [T ]E con E la base canónica de |R3 .
 
0 1 0
3. Sea T ∈ L(P2 ) con [T ]B =  0 0 1  y B = {1 + t; 1 − t; t2 }. (a) Determinar para qué valores de
−a 1 a
a resulta diagonalizable T .
(b) Para a = 2, encontrar una base B 0 de P2 tal que la representación matricial de S = T 3 − T + 2I
sea diagonal. Hallar [S]B 0 .

4. Sea T ∈ L(P2 ) definida por T (p) = t3 p00 (t) − 6αp0 (t) − 2tp(t), (α ∈ |R).
(a) Demostrar que existe una base B de P2 compuesta por autovectores de T si y sólo si α > 0.
(b) Para α = 1, encuentre una base B de P2 tal que [T ]B sea diagonal.

5. (a) Definir T ∈ L(R| 3 ) tal que: T sea diagonalizable, S = {x ∈ |R3 : x1 + x2 + x3 = 0} y S ⊥ sean


invariantes por T y det([T ]B ) = −4 y traza([T ]B ) = 3 para toda base B de |R3 .
(b) Demuestre que si A ∈ C | n×n es diagonalizable y 0 no es autovalor de A, entonces A es inversible y
B = αAn + β(A−1 )n con α, β ∈ C | y n ∈ IN es diagonalizable.
 
3 −1 0
6. (a) Sea C = B(A3 − A)B −1 , con B ∈ C | 3×3 inversible y A = 1  −1 3 0 . Calcule los autovalores
4
−1 1 2
de C y determine si C es diagonalizable.
(b) Considere la matriz C de (a) con B = I y calcule lı́mn→∞ C n v para v ∈ |R3 .
| 2×2 ) definida por T (A) = A + AT .
7. Sea T ∈ L(R
(a) Encontrar, si existe, una base B de |R2×2 en la cual [T ]B sea diagonal.
1
(b) Demostrar que para n ∈ IN , 2n−1 Tn = T.

8. (a) Encontrar A ∈ |R3×3 simétrica e indefinida tal que A2 + 3A = 4I y que [1 1 2]T sea un autovector
de A.
(b) Demostrar que Q(x) = xT Ax, con A la matriz hallada en (a) verifica −4kxk2 ≤ Q(x) ≤ kxk2
∀x ∈ |R3 .
· ¸
6 2
9. Considere la ecuación 6A2 + αI = , (α ∈ |R ). (a) Demuestre que si A ∈ C | 2×2 verifica la
2 3
ecuación, entonces A es diagonalizable. (b) Determine para qué valores de α la ecuación admite una
solución A ∈ |R2×2 simétrica y definida negativa y, para cada uno de esos valores, halle una solución
en esas condiciones.
10. (a) Encontrar A ∈ |R3×3 simétrica tal que S = {x ∈ |R3 : x1 + x2 = 0} sea el autoespacio asociado al
menor autovalor de A, det(A) = 2 y B = A3 − A2 + A − I no sea inversible.
(b) Considere la forma cuadrática Q(x) = xT Ax con A la matriz hallada en (a). Calcule máxkxk2 =1 Q(x),
mı́nkxk2 =1 Q(x) y halle los x en los que se alcanza cada extremo.

11. (a) Hallar A ∈ |R3×3 simétrica e indefinida tal que máxkxk2 =1 xT Ax = 3, Ax ∈ S para todo x ∈ S =
{x ∈ |R3 : x1 + x2 = 0} y 6 sea autovalor de A2 + A.
(b) Demuestre que si A ∈ |Rn×n es simétrica y xT (A2 − 4I)x < 0 para todo x ∈ |Rn con x 6= 0, entonces
todos los autovalores de A se encuentran en el intervalo (−2, 2).
12. (a) La energı́a cinética de un cuerpo rı́gido que rota con velocidad angular ω (ω ∈ |R3 ) viene dada
por la expresión E(ω) = 21 (ω T M ω), donde M es una matriz simétrica y definida positiva denominada
 
6 2 0
tensor de inercia. Suponiendo M =  2 3 0  y que el cuerpo rota con velocidad angular unitaria
0 0 2
(kwk2 = 1), se pide hallar el valor mı́nimo de E, y las velocidades para las cuales se alcanza tal
extremo.
(b) Suponga que Q : |R3 → |R es una forma cuadrática tal que Q([1 1 1]T ) = 3 y Q([1 1 0]) = −4. Si λM
y λm son, respectivamente, el máximo y el mı́nimo autovalor de la matriz asociada a Q, determinar
cuánto pueden valer como mı́nimo λM y como máximo λm .
p
13. (a) Demostrar que en |R2 , kxk = 2x21 + 2x1 x2 + 2x22 es una norma inducida por un producto interno.
(b) Para la norma k · k del punto (a), hallar máxkxk2 =1 kxk, mı́nkxk2 =1 kxk y graficar la bola unitaria.
14. (a) Encuentre los puntos de la curva 6x21 + 4x1 x2 + 3x22 = 1 más cercanos al origen.
(b) Demuestre que si A ∈ |Rn×n es simétrica y definida positiva entonces existe B ∈ |Rn×n simétrica y
definida positiva tal que A = B T B.
15. (a)Encuentre los puntos de la curva

2x21 + 6x22 − 2 5x1 x2 = 1
de norma Euclı́dea máxima.
(b) Demuestre las siguientes afirmaciones:
i) Si A es antisimétrica (A = −AT ) entonces A2 es simétrica y semidefinida negativa.
ii) Si A es simétrica y definida positiva y B es semejante ortogonalmente a A, entonces B también es
simétrica y definida positiva.
x2 x2
16. (a) Encontrar, entre todos los rectángulos de vértices ±(x1 , x2 ), ±(x1 , −x2 ) con 91 + 42 = 1, el de área
máxima.
(b) Sea (x, y)G = xT Gy con G ∈ |Rn×n un producto interno en |Rn . Demostrar que existe una matriz
simétrica y definida positiva A ∈ |Rn×n tal que (x, y)G = (Ax, Ay), ∀x, y ∈ |Rn , con (·, ·) el producto
interno canónico de |Rn .
  
1 −2 0 2 0 · √ √ ¸
1/√2 1/√2
17. (a) Dada A =  1 1 −1   0 2  determinar el rango de A, sus valores
1/ 2 −1/ 2
1 1 1 0 0
¡ ¢
singulares y la matriz de proyección sobre Nul AT . (Sugerencia: obtenga una DVS de A sin calcular
A.)
(b) Dada A ∈ |Rn×n decidir si son ciertas las siguientes afirmaciones:
i) El producto de los valores singulares de A es el determinante de A.
ii) Si todos los valores singulares de A valen 1 entonces A es ortogonal.
18. (a) Sea T ∈ L(R | 3 , |R2 ), con T (x) = [x1 + x2 + x3 x1 − x2 + x3 ]T . Dado b = [1 2]T , hallar entre los
x ∈ |R3 que minimizan kT (x) − bk2 , el de longitud mı́nima.
(b) Sea A ∈ |Rn×m . Demostrar las siguientes afirmaciones: i) si las columnas de A son ortonormales
entonces los valores singulares de A son todor iguales a 1; ii) Si las filas de A son ortonormales, entonces
los valores singulares no nulos de A son iguales a 1. (Sugerencia: trabaje con AT ).

19. (a) Sea A = U ΣV T con Σ ∈ |R3×3√diagonal, √ Σ11 = 2, Σ22 = 1 y Σ33 = 0, U y √ V ortogonales


√ y tales
que la última columna de V es [1/ 2 0 1/ 2]T y la última columna de U es [1/ 5 2/ 5 0]T . Hallar
las matrices de proyección a los subespacios col(A) y nul(A) y calcular máxkxk2 =1 kAxk2 .
(b) Decidir, justificando la respuesta, si las siguientes afirmaciones son ciertas:
i) Si B es ortogonal A y AB tienen los mismos valores singulares.
ii) Si A y B tienen los mismos autovalores, entonces A y B tienen los mismos valores singulares.
  
1 1 1 2 0 · ¸
    1 1
20. (a) Sabiendo que A = 1 −1 1 0 1 ,
1 −1
1 0 −2 0 0
i) encontrar los valores singulares de A; ii) hallar la solución por cuadrados mı́nimos de longitud
mı́nima de Ax = b con b = [1 1 2]T .
(b) Decidir si cada una de las siguientes afirmaciones es verdadera o falsa, justificando la respuesta:
i) Para A ∈ |Rm×n , AA+ = Pcol(A) y A+ A = PNul(A)⊥ , siendo A+ la seudoinversa de Moore-Penrose de
A.
ii) Si A es inversible, su inversa es igual a A+ .

21. (a) Sean T : |R2 → |R3 definida por T (x) = [x1 + x2 2x1 + 2x2 x1 + x2 ]T y b = [4 2 4]T . Halle de entre
todos los x ∈ |R3 que minimizan kb − T (x)k2 el de norma Euclı́dea mı́nima.
(b) Suponga que A = U ΣV T es una descomposición en valores singulares de A ∈ |Rm×n . i) Demuestre
que las columnas de U son autovectores de AAT y que los autovalores no nulos de AAT coinciden con
los autovalores no nulos de AT A; ii) deduzca de i) que los valores singulares no nulos de A y de AT
coinciden y que máxkxk2 =1 kAxk2 = máxkzk2 =1 kAT zk2 .

22. Sea T ∈ L(R| 2 , |R3 ) definida por T (x) = [x2 x1 + x2 x1 ]T . (a) Hallar bases ortonormales de |R2 y |R3 de
modo tal que la representación matricial
√ de T en esas bases tenga ceros fuera de la diagonal principal.
(b) Demostrar que 1 ≤ kT (x)k2 ≤ 3 para todo x ∈ |R2 con kxk2 = 1.
· ¸
T 1 2 −1
23. (a) Sea A = BP con B = y P ∈ |R3×3 ortogonal. Calcular los valores singulares de A
2 4 −2
y hallar bases ortonormales de col(A)⊥ y Nul(A).
(b) Sea A ∈ |Rn×n . Encontrar la relación entre det(A) y el producto de los valores singulares de A.

24. (a) Demostrar que si k · ka y k · kb son dos normas en un espacio vectorial V entonces k · kc , definida
por kvkc = kvka + kvkb ∀v ∈ V , es norma enpV .
(b) Demuestre que kxk = máx{|x1 |, |x2 |} + x21 + 4x22 es norma en |R2 . (Sugerencia: use a)).

25. (a) Encontrar los | 2


p valores de α ∈ R para los cuales kxk = |x1 − αx2 | + |αx1 − x2 | es norma en R .
|
(b) Sea kxk = 2x21 − 2x1 x2 + 2x22 . Demuestre que k · k es norma en |R2 y grafique la bola unitaria.

26. (a) Dadas k · ka y k · kb normas en un espacio vectorial V , demostrar que kxk = αkxka + βkxkb es una
norma en V si α ≥ 0, β ≥ 0 y α y β no son simultáneamente
p nulos.
(b) Demostrar que k[x1 x2 ]T k = 3 máx{|x1 |, |x2 |} + 5 3x21 + 2x1 x2 + 2x22 es una norma en |R2 .
27. (a) Demostrar que si T ∈ L(V, W ) es inyectiva y k · kW es norma en W entonces k · kV , definida por
kvkV = kT (v)kW , es norma en V .
(b) Demostrar que kpk = |p(1)| + |p(0)| + |p(−1)| es norma en P2 pero no en P3 .(Sugerencia: use (a)).

28. (a) Encontrar todos los valores de k ∈ |R para los cuales kxk = |3x1 + x2 + x3 | + |kx1 + (1 + k)x2 −
x3 | + |x1 + x3 | es norma en |R3 .
(b) Sea B = {v1 ; . . . ; vn } una base de un espacio vectorial real V y sea k·k una norma en |Rn . Demostrar
que kvkB = k[v]B k es norma en V .

29. (a) Sea T ∈ L(P1 , P2 ), T (p) = (1 − t)(p + p0 ). Mostrar que kpka = |g(0)| + |g(1)| + 2|g(−1)| con
g = T (p), define una norma en P1 .
(b) Demostrar que en P1 , la norma kpkb = máx{|g(0)|, |g(1)|, |g(−1)|}, con g = T (p), es equivalente a
la norma k · ka .

Vous aimerez peut-être aussi